You are on page 1of 47

LABOR STANDARDS CASES reinstated.

Hacienda Fatima being guilty of unfair labor


Labor Arbiter Augusto L. Villanueva practice are further ordered to pay the union the sum of
SSCR P10,000.00 as moral damages and P5,000.00 as exemplary
nd
2 Sem A.Y. 2015-2016 damages.
Hacienda Fatima appealed the decision of the NLRC
Dan Gloria// Kat Ramirez// Jessica Bernardo// Joseph De
before the CA.
Mesa// Arvin Figueroa// ChriszelQueano// Marvin Villardo
Jr.// Bowdy Sales
CA:The CA affirmed the decision of the NLRC. Thus,
Hacienda appealed the decision of the CA before the SC.

ISSUE:WON the respondents, admittedly seasonal workers,


were regular employees, contrary to the clear provisions of
Chapter III Kinds of Employment/ Employee Article 280 of the Labor Code, which categorically state that
Classification seasonal employees are not covered by the definition of
regular employees under paragraph 1, nor covered under
1. HACIENDA FATIMA and/or PATRICIO VILLEGAS, paragraph 2 which refers exclusively to casual employees who
ALFONSO VILLEGAS and CRISTINE SEGURA, have served for at least one year.
petitioners, vs. NATIONAL FEDERATION OF
SUGARCANE WORKERS FOOD AND GENERAL HELD:YES.Contrary to petitioners contention, the CA did
TRADE, respondents. not err when it held that respondents were regular employees.
G.R. No. 149440. January 28, 2003. Article 280 of the Labor Code, as amended, states:
Art. 280. Regular and Casual Employment.The
FACTS:When the newly elected officers of the National provisions of agreement to the contrary notwithstanding and
Federation of Sugarcane Workers Food and General Trade regardless of the oral agreement of the parties, an
was certified as the collective bargaining representative in the employment shall be deemed to be regular where the employee
certification elections, Hacienda Fatima, refused to sit down has been engaged to perform activities which are usually
with the union for the purpose of entering into a collective necessary or desirable in the usual business or trade of the
bargaining agreement. Moreover, the workers were not given employer, except where the employment has been fixed for a
work for more than one month. In protest, the workers staged specific project or undertaking the completion or termination
a strike which was however settled upon the signing of a of which has been determined at the time of the engagement of
Memorandum of Agreement which stipulated that the parties the employee or where the work or services to be performed is
will initially meet for CBA negotiations on the 11th day of seasonal in nature and the employment is for the duration of
January 1991 and will endeavor to conclude the same within the season. An employment shall be deemed to be casual if it
thirty (30) days; that the management will give priority to the is not covered by the preceding paragraph: Provided, That,
women workers who are members of the union in case work any employee who has rendered at least one year of service,
relative or amounting to gahitand dipol arises; and that the whether such service is continuous or broken, shall be
management will provide fifteen wagons for the workers and considered a regular employee with respect to the activity in
that existing workforce prior to the actual strike will be given which he is employed and his employment shall continue while
priority. such activity exist.
However, alleging that the workers failed to load the For respondents to be excluded from those classified as
fifteen wagons, Hacienda Fatima reneged on its commitment regular employees, it is not enough that they perform work or
to sit down and bargain collectively. Instead, Hacienda services that are seasonal in nature. They must have also been
employed all means including the use of private armed guards employed only for the duration of one season. The evidence
to prevent the organizers from entering the premises. proves the existence of the first, but not of the second,
Moreover, starting September 1991, the Hacienda did not any condition. The fact that respondents repeatedly worked as
more give work assignments to the workers forcing the union sugarcane workers for petitioners for several years is not
to stage a strike on January 2, 1992. But due to the denied by the latter. Evidently, petitioners employed
conciliation efforts by the DOLE, another Memorandum of respondents for more than one season. Therefore, the general
Agreement was signed by the workers and the Hacienda. rule of regular employment is applicable. WHEREFORE, the
However, Hacienda Fatima again reneged on its commitment. Petition is hereby DENIED and the assailed Decision
Thus, the workers filed a complaint against the Hacienda. AFFIRMED. Costs against petitioners.SO ORDERED.

LABOR ARBITER:The labor Arbiter rendered judgment in


favor of the Hacienda Fatima for the reason that the said 2. ASSOCIATION OF TRADE UNIONS (ATU),
workers refused to work or were being choosy in their work. RODOLFO MONTECLARO and EDGAR JUESAN,
The workers appealed the said judgment before the NLRC. petitioners, vs. HON. COMMISSIONERS OSCAR N.
ABELLA, MUSIB N. BUAT, LEON GONZAGA, JR.,
NLRC:The NLRC rendered a judgment reversing the Labor ALGON ENGINEERING CONSTRUCTION CORP.,
Arbiters decision and declaring the workers were illegally ALEX GONZALES and EDITHA YAP, respondents.
dismissed. Furthermore the Hacienda Fatima was ordered by
the NLRC to reinstate the workers to their previous position G.R. No. 100518. January 24, 2000.
and to pay full backwages from September 1991 until

Page 1
Dan Gloria// Kat Ramirez// Jessica Bernardo// Joseph De Mesa// Arvin Figueroa// ChriszelQueano// Marvin Villardo Jr.// Bowdy Sales
contrast, project employees are those whose employment has
FACTS:Algon Engineering Construction Corporationg is a been fixed for a specific project or undertaking the completion
domestic corporation engaged in road construction projects of or termination of which has been determined at the time of the
the government. From 1968 to 1989, it engaged the services of engagement of the employee, or where the work or services to
the following workers to work on various projects on different be performed is seasonal in nature and the employment is for
dates: Rodolfo Monteclaro (mechanic), Edgar Juesan(painter), the duration of the season. Furthermore, Policy Instruction No.
VictorioLunzaga (tanker driver), Alfredo Jalet (batteryman), 20,which was in force during the period of petitioners
JulitoMacabodbod (trailer helper), Ramon Tabada (carpenter), employment, stated: Project employees are those employed
RemsyAsensi (machinist), Armand Acero (helper mechanic), in connection with a particular construction project. Non-
LorditoTatad (painter helper), Rogelio Tantuan (painter), project (regular) employees are those employed by a
TeodoroTabio (checker), GemudoAsejo (electrician), Roland construction company without reference to any particular
Olivar (latheman), ValerianoMijas (driver), Jose Noval project. Project employees are not entitled to termination pay
(welder), FelimonLagbao (mechanic), Pedro Roche (head if they are terminated as a result of the completion of the
welder), and JustinianoSollano (carpenter). Their contracts project or any phase thereof in which they are employed,
indicate the particular project they are assigned, the duration regardless of the number of projects in which they have been
of their employment and their daily wage. employed by aparticular construction company. Moreover, the
In February 1989, the above named workers joined company is notrequired to obtain clearance from the Secretary
petitioner union as members. Accordingly, petitioner union of Labor inconnection with such termination. What is required
filed a petition for certification election with the regional of thecompany is report to the nearest Public Employment
office of the labor department. Respondent company opposed Office forstatistical purposes.
the petition on the ground that the workers were project In the case at bar, the contracts of employment of
employees and therefore not qualified to form part of the rank thepetitioners attest to the fact that they had been hired
and file collective bargaining unit. Not for long, the Med- forspecific projects, and their employment was
Arbiter dismissed the petition for certification election. On coterminouswith the completion of the project for which they
appeal, the Secretary of Labor and Employment reversed the had been hired.Said contracts expressly provide that the
Med-Arbiters decision and ordered the immediate holding of workers tenure of employment would depend on the duration
a certification election. of any phase ofthe project or the completion of the awarded
Shortly thereafter, respondent company terminated governmentconstruction projects in any of their planned
the employment of aforementioned workers owing to the phases.Further, petitioners were informed in advance that
completion of its projects or the expiration of workers saidproject or undertaking for which they were hired would
contracts. Thus, the aggrieved workers filed with the Regional endon a stated or determinable date. Besides, publicrespondent
Arbitration Branch of the NLRC their individual complaints noted that respondent company regularlysubmitted reports of
against private respondent company for illegal dismissal, termination of services of projectworkers to the regional office
unfair labor practice, underpayment of wages, 13th month pay, of the labor department asrequired under Policy Instruction
holiday pay and overtime pay. They also sought reinstatement No. 20. This compliancewith the reportorial requirement
with back wages. confirms that petitionerswere project employees.Considering
that petitioners were project employees,whose nature of
ISSUE:WON the said workers were illegally dismissed. employment they were fully informedabout, at the time of
their engagement, related to a specificproject, work or
HELD:NO.The Labor Code defines regular, project and undertaking, their employment legallyended upon completion
casual employees as follows: ART. 280. Regular and Casual of said project. The termination oftheir employment could not
EmploymentThe provisions of written agreement to the be regarded as illegaldismissal.
contrary notwithstanding and regardless of the oral agreement WHEREFORE, the instant petition is DISMISSED,
of the parties, an employment shall be deemed to be regular and the assailed RESOLUTION of respondent NLRC dated
where the employee has been engaged to perform activities May 17, 1991, is AFFIRMED. No pronouncement as to costs.
which are usually necessary or desirable in the usual business SO ORDERED.
or trade of the employer, exceptwhere the employment has
been fixed for a specific project orundertaking the completion
or termination of which has beendetermined at the time of the 3. ABS-CBN BROADCASTING CORPORATION,
engagement of the employee or wherethe work or services to petitioner, vs. MARLYN NAZARENO, MERLOU
be performed is seasonal in nature and theemployment is for GERZON, JENNIFER DEIPARINE, and JOSEPHINE
the duration of the season. And employment shall be deemed LERASAN, respondents.
to be casual if it is not covered by the preceding paragraph:
Provided, That, any employee who has rendered at least one G.R. No. 164156. September 26, 2006.
year of service, whether such service is continuous or broken,
shall be considered a regular employee withrespect to the FACTS: ABS-CBN employed, for an average of 5
activity in which he is employed and hisemployment shall years,Nazareno, Gerzon, Deiparine, and Lerasan as
continue while such activity exists. production assistants (PAs) on different dates. They were
Thus, regular employees are those who have been assigned at the news and public affairs, for various radio
engaged to perform activities which are usually necessary or programs in the Cebu Broadcasting Station, with a monthly
desirable in the usual business or trade of the employer even if compensation of P4,000. They were issued ABS-CBN
the parties enter into an agreement stating otherwise.In

Page 2
Dan Gloria// Kat Ramirez// Jessica Bernardo// Joseph De Mesa// Arvin Figueroa// ChriszelQueano// Marvin Villardo Jr.// Bowdy Sales
employees identification cards and were required to work for ART. 280. REGULAR AND CASUAL
a minimum of eight hours a day, including Sundays and EMPLOYMENT.The provisions of written agreement to the
holidays. The PAs were under the control and supervision of contrary notwithstanding and regardless of the oral
Assistant Station Manager Dante J. Luzon, and News Manager agreement of the parties, an employment shall be deemed to
Leo Lastimosa. be regular where the employee has been engaged to perform
On December 19, 1996, ABS-CBN and its Rank-and- activities which are usually necessary or desirable in the usual
File Employees executed a Collective Bargaining Agreement business or trade of the employer except where the
(CBA) to be effective during the period from December 11, employment has been fixed for a specific project or
1996 to December 11, 1999. However, since petitioner refused undertaking the completion or termination of which has been
to recognize PAs as part of the bargaining unit, respondents determined at the time of the engagement of the employee or
were not included to the CBA. where the work or services to be performed is seasonal in
On October 12, 2000, respondents filed a Complaint nature and the employment is for the duration of the season.
for Recognition of Regular Employment Status, In Universal Robina Corporation v. Catapang,the
Underpayment of Overtime Pay, Holiday Pay, Premium Pay, Court reiterated the test in determining whether one is a
Service Incentive Pay, Sick Leave Pay, and 13th Month Pay regular employee: The primary standard, therefore, of
with Damages against the petitioner before the NLRC. determining regular employment is the reasonable connection
between the particular activity performed by the employee in
LABOR ARBITER:On July 30, 2001, the Labor Arbiter relation to the usual trade or business of the employer. The test
rendered judgment in favor of the respondents, and declared is whether the former is usually necessary or desirable in the
that they were regular employees of petitioner as such, they usual business or trade of the employer. The connection can be
were awarded monetary benefits. However, the Labor Arbiter determined by considering the nature of work performed and
did not award money benefits as provided in the CBA on his its relation to the scheme of the particular business or trade in
belief that he had no jurisdiction to interpret and apply the its entirety. Also, if the employee has been performing the job
agreement, as the same was within the jurisdiction of the for at least a year, even if the performance is not continuous
Voluntary Arbitrator as provided in Article 261 of the Labor and merely intermittent, the law deems repeated and
Code. ABS-CBN filed a motion for reconsideration, which the continuing need for its performance as sufficient evidence of
Labor Arbiter denied. Petitioner forthwith appealed the the necessity if not indispensability of that activity to the
decision to the NLRC, while respondents filed a partial appeal. business. Hence, the employment is considered regular, but
only with respect to such activity and while such activity
NLRC:NLRC modified the decision of the Labor Arbiter exists.
stating that respondents were entitled to the benefits under the Not considered regular employees are project
CBA because they were regular employees who contributed to employees, the completion or termination of which is more
the profits of petitioner through their labor. Petitioner filed a or less determinable at the time of employment, such as those
motion for reconsideration, which the NLRC denied. employed in connection with a particular construction project,
Petitioner thus filed a petition for certiorari under Rule 65 of and seasonal employees whose employment by its nature is
the Rules of Court before the CA. only desirable for a limited period of time. Eventhen, any
employee who has rendered at least one year ofservice,
CA:On February 10, 2004, the CA rendered judgment whether continuous or intermittent, is deemedregular with
dismissing the petition. The appellate court stated that respect to the activity performed and whilesuch activity
respondents are not mere project employees, but regular actually exists. It is of no moment that petitioner hired
employees who perform tasks necessary and desirable in the respondents as talents. The fact that respondents received
usual trade and business of petitioner and not just its project pre-agreed talent fees instead of salaries, that they did not
employees. Moreover, the CA added, the award of benefits observe the required office hours, and that they were permitted
accorded to rank-and-file employees under the 1996-1999 to join other productions during their free time are not
CBA is a necessary consequence of the NLRC ruling that conclusive of the nature of their employment. Respondents
respondents, as PAs, are regular employees. Finding no merit cannot be considered talents because they are not actors or
in petitioners motion forreconsideration, the CA denied the actresses or radio specialists or mere clerks or utility
same in a Resolutiondated June 16, 2004.Petitioner thus filed employees. They are regular employees who perform several
before the SC a petition for review onCertiorari. different duties under the control and direction of ABS-CBN
executives and supervisors.
ISSUE:WON the respondents, as production assistants, are Thus, there are two kinds of regular employees
considered regular employees. under the law: (1) those engaged to perform activities which
are necessary or desirable in the usual business or trade of
HELD:YES.We agree with respondents contention that the employer and (2) those casual employees who have
where a person has rendered at least one year of service, rendered at least one year of service, whether continuous
regardless of the nature of the activity performed, or where the or broken, with respect to the activities in which they are
work is continuous or intermittent, the employment is employed.The law overrides such conditions which are
considered regular as long as the activity exists, the reason prejudicialto the interest of the worker whose weak
being that a customary appointment is not indispensable bargainingsituation necessitates the succor of the State.
before one may be formally declared as having attained Whatdetermines whether a certain employment is regular or
regular status. Article 280 of the Labor Code provides: otherwise is not the will or word of theemployer, to which the
worker oftentimes acquiesces, muchless the procedure of

Page 3
Dan Gloria// Kat Ramirez// Jessica Bernardo// Joseph De Mesa// Arvin Figueroa// ChriszelQueano// Marvin Villardo Jr.// Bowdy Sales
hiring the employee or the manner ofpaying the salary or the Labor Secretarys decision, ruling that Alegre was a
actual time spent at work. It is thecharacter of the activities permanent employee who could not be dismissed except for
performed in relation to theparticular trade or business taking just cause, and expiration of the employment contract was not
into account all thecircumstances, and in some cases the one of the just causes provided in the Labor Code for
length of time of itsperformance and its continued existence.It termination of services. Thus, Brent appealed before the SC.
is obvious thatone year after they were employed by
petitioner,respondents became regular employees by operation ISSUE:WON the termination of Alegres contract of
oflaw. employment with Brent School by reason of the expiration of
IN LIGHT OF ALL THE FOREGOING, the petition the agreed term of period is valid.
is DENIED for lack of merit. The assailed Decision and
Resolution of the Court of Appeals in CAG. R. SP No. 76582 HELD:YES.Accordingly, and since the entire purpose behind
are AFFIRMED. Costs against petitioner.SO ORDERED. the development of legislation culminating in the present
Article 280 of the Labor Code clearly appears to have been, as
already observed, to prevent circumvention of the employees
4. BRENT SCHOOL, INC., and REV. GABRIEL right to be secure in his tenure, the clause in said article
DIMACHE, petitioners, vs. RONALDO ZAMORA, the indiscriminately and completely ruling out all written or oral
Presidential Assistant for Legal Affairs, Office of the agreements conflicting with the concept of regular
President, and DOROTEO R. ALEGRE, respondents. employment as defined therein should be construed to refer to
the substantive evil that the Code itself has singled out:
G.R. No. 48494. February 5, 1990. agreements entered into precisely to circumvent security of
tenure. It should have no application to instances where a
FACTS:Doroteo R. Alegre was engaged as athletic director fixed period of employment was agreed upon knowingly and
by Brent School, Inc. at a yearly compensation of voluntarily by the parties, without any force, duress or
P20,000.00.The contract fixed a specificterm for its existence, improper pressure being brought to bear upon the employee
five (5) years.Some three months before the expiration of and absent any other circumstances vitiating his consent, or
thestipulated period, or more precisely on April 20, where it satisfactorily appears that the employer and
1976,Alegre was given a copy of the report filed by Brent employee dealt with each other on more or less equal terms
Schoolwith the Department of Labor advising of the with no moral dominance whatever being exercised by the
terminationof his services effective on July 16, 1976. The former over the latter. Unless thus limited in its purview, the
stated groundfor the termination was completion of contract, lawwould be made to apply to purposes other than
expirationof the definite period of employment. And a month thoseexplicitly stated by its framers it thus becomes
or solater, on May 26, 1976, Alegre accepted the amount pointlessand arbitrary, unjust in its effects and apt to lead to
ofP3,177.71, and signed a receipt there for containing absurdand unintended consequences.
thephrase, in full payment of services for the period May WHEREFORE, the public respondents Decision
16,to July 17, 1976 as full payment of contract. complained of is REVERSED and SET ASIDE. Respondent
However, at the investigation conducted by a Alegres contract of employment with Brent School having
LaborConciliator of said report of termination of his lawfully terminated with and by reason of the expiration of the
services,Alegre protested the announced termination of agreed term of period thereof, he is declared not entitled to
hisemployment. He argued that although his contract did reinstatement and the other relief awarded and confirmed on
stipulate that the same would terminate on July 17, 1976, since appeal in the proceedings below. No pronouncement as to
his services were necessary and desirable in the usualbusiness costs. SO ORDERED.
of his employer, and his employment had lasted for five years,
he had acquired the status of a regular
employee and could not be removed except for valid cause. 5. COLUMBUS PHILIPPINES BUS CORPORATION,
petitioner, vs. NATIONAL LABOR RELATIONS
REGIONAL DIRECTOR:The Regional Director considered COMMISSION, ZENAIDA DOMASIG and ROMAN
Brent Schools report as an application for clearance to DOMASIG, respondents.
terminate employment (not a report of termination), and
accepting the recommendation of the Labor Conciliator, G.R. Nos. 11485859. September 7, 2001.
refused to give such clearance and instead required the
reinstatement of Alegre, as a permanent employee, to his FACTS:Columbus Philippines Bus Corporation is engaged in
former position without loss of seniority rights and with full the business of operating passenger buses. Since the start of its
back wages. Brent School filed a motion for reconsideration. operations in 1990, it has maintained a list of drivers and
TheRegional Director denied the motion and forwarded conductors who rendered service in its bus units allegedly on a
thecase to the Secretary of Labor for review. first come first served basis and compensated purely on
commission. The drivers and conductors/conductress worked
SECRETARY OF LABOR:Sustained the Regional for about ten to fifteen days a month and were allegedly not
Director.Brent appealed to the Office of the President. required to work every day.
Roman Domasig started working as a driver with the
OFFICE OF THE PRESIDENT:Again it was rebuffed. That petitioner on August 30, 1990 with a daily income ranging
Office dismissed its appeal for lack of merit and affirmed the from P350.00 to P650.00, while his wife ZenaidaDomasig,

Page 4
Dan Gloria// Kat Ramirez// Jessica Bernardo// Joseph De Mesa// Arvin Figueroa// ChriszelQueano// Marvin Villardo Jr.// Bowdy Sales
was employed as a bus conductress on October 1, 1990 with a collecting agents of the company,nevertheless, per a certain
daily income of P250.00) to P500.00. The employment of written agreement they wereconsidered as independent
Roman and ZenaidaDomasig with the bus company was contractors and not employeesof the company.
abruptly terminated on January 21 and 22, 1992, respectively, WHEREFORE, the petition is hereby DISMISSED,
for their having allegedly formed a labor union. and thechallenged Resolution of public respondent NLRC
Thus, these two (2) related cases of unfair labor isAFFIRMED. The computation of the amount of back
practice, illegal dismissal, illegal deductions from salary, and wagesto which private respondents Roman Domasig and
nonpayment of service incentive leave pay and 13th month ZenaidaDomasig are entitled is hereby REMANDED to the
pay were instituted by private respondents against Columbus LaborArbiter for appropriate action.SO ORDERED.
Philippines Bus Corporation and its officers, Atty. Ferdinand
Catabian and Mrs. Amelia de Dios, before the DOLE.
6. SINGER SEWING MACHINE COMPANY, petitioner,
LABOR ARBITER:Labor Arbiter found for Ramon and vs. HON. FRANKLIN M. DRILON, MED-
ZenaidaDomasig and ordered the bus company to reinstate ARBITERFELIX B.CHAGUILE, JR., and SINGER
Roman and ZenaidaDomasig to their former positions as MACHINE COLLECTORSUNION
driver and bus conductress, respectively, without loss of BAGUIO(SIMACUB), respondents.
seniority, rights and with back pay accruing from January 21,
1992 and January 22, 1992 up to their actual reinstatement. G.R. No. 91307. January 24, 1991.
Aggrieved by the adverse judgment of the Labor Arbiter,
petitioner appealed to NLRC. FACTS:On February 15, 1989, the union filed a petition for
direct certification as the sole and exclusive bargaining agent
NLRC:NLRC affirmed the Labor Arbiters decision, and in of all collectors of the Singer Sewing Machine Company,
its OrderdatedJanuary 7, 1994 denied the petitioners motion Baguio City. The Company opposed the petition mainly on the
forreconsideration. Thus, the petitioner filed an appeal before ground that the union members are actually not employees but
the SC. are independent contractors as evidenced by the collection
agency agreement which they signed. The respondent Med-
ISSUE:WON Ramon and ZenaidaDomasig were regular Arbiter, finding that there exists an employer-employee
employees. relationship between the union members and the Company,
granted the petition for certification election. On appeal,
HELD:YES.To determine whether the employment of an Secretary of Labor Franklin M. Drilon affirmed it. The motion
employee is regular or casual, Article 280of the Labor Code is for reconsideration of the Secretarys resolution was denied.
definitive and whether such employment is regular or casual
has nothing to do with the manner of computing and paying ISSUE:WON article 280 is the yardstick for determining the
the employees wages or compensation. Rather the said existence of an employment relationship.
provision of the Labor Code provides that:
The primary standard, x xx of determining a regular HELD:NO.The Court finds the contention of the respondents
employment is the reasonable connection between the that the union members are employees under Article 280 of
particular activity performed by the employee in relation to the Labor Code to have no basis. The definition that regular
the usual business or trade of the employer. The test is employees are those who perform activities which are
whether the former is usually necessary or desirable in the desirable and necessary for the business of the employer is not
usual business or trade of the employer. The connection can determinative in this case. Any agreement may provide that
be determined by considering the nature of the work one party shall render services for and in behalf of another for
performed and its relation to the scheme of the particular a consideration (no matter how necessary for the latters
business or trade in its entirety. Also, if the employee has been business) even without being hired as an employee. This is
performing the job for at least one year, even if the precisely true in the case of an independent contractorship as
performance is not continuous or merely intermittent, the law well as in an agency agreement. The Court agrees with the
deems the repeated and continuing need for its performance petitioners argument that Article 280 is not the yardstick for
as sufficient evidence of the necessity if not indispensability of determining the existence of an employment relationship
that activity to the business. Hence, the employment is also because it merely distinguishes between two kinds of
considered regular, but only with respect to such activity and employees, i.e., regular employees and casual employees, for
while such activity exists. purposes of determining the right of an employee to certain
Considering the above quotedstandard for benefits, to join or form a union, or to security of tenure.
determining aregular employment, it appears that the Article 280 does not apply where the existence of an
employment ofprivate respondents is regular. They perform employment relationship is in dispute.
worknecessary and desirable in the business of the petitioner. WHEREFORE, the Order dated June 14, 1989 of
Without the services of the bus drivers and conductors, likethe Med-Arbiter Designate Felix B. Chaguile, Jr., the Resolution
private respondents, the petitioner could not haveoperated and and Order of Secretary Franklin M. Drilon dated November 2,
managed its business of providingtransportation services to the 1989 and December 14, 1989, respectively are hereby
public. However, not allemployees paid on commission basis REVERSED and SET ASIDE. The petition for certification
can legally beconsidered as regular employees. In the case of election is ordered dismissed and the temporary restraining
Singer Sewing Machine Company v. Drilon,it was held that
while certain individuals were hired to work ascollectors or

Page 5
Dan Gloria// Kat Ramirez// Jessica Bernardo// Joseph De Mesa// Arvin Figueroa// ChriszelQueano// Marvin Villardo Jr.// Bowdy Sales
order issued by the Court on December 21, 1989 is made Apropos to this is the rule that the term or nomenclature given
permanent. SO ORDERED. to a stipulated benefit is not controlling, but the intent of the
parties to the Agreement conferring such benefit. The fact
that complainant was made subject to respondents Rules and
7. 1 Sonza vs. ABSCBN Broadcasting Corporation Regulations, likewise, does not detract from the absence of
G.R. No. 138051. June 10, 2004. employer-employee relationship. As held by the Supreme
Court, The line should be drawn between rules that merely
FACTS: In May 1994, respondent ABSCBN Broadcasting serve as guidelines towards the achievement of the mutually
Corporation (ABSCBN) signed an Agreement desired result without dictating the means or methods to be
(Agreement) with the Mel and Jay Management and employed in attaining it, and those that control or fix the
Development Corporation (MJMDC). ABSCBN was methodology and bind or restrict the party hired to the use of
represented by its corporate officers while MJMDC was such means. The first, which aim only to promote the result,
represented by SONZA, as President and General Manager, create no employer-employee relationship unlike the second,
and Carmela Tiangco (TIANGCO), as EVP and Treasurer. which address both the result and the means to achieve it.
Referred to in the Agreement as AGENT, MJMDC agreed (Insular Life Assurance Co., Ltd. vs. NLRC, et al., G.R. No.
to provide SONZAs services exclusively to ABSCBN as 84484, November 15, 1989).
talent for radio and television. The Agreement listed the SONZA appealed to the NLRC. On 24 February
services SONZA would render to ABSCBN. ABSCBN agreed 1998, the NLRC rendered a Decision affirming the Labor
to pay for SONZAs services a monthly talent fee of P310,000 Arbiters decision. He filed a motion for reconsideration,
for the first year and P317,000 for the second and third year of which the NLRC denied in its Resolution dated 3 July 1998.
the Agreement. ABSCBN would pay the talent fees on the On 6 October 1998, SONZA filed a special civil action for
10th and 25th days of the month. certiorari before the Court of Appeals assailing the decision
On 30 April 1996, SONZA filed a complaint against and resolution of the NLRC. On 26 March 1999, the Court of
ABSCBN before the Department of Labor and Employment, Appeals rendered a Decision dismissing the case.Hence, this
National Capital Region in Quezon City. SONZA complained petition.
that ABSCBN did not pay his salaries, separation pay, service
incentive leave pay, 13th month pay, signing bonus, travel ISSUE: W/N Sonza is a regular employee.
allowance and amounts due under the Employees Stock
Option Plan (ESOP). On 10 July 1996, ABSCBN filed a HELD: No.We affirm the assailed decision. The present
Motion to Dismiss on the ground that no employer-employee controversy is one of first impression. Although Philippine
relationship existed between the parties. SONZA filed an labor laws and jurisprudence define clearly the elements of an
Opposition to the motion on 19 July 1996. employer-employee relationship, this is the first time that the
The Labor Arbiter rendered his Decision dated 8 July Court will resolve the nature of the relationship between a
1997 dismissing the complaint for lack of jurisdiction.The television and radio station and one of its talents. There is
pertinent parts of the decision read as follows: While no case law stating that a radio and television program host is
Philippine jurisprudence has not yet, with certainty, touched an employee of the broadcast station.
on the true nature of the contract of a talent, it stands to SONZA contends that the Labor Arbiter has jurisdiction over
reason that a talent as above described cannot be considered the case because he was an employee of ABSCBN. On the
as an employee by reason of the peculiar circumstances other hand, ABSCBN insists that the Labor Arbiter has no
surrounding the engagement of his services. It must be noted jurisdiction because SONZA was an independent contractor.
that complainant was engaged by respondent by reason of his SONZA maintains that all essential elements of an Employer-
peculiar skills and talent as a TV host and a radio employee relationship are present in this case. Since there is
broadcaster. Unlike an ordinary employee, he was free to no local precedent on whether a radio and television program
perform the services he undertook to render in accordance host is an employee or an independent contractor, we refer to
with his own style. The benefits conferred to complainant foreign case law in analyzing the present case. The United
under the May 1994 Agreement are certainly very much States Court of Appeals, First Circuit, recently held in
higher than those generally given to employees. For one, AlbertyVlez v. Corporacin De Puerto Rico Para La
complainant Sonzas monthly talent fees amount to a DifusinPblica(WIPR)that a television program host is an
staggering P317,000. Moreover, his engagement as a talent independent contractor.
was covered by a specific contract. Likewise, he was not Applying the control test to the present case, we find
bound to render eight (8) hours of work per day as he worked that SONZA is not an employee but an independent
only for such number of hours as may be necessary. The fact contractor. The control test is the most important test our
that per the May 1994 Agreement complainant was accorded courts apply in distinguishing an employee from an
some benefits normally given to an employee is independent contractor.This test is based on the extent of
inconsequential. Whatever benefits complainant enjoyed arose control the hirer exercises over a worker. The greater the
from specific agreement by the parties and not by reason of supervision and control the hirer exercises, the more likely the
employer-employee relationship. As correctly put by the worker is deemed an employee.
respondent, All these benefits are merely talent fees and other
contractual benefits and should not be deemed as salaries, 7. 2 THELMA DUMPIT-MURILLO, petitioner, vs.
wages and/or other remuneration accorded to an employee, COURT OF APPEALS, ASSOCIATED
notwithstanding the nomenclature appended to these benefits. BROADCASTING COMPANY, JOSE JAVIER AND
EDWARD TAN, respondents.
Page 6
Dan Gloria// Kat Ramirez// Jessica Bernardo// Joseph De Mesa// Arvin Figueroa// ChriszelQueano// Marvin Villardo Jr.// Bowdy Sales
G.R. No. 164652. June 8, 2007 HELD: FIRST. NO. The Court of Appeals committed
reversible error when it held that petitioner was a fixed term
FACTS:On October 2, 1995, under a Talent Contract, employee. Petitioner was a regular employee under
Associated Broadcasting Company (ABC) hired petitioner contemplation of law. The practice of having fixed term
Thelma Dumpit-Murillo as a newscaster and co-anchor for contracts in the industry does not automatically make all talent
BalitangBalita, an early evening news program. The contract contracts valid and compliant with labor law. The assertion
was for a period of three months and was renewed. In that a talent contract exists does not necessarily prevent a
addition, petitioners services were engaged for the program regular employment status. Further, the Sonzacase is not
Live onFive. On September 30, 1999, after four years of applicable. In Sonza, thetelevision station did not instruct
repeated renewals, petitioners talent contract expired. Two Sonza how to perform hisjob. How Sonza delivered his lines,
weeks after the expiration of the last contract, petitioner sent a appeared on television,and sounded on radio were outside the
letter to Mr. Jose Javier, Vice President for News and Public television stationscontrol. Sonza had a free hand on what to
Affairs of ABC, informing the latter that she was still say or discuss inhis shows provided he did not attack the
interested in renewing her contract subject to a salary increase. television station orits interests. Clearly, the television station
Thereafter, petitioner stopped reporting for work. did not exercisecontrol over the means and methods of the
A month later, petitioner sent a demand letterto ABC, performance ofSonzas work.In the case at bar, ABC had
demanding: (a) reinstatement to her former position (b) control over theperformance of petitioners work. Noteworthy
payment of unpaid wages for services rendered from too, is thecomparatively low P28,000 monthly pay of
September 1 to October 20, 1999 and full backwages petitionervisthe P300,000 a month salary of Sonza,that all the
(c)payment of 13th month pay, vacation/sick/service morebolsters the conclusion that petitioner was not in the
incentiveleaves and other monetary benefits due to a samesituation as Sonza.
regularemployee starting March 31, 1996. ABC replied that a SECOND. YES. In Manila Water Company, Inc. v.
checkcovering petitioners talent fees for September 16 to Pena,we said that the elements to determine the existence of
October20, 1999 had been processed and prepared, but that an employment relationship are: (a) the selection and
theother claims of petitioner had no basis in fact or in law. engagement of the employee, (b) the payment of wages, (c)
On December 20, 1999, petitioner filed a the power of dismissal, and (d) the employers power to
complaintagainst ABC, Mr. Javier and Mr. Edward Tan, for control. The most important element is the employers control
illegal constructive dismissal, nonpayment of salaries, of the employees conduct, not only as to the result of the
overtime pay, premium pay, separation pay, holiday pay, work to be done, but also as to the means and methods to
service incentive leave pay, vacation/sick leaves and 13th accomplish it. The duties of petitioner as enumerated in her
month pay. She likewise demanded payment for moral, employment contract indicate that ABC had control over the
exemplary and actual damages, as well as for attorneys fees. work of petitioner. Aside from control, ABC also dictated the
work assignments and payment of petitioners wages. ABC
LABOR ARBITER:Labor Arbiter dismissed the complaint. also had power to dismiss her. All these being present, clearly,
there existed an employment relationship between petitioner
NLRC:On appeal, the NLRC reversed the Labor Arbiter in a and ABC. Concerning regular employment, the law provides
Resolution dated August 30, 2000. The NLRC held that an for two
employer-employee relationship existed between petitioner kinds of employees, namely: (1) those who are engaged to
and ABC that the subject talent contract was void that the perform activities which are usually necessary or desirable in
petitioner was a regular employee illegally dismissed and that the usual business or trade of the employer and (2) those who
she was entitled to reinstatement and backwages or separation have rendered at least one year of service, whether continuous
pay, aside from 13th month pay and service incentive leave or broken, with respect to the activity in which they are
pay, moral and exemplary damages and attorneys fees. employed.In other words, regular status arises from either the
nature of work of the employee or the duration of his
CA:The appellate court ruled that the NLRC committed grave employment. In Benares v. Pancho,we very succinctly said:.
abuse of discretion, and reversed the decision of the . . [T]he primary standard for determining regular
NLRC.The appellate court reasoned that petitioner should not employment is the reasonable connection between the
be allowed to renege from the stipulations she had voluntarily particular activity performed by the employee vis--vis the
and knowingly executed by invoking the security of tenure usual trade or business of the employer. This connection can
under the Labor Code. According to the appellate court, be determined by considering the nature of the work
petitioner was a fixed term employee and not a regular performed and its relation to the scheme of theparticular
employee within the ambit of Article 280of the Labor Code business or trade in its entirety. If the employee has
because her job, as anticipated and agreed upon, was onlyfor a beenperforming the job for at least a year, even if the
specified time. performance is not continuous and merely intermittent, the
law deems repeated andcontinuing need for its performance
ISSUE: 1. WON the prevailing jurisprudence citing Sonza vs. as sufficient evidence of thenecessity if not indispensability of
ABS-CBN, that talent contracts do not create employer- that activity to the business. Hence, the employment is
employee relationship, would apply in the case at bar. considered regular, but only with respectto such activity and
while such activity exists.In our view, the requisites for
2. WON Dumpit-Murillo is a regular employee regularity of employment have been met in the instant case.
contemplated in art. 280 of the Labor Code. Gleaned from the description of the scope of services

Page 7
Dan Gloria// Kat Ramirez// Jessica Bernardo// Joseph De Mesa// Arvin Figueroa// ChriszelQueano// Marvin Villardo Jr.// Bowdy Sales
aforementioned, petitioners work was necessary or desirable vital, necessary and indispensable to the usual business of
in the usual business or trade of the employer which includes, petitioner, nonetheless the second paragraph of Article 280 of
as a precondition for its enfranchisement, its participation in the Labor Code still applies. It reads:
the governments news and public information dissemination. ART. 280. REGULAR AND CASUAL
In addition, her work was continuous for a period of four EMPLOYMENT. An employment shall be deemed to be casual
years. This repeated engagement under contract of hire is if it is not covered by the preceding paragraph.Provided,
indicative of the necessity and desirability of the petitioners That, any employee who has rendered at least one year of
work in private respondent ABCs business. service whether such service is continuous or broken, shall
WHEREFORE, the challenged Decision dated be considered a regular employee with respect to the activity
January 30, 2004 and Resolution dated June 23, 2004 of the in which he is employed and his employment shall continue
Court of Appeals in CA G.R. SP No. 63125, which held that while such activity exists. (Emphasis supplied).
the petitioner was a fixed term employee, are REVERSED and We thus rule and so hold that respondents are
SET ASIDE. The NLRC decision is AFFIRMED. Costs petitioner's regular employees, at least with respect to the
against private respondents.SO ORDERED. production of petitioner's Visayan tele-series programs and
until such activity exists.
WHEREFORE, the instant petition is hereby
8. ABS-CBN Broadcasting Corp. vs. Marquez DENIED.

G.R. NO. 167638 June 12, 2005


9. Consolidated Broadcasting System, Inc. vs. Oberio
FACTS: Petitioner hired the services of respondents on G.R. No. 168424. June 8, 2007.
various dates starting December, 1994 to undertake the
production in the Cebuano dialect of television serial The test to determine whether employment is regular or not is
programs for petitioner's week-day afternoon time slots in the reasonable connection between the particular activity
Cebu. Respondents were assigned among three (3) production performed by the employee in relation to the usual business or
groups, each with its own set of directors, writers, trade of the employeralso, if the employee has been
videographers, lightsmen, editors, actors and utility personnel. performing the job for at least one year, even if the
Each production group was given a weekly budget, initially performance is not continuous or merely intermittent, the law
at P30,000.00, which was later increased to P40,000.00 a deems the repeated and continuing need for its performance
week. as sufficient evidence of the necessity, if not indispensability of
The television-series did so well that several more that activity to the business.
were subsequently produced. The production groups were
continuously engaged to film succeeding programs to replace FACTS: Sometime in August 1998, petitioner reduced the
the concluded ones. number of its drama productions from 14 to 11, but was
On June 15, 1999, respondents addressed a letter to opposed by respondents. After the negotiations failed, the
petitioner asking for a 25% increase in their weekly budget, latter sought the intervention of the Department of Labor and
but the same was denied by petitioner's AVP for the Visayas Employment (DOLE), which on November 12, 1998,
Cluster, Ma. Luisa L. Ascalon. Instead, respondents were conducted through its Regional Office, an inspection of
informed of the termination of their services effective August DWYB station. The results thereof revealed that petitioner is
13, 1999. On August 27, 1999, respondents filed with the guilty of violation of labor standard laws, such as
Regional Arbitration Branch (RAB) at Region VII of the underpayment of wages, 13th month pay, non-payment of
Department of Labor and Employment their consolidated service incentive leave pay, and noncoverage of respondents
complaint for illegal dismissal. under the Social Security System.
Respondents alleged that they were employed as
LABOR ARBITER: On June 15, 2000, the Executive Labor drama talents by DYWBBombo
Arbiter rendered a decision in favor of respondents and Radyo, a radio station owned and operated by petitioner
ordered petitioner to pay to them their money claims. Consolidated Broadcasting System, Inc. They reported for
work daily for six days in a week and were required to record
NLRC: Reveresed the decision of the Labor Arbiter. their drama production in advance. Some of them were
employed by petitioner since 1974, while the latest one was
CA: Reversed the decision of the NLRC and reinstating the hired in 1997.Their drama programs were aired not only in
decision of the Labor Arbiter that there was an employer- Bacolod City but also in the sister stations of DYWB in the
employee relationship. Visayas and Mindanao
areas.
ISSUE: WON the respondent is a regular employee. Petitioner contended that respondents are not its
employees and refused to submit the payroll and daily time
HELD: YES.It is a matter of record that respondents have records despite the subpoena ducestecumissued by the DOLE
rendered almost five (5) years of continuous service to Regional Director. Petitioner further argued that the case
petitioner, doing work that is necessary and desirable to the should be referred to the NLRC because the Regional Director
usual business of the latter. Hence, even granting on the has no jurisdiction over the determination of the existence of
extreme that respondents were not performing work that is employer- employee relationship which involves evidentiary

Page 8
Dan Gloria// Kat Ramirez// Jessica Bernardo// Joseph De Mesa// Arvin Figueroa// ChriszelQueano// Marvin Villardo Jr.// Bowdy Sales
matters that are not verifiable in the normal course of employment is regular or not is the reasonable connection
inspection. between the particular activity performed by the employee in
Vexed by the respondents complaint, petitioner relation to the usual business or trade of the employer. Also, if
allegedly pressured and intimidated respondents. Respondents the employee has been performing the job for at least one
Oberio and Delta were suspended for minor lapses and the year, even if the performance is not continuous or merely
payment of their salaries were purportedly delayed. intermittent, the law deems the repeated and continuing need
Eventually, on February 3, 1999, pending the outcome of the for its performance as sufficient evidence of the necessity, if
inspection case with the Regional Director, respondents were not indispensability of that activity to the business. Thus, even
barred by petitioner from reporting for work thus, the former assuming that respondents were initially hired as
claimed constructive dismissal. project/contractual employees who were paid per drama or per
Respondents filed a case for illegal dismissal, project/contract, the engagement of their services for 2 to 25
underpayment/non-payment of wages and benefits plus years justify their classification as regular employees, their
damages against petitioner. Labor Arbiter dismissed the case services being deemed indispensable to the business of
without prejudice while waiting for the decision of the petitioner.
Secretary of Labor on the same issue of the existence of an
employer-employee relationship between petitioner and 10. Orozco vs. Fifth Division of the Court of Appeals
respondents. On appeal to the NLRC, respondents raised the G.R. No. 155207. August 13, 2008.
issue of employer-employee relationship and submitted
evidence. Petitioner, on the other hand, did not present any The inevitable conclusion is that petitioner columnist was not
documentary evidence in its behalf and merely denied the an employee of respondent newspaper but an independent
allegations of respondents. It claimed that the radio station contractor, engaged to do independent work.
pays for the drama recorded by piece and that it has no control
over the conduct of respondents. NLRC rendered a decision FACTS: In March 1990, PDI engaged the services of
holding that respondents were regular employees of petitioner petitioner Wilhelmina S. Orozco to write a weekly column for
who were illegally dismissed by the latter. Hence, petitioner its Lifestyle section. She religiously submitted her articles
filed the instant recourse. every week, except for a Six-month stint in New York City
when she, nonetheless, sent several articles through mail. She
ISSUE:W/N the respondents are regular employees of the received compensation of P250.00later increased to
petitioner. P300.00for every column published. Petitioner claims that
her then editor, Ms.Lita T. Logarta, told her that respondent
HELD:Yes.substantial justice dictates that this case be Leticia Jimenez Magsanoc, PDI Editor in Chief, wanted to
resolved on the merits considering that the NLRC and the stop publishing her column for no reason at all and advised
Court of Appeals correctly found that there existed an petitioner to talk to Magsanoc herself. Petitioner narrates that
employer-employee relationship between petitioner and when she talked to Magsanoc, the latter informed her that it
respondents and that the latters dismissal was illegal. was PDI Chairperson Eugenia Apostol who had asked to stop
Respondents employment with petitioner passed the fourfold publication of her column, but that in a telephone conversation
test on employer-employee relations, namely: (1) the with Apostol, the latter said that Magsanoc informed her
selection and engagement of the employee, or the power to (Apostol) that the Lifestyle section already had many
hire (2) the payment of wages (3) the power to dismiss and columnists.
(4) the power to control the employee. Petitioner failed to PDI claims that in June 1991, Magsanoc met with the
controvert with substantial evidence the allegation of Lifestyle section editor to discuss how to improve said section.
respondents that they were hired by the former on various They agreed to cut down the number of columnists by keeping
dates from 1974 to 1997. If petitioner did not hire respondents only those whose columns were well written, with regular
and if it was the director alone who chose the talents, feedback and following. In their judgment, petitioners column
petitioner could have easily shown, being in possession of the failed to improve, continued to be superficially and poorly
records, a contract to such effect. However, petitioner merely written, and failed to meet the high standards of the
relied on its contention that respondents were piece rate newspaper. Hence, they decided to terminate petitioners
contractors who were paid by results.Note that under Policy column. Aggrieved by the newspapers action, petitioner filed
Instruction No. 40, petitioner is obliged to execute the a complaint for illegal dismissal, backwages, moral and
necessary contract specifying the nature of the work to be exemplary damages, and other money claims before the
performed, rates of pay, and the programs in which they will NLRC.
work. Moreover, project or contractual employees are required Labor Arbiter Arthur Amansec rendered a Decision
to be apprised of the project they will undertake under a in favor of petitioner. PDI appealed the Decision to the NLRC.
written contract. This was not complied with by the petitioner, In a Decision dated August 23, 1994, the NLRC Second
justifying the reasonable conclusion that no such contracts Division dismissed the appeal thereby affirming the Labor
exist and that respondents were in fact regular employees. Arbiters Decision. PDI then filed a Petition for Review before
Moreover, the engagement of respondents for a period ranging this Court seeking the reversal of the NLRC Decision.
from 2 to 25 years and the fact that their drama programs were However, in a Resolution dated December 2, 1998, this Court
aired not only in Bacolod City but also in the sister stations of referred the case to the Court of Appeals, pursuant to our
DYWB in the Visayas and Mindanao areas, undoubtedly show ruling in St. Martin Funeral Home v. National Labor
that their work is necessary and indispensable to the usual Relations Commission. CA set aside the NLRC Decision and
business or trade of petitioner. The test to determine whether dismissed petitioners Complaint. It held that the NLRC
Page 9
Dan Gloria// Kat Ramirez// Jessica Bernardo// Joseph De Mesa// Arvin Figueroa// ChriszelQueano// Marvin Villardo Jr.// Bowdy Sales
misappreciated the facts and rendered a ruling wanting in he claims that he is entitled to reinstatement with full
substantial evidence backwages. He also claims that he is entitled to moral and
exemplary damages. He includes payment of his overtime pay,
ISSUE: W/N a newspaper columnist is an employee of the project allowance, minimum wage increase adjustment,
newspaper which publishes the column. proportionate 13th month pay and attorneys fees.
Labor Arbiter Cresencio R. Iniego rendered a
HELD:No.It is true that petitioner herself admitted that she decision in favor of the respondent. Petitioner appealed to the
was not, and [had] never been considered respondents National Labor Relations Commission which rendered the
employee because the terms of works were arbitrarily decided questioned Resolution dated March 24, 1992 dismissing the
upon by the respondent. However, the employment status of appeal. The motion for reconsideration filed by petitioner was
a person is defined and prescribed by law and not by what the denied by the NLRC in its Order dated July 31, 1992.
parties say it should be. This Court has constantly adhered to
the fourfold test to determine whether there exists an ISSUE:W/N the respondent NLRC committed grave abuse of
employer-employee relationship between parties. Of these discretion amounting to lack or excess of jurisdiction when it
four elements, it is the power of control which is the most ruled that private respondent was a regular employee and not a
crucial and most determinative factor. Petitioner has project employee.
misconstrued the control test, as did the Labor Arbiter and
the NLRC. Not all rules imposed by the hiring party on the HELD:No. Private respondents employment status was
hired party indicate that the latter is an employee of the established by the Certification of Employment dated April
former. Rules which serve as general guidelines towards the 10, 1989 issued by petitioner which certified that private
achievement of the mutually desired result are not indicative respondent is a bonafide employee of the petitioner from June
of the power of control. The control that PDI exercised over 30, 1976 up to the time the certification was issued on April
petitioner was only as to the finished product of her efforts, 10, 1989. The same certificate of employment showed that
i.e., the column itself, by way of either shortening or outright private respondents exposure to their field of operation was as
rejection of the column. The newspapers power to approve or fabricator, helper/electrician, stockman/timekeeper. This
reject publication of any specific article she wrote for her proves that private respondent was regularly and continuously
column cannot be the control contemplated in the control employed by petitioner in various job assignments from 1976
test, as it is but logical that one who commissions another to to 1989, for a total of 13 years. The alleged gap in
do a piece of work should have the right to accept or reject the employment service cited by petitioner does not defeat private
product. The important factor to consider in the control test respondents regular status as he was rehired for many more
is still the element of control over how the work itself is done, projects without interruption and performed functions which
not just the end result thereof. Although petitioner had a are vital, necessary and indispensable to the usual business of
weekly deadline to meet, she was not precluded from petitioner.We have held that where the employment of project
submitting her column ahead of time or from submitting employees is extended long after the supposed project has
columns to be published at a later time. More importantly, been finished, the employees are removed from the scope of
respondents did not dictate upon petitioner the subject matter project employees and considered regular employees.Private
of her columns, but only imposed the general guideline that respondent had presented substantial evidence to support his
the article should conform to the standards of the newspaper position, while petitioner merely presented an unverified
and the general tone of the particular section. position paper merely stating therein that private respondent
has no cause to complain since the employment contract
11. Audion Electric Co., Inc. vs. NLRC signed by private respondent with petitioner was coterminous
G.R. No. 106648. June 17, 1999. with the project. Notably, petitioner failed to present such
employment contract for a specific project signed by private
Where the employment of project employees is extended long respondent that would show that his employment with the
after the supposed project has been finished, the employees petitioner was for the duration of a particular project.
are removed from the scope of project employees and Petitioner should have submitted or filed as many reports of
considered regular employees. termination as there were construction projects actually
finished, considering that private respondent had been hired
FACTS: The complainant was employed by Audion Electric since 1976. The failure of petitioner to submit reports of
Company on June 30, 1976 as fabricator and continuously termination supports the claim of private respondent that he
rendered service assigned in different offices or projects as was indeed a regular employee. This court has consistently
helper electrician, stockman and timekeeper. He has rendered held that failure of the employer to file termination reports
thirteen (13) years of continuous, loyal and dedicated service after every project completion with the nearest public
with a clean record. On August 3, complainant was surprised employment office is an indication that private respondent was
to receive a letter informing him that he will be considered not and is not a project employee.
terminated after the turnover of materials, including
respondents tools and equipments not later than August 15, 12. Universal Robina Corporation vs. Catapang
1989. Complainant claims that he was dismissed without G.R. No. 164736. October 14, 2005.
justifiable cause and due process and that his dismissal was
done in bad faith which renders the dismissal illegal. On its The five (5)month contract of employment was used by
part, Audion Electric Co. merely relied on its unverified letter petitioners as a convenient subterfuge to prevent private
communication signed by its project manager. For this reason, respondents from becoming regular employees. Petitioners
Page 10
Dan Gloria// Kat Ramirez// Jessica Bernardo// Joseph De Mesa// Arvin Figueroa// ChriszelQueano// Marvin Villardo Jr.// Bowdy Sales
act of repeatedly hiring private respondents negates their
contention that private respondents were hired for a specific HELD:Yes. In any case, we find that the CA, the NLRC and
project or undertaking only. the Labor Arbiter correctly categorized the respondents as
regular employees of the petitioner company. Thus, we quote
FACTS: The individual respondents were hired by the with approval the following excerpt from the decision of the
petitioner company on various dates from 1991 to 1993 to CA:
work at its duck farm in Barangay Sto. Tomas, Calauan, It is obvious that the said five-month contract of employment
Laguna. The respondents were hired under an employment was used by petitioners as a convenient subterfuge to prevent
contract which provided for a five-month period. After the private respondents from becoming regular employees. Such
expiration of the said employment contracts, the petitioner contractual arrangement should be struck down or disregarded
company would renew them and reemploy the respondents. as contrary to public policy or morals. To uphold the same
This practice continued until sometime in 1996, when the would, in effect, permit petitioners to avoid hiring permanent
petitioners informed the respondents that they were no longer or regular employees by simply hiring them on a temporary or
renewing their employment contracts. In October 1996, the casual basis, thereby violating the employees security of
respondents filed separate complaints for illegal dismissal, tenure in their jobs.
reinstatement, backwages, damages and attorneys fees against ...
the petitioners. The complaints were later consolidated. Petitioners act of repeatedly and continuously hiring private
The Labor Arbiter rendered a decision in favor of the respondents in a span of . . . 3 to 5 years to do the same kind
respondents. On May 17, 1999, the petitioners filed an Appeal of work negates their contention that private respondents were
Memorandum with the National Labor Relations Commission hired for a specific project or undertaking only.
(NLRC) on the ground that the Labor Arbiter erred in ruling
that the respondents are the petitioner companys regular 13. Abesco Construction and Development Corporation vs.
employees. Meanwhile, on May 18, 1999, the respondents Ramirez
filed a Motion for Enforcement of Reinstatement Order with G.R. No. 141168. April 10, 2006.
the Labor Arbiter. On June 21, 1999, the Labor Arbiter issued
a Writ of Execution enforcing the immediate reinstatement of Employees who work under different project employment
the respondents as mandated in the March 30, 1999 Decision. contracts for several years do not automatically become
On July 13, 1999, the petitioners manifested to the Labor regular employees Employees who are members of a work
Arbiter that they can reinstate only 17 of the 30 employees in pool from which a company draws workers for deployment to
view of the phase out of the petitioner companys Agricultural its different projects do not become regular employees by
Section as early as 1996. They averred that there were no reason of that fact alone. The principal test in determining
other available positions substantially similar to the positions whether employees are project employees or regular
previously occupied by the other 13 respondents, but that 10 employees is whether they are assigned to carry out a
of them could be accommodated at the farms Duck Dressing specific project or undertaking, the duration and scope of
Section which operates at an average of three days a week which are specified at the time they are engaged for that
only.On December 16, 1999, 17 employees were reinstated to project.
their former positions. Thereafter, the respondents moved for
the immediate reinstatement of the remaining 13 respondents. FACTS: Petitioner company was engaged in a construction
In case of failure to comply with the reinstatement of the 13 business where respondents were hired on different dates from
respondents, the LaborArbiter directed the petitioner company 1976 to 1992 either as laborers, road roller operators, painters
to pay them separation pay instead.Aggrieved, the petitioners or drivers. In 1997, respondents filed two separate
filed a petition for certiorari with the Court of Appeals (CA). complaintsfor illegal dismissal against the company and its
On August 21, 2003, the CA denied the petition for lack of General Manager, Oscar Banzon, before the Labor Arbiter
merit. (LA). Petitioners allegedly dismissed them without a valid
The petitioners submit that the respondents are not reason and without due process of law. The complaints also
regular employees. They aver that it is of no moment that the included claims for non-payment of the 13th month pay, five
respondents have rendered service for more than a year since days service incentive leave pay, premium pay for holidays
they were covered by the five-month individual contracts to and rest days, and moral and exemplary damages. The LA
which they duly acquiesced. The petitioners contend that they later on ordered the consolidation of the two complaints.
were free to terminate the services of the respondents at the Petitioners denied liability to respondents and
expiration of their individual contracts. The petitioners countered that respondents were project employees since
maintain that, in doing so, they merely implemented the terms their services were necessary only when the company had
of the contracts. projects to be completed. Petitioners argued that, being project
The respondents aver that they acquired the status as employees, respondents employment was coterminous with
regular employees after rendering one year of service to the the project to which they were assigned. They were not regular
petitioner company. They contend that the contracts providing employees who enjoyed security of tenure and entitlement to
for a fixed period of employment should be struck down as separation pay upon termination from work.
contrary to public policy, morals, good customs or public After trial, the LA declared respondents as regular
order as it was designed to preclude the acquisition of tenurial employees because they belonged to a work pool from
security. which the company drew workers for assignment to different
projects, at its discretion. He ruled that respondents were hired
ISSUE:W/N the respondents are regular employees. and rehired over a period of 18 years, hence, they were
Page 11
Dan Gloria// Kat Ramirez// Jessica Bernardo// Joseph De Mesa// Arvin Figueroa// ChriszelQueano// Marvin Villardo Jr.// Bowdy Sales
deemed to be regular employees. He likewise found that their It is an elementary rule in the law on labor relations that a
employment was terminated without just cause. Petitioners probationary employee who is engaged to work beyond the
appealed to the National Labor Relations Commission probationary period of six months, as provided under Art. 281
(NLRC) which affirmed the LAs decision.Subsequently, of the Labor Code, as amended, or for any length of time set
petitioners filed a petition for review in the Court of Appeals forth by the employer, shall be considered a regular employee.
(CA) but dismissed petitionersappeal.
FACTS: Sometime in September 1982, private respondent
ISSUE:Whether the respondents were project employees or Victoria Abril was employed by petitioner Philippine
regular employees. Federation of Credit Cooperatives, Inc. (PFCCI), a corporation
engaged in organizing services to credit and cooperative
HELD:We rule that respondents were regular employees. entities, as Junior Auditor/Field Examiner and thereafter held
However, we take exception to the reasons cited by the LA positions in different capacities, to wit: as office secretary in
(which both the NLRC and the CA affirmed) in considering 1985 and as cashier designate for four (4) months ending in
respondents as regular employees and not as project April 1988. Respondent, shortly after resuming her position as
employees. Contrary to the disquisitions of the LA, employees office secretary, subsequently went on leave until she gave
(like respondents) who work under different project birth to a baby girl. Upon her return sometime in November
employment contracts for several years do not automatically 1989, however, she discovered that a certain Vangie Santos
become regular employees they can remain as project had been permanently appointed to her former position. She,
employees regardless of the number of years they nevertheless, accepted the position of Regional Field Officer
work.Length of service is not a controlling factor in as evidenced by a contract which stipulated, among other
determining the nature of ones employment. Moreover, things, that respondents employment status shall be
employees who are members of a work pool from which a probationary for a period of six (6) months. Said period having
company (like petitioner corporation) draws workers for elapsed, respondent was allowed to work until PFCCI
deployment to its different projects do not become regular presented to her another employment contract for a period of
employees by reason of that fact alone. The Court has one year commencing on January 2, 1991 until December 31,
enunciated in some casesthat members of a work pool can 1991, after which period, her employment was terminated.
either be project employees or regular employees. In a complaint for illegal dismissal filed by
The principal test for determining whether employees respondent against PFCCI on April 1, 1992, Labor Arbiter
are project employees or regular employees is whether Cornelio L. Linsangan rendered a decision on March 10, 1993
they are assigned to carry out a specific project or dismissing the same for lack of merit but ordered PFCCI to
undertaking, the duration and scope of which are specified at reimburse her the amount of P2,500.00 which had been
the time they are engaged for that project.Such duration, as deducted from her salary. On appeal, however, the said
well as the particular work/service to be performed, is defined decision was reversed by the National Labor Relations
in an employment agreement and is made clear to the Commission (NLRC).
employees at the time of hiring.
In this case, petitioners did not have that kind of ISSUE:Whether the respondent is a probationary employee or
agreement with respondents. Neither did they inform a regular employee.
respondents of the nature of the latters work at the time of
hiring. Hence, for failure of petitioners to substantiate their HELD:Article 281 of the Labor Code, as amended, allows the
claim that respondents were project employees, we are employer to secure the services of an employee on a
constrained to declare them as regular probationary basis which allows him to terminate the latter for
employees.Furthermore, petitioners cannot belatedly argue just cause or upon failure to qualify in accordance with
that respondents continue to be their employees (so as to reasonable standards set forth by the employer at the time of
escape liability for illegal dismissal). Before the LA, his engagement. Probationary employees, notwithstanding
petitioners staunchly postured that respondents were only their limited tenure, are also entitled to security of tenure.
project employees whose employment tenure was Thus, except for just cause as provided by law,or under the
coterminous with the projects they were assigned to. However, employment contract, a probationary employee cannot be
before the CA, they took a different stance by insisting that terminated. The Court is constrained to review the contract of
respondents continued to be their employees. Petitioners employment entered into between the party litigants. While
inconsistent and conflicting positions on their true relation the initial statements of the contract show that respondents
with respondents make it all the more evident that the latter employment was for a fixed period, the succeeding provisions
were indeed their regular employees. thereof contradicted the same when it provided that
respondent shall be under probationary status commencing on
14. Phil. Federation of Credit Cooperatives, Inc. vs. NLRC February 17, 1990 and ending six (6) months thereafter.
G.R. No. 121071. December 11, 1998. Petitioner manifested that respondents employment for a
period of one year, from January until December 1991, having
Regardless of the designation an employer may have been fixed for a specified period, could not have converted her
conferred upon a workers employment status, where the latter employment status to one of regular employment. Conversely,
has completed the probationary period and allowed to work it likewise insisted that respondent was employed to perform
thereafter, he becomes a regular employee, and a dismissal work related to a project funded by the World Council of
premised on an alleged expiration of the contract is illegal. Credit Unions (WOCCU) and hence, her status is that of a
project employee. The Court is, thus, confronted with a

Page 12
Dan Gloria// Kat Ramirez// Jessica Bernardo// Joseph De Mesa// Arvin Figueroa// ChriszelQueano// Marvin Villardo Jr.// Bowdy Sales
situation under which the terms of the contract are so she damaged said parts Third, she frequently completes her
ambiguous as to preclude a precise application of the pertinent assigned tasks in twenty (20) to even twentyfive (25) seconds,
labor laws. over and above the required time limit, which is only eight (8)
Amidst the muddled assertions by petitioner, we to ten (10) seconds. Resultantly, the chickens/parts which
adhere to the pronouncement stated in the recent case of passed through her hands frequently suffer from premature
Villanueva v. NLRC,where the Court ruled that where a decomposition/bacterial or salmonella contamination By
contract of employment, being a contract of adhesion, is reason of the foregoing, Cals management deemed it best to
ambiguous, any ambiguity therein should be construed strictly terminate her probationary employment.
against the party who prepared it. Furthermore, Article 1702 However, the Court of Appeals set aside the NLRC
of the Civil Code provides that, in case of doubt, all labor ruling on the ground that at the time Candelarias services
contracts shall be construed in favor of the laborer. were terminated, she had attained the status of a regular
After a careful scrutiny of the subject contract, we employee as the termination on November 15, 1995 was
arrive at the conclusion that there was no grave abuse of effected four (4) days after the 6month probationary period
discretion on the part of the NLRC and, thus, affirm the had expired, hence, she is entitled to security of tenure in
finding that respondent has become a regular employee accordance with Article 281 of the Labor Code. CALS argues
entitled to security of tenure guaranteed under the Constitution that the Court of Appeals computation of the 6month
and labor laws. Regardless of the designation petitioner may probationary period is erroneous as the termination of
have conferred upon respondents employment status, it is, Candelarias services on November 15, 1995 was exactly on
however, uncontroverted that the latter, having completed the the last day of the 6monthperiod.
probationary period and allowed to work thereafter, became a
regular employee who may be dismissed only for just or ISSUE: WON Candelaria had attained the status of a regular
authorized causes under Articles 282, 283 and 284 of the employee.
Labor Code, as amended. Therefore, the dismissal, premised
on the alleged expiration of the contract, is illegal and entitles HELD:No. We agree with CALS contention as upheld by
respondent to the reliefs prayed for. both the Labor Arbiter and the NLRC that Candelarias
services was terminated within and not beyond the 6month
15. CALS Poultry Supply Corporation vs. Roco probationary period.
G.R. No.150660. July 30, 2002
In Cebu Royal v. Deputy Minister of Labor,our
FACTS:CALS Poultry Supply Corporation is engaged in computation of the 6months probationary period is reckoned
thebusiness of selling dressed chicken and other from the date of appointment up to the same calendar date of
relatedproduct and managed by Danilo Yap.On March 15, the 6th month following. Thus, we held: The original findings
1984, CALS hired Alfredo Roco as itsdriver. On the same were contained in aonepageorder reciting simply that
date, CALS hired Edna Roco, Alfredossister, as a helper in complainant was employed on a probationary period of
the dressing room of CALS.On May16, 1995, it hired employment for six (6) months. After said period, he
CandelariaRoco, another sister, ashelper,also at its chicken underwent medical examination for qualification as regular
dressing plant on a probationarybasis. On March 5, 1996, employee but the results showed that he is suffering from PTB
Alfredo Roco and CandelariaRoco filed a complaint for illegal minimal. Consequently, he was informed of the termination of
dismissal against CALS and Danilo Yap alleging that Alfredo his employment by respondent. The order then concluded that
and Candelaria were illegally dismissedon January 20, 1996 the termination was justified. The six-month period of
and November 5, 1996, respectively.Both also claimed that probation started from the said date of appointment and so
they were underpaid of their wages. ended on August 17, 1978, but it is not shown that the private
In the case of CandelariaRoco, the Labor Arbiter respondents employment also ended then on the contrary, he
upheld CALS decision not to continue with her probationary continued working as usual. Under Article 282 of the Labor
employment having been found her unsuited for the work for Code, an employee who is allowed to work after a
which her services were engaged. She was hired on May 16, probationary period shall be considered a regular employee.
1995 and her services were terminated on November 15, 1995. Hence, Pilones was already on permanent status when he was
There is no dispute that she was employed on dismissed on August 21, 1978, or four days after he ceased to
probationary basis. She was hired on May 16, 1995 and her be a probationer.
services were terminated on November 15, 1995 due to poor
work performance. She did not measure up to the work 16. Mitsubishi Motors Philippines Corporation vs. Chrysler
standards on the dressing of chicken. The Labor Arbiter Philippines Labor Union
sustained CALS in terminating her employment. For the
G.R. No. 148738. June 29, 2004.
duration of CandelariaRocos probationary employment, she
failed to comply with Cals standards in the work assigned to
FACTS:Mitsubishi Motors Philippines Corporation (MMPC)
her. First, she frequently failed to observe the allowable
is a domestic corporation engaged in the assembly and
inches to be cut, which must only be 1.5 inches, in performing
distribution of Mitsubishi motor vehicles. Chrysler Philippines
the surgical incision of the Chicken butt, either she cuts it too
Labor Union (CPLU) is a legitimate labor organization and the
long, thereby distorting the appearance of the chickens or she
duly certified bargaining agent of the hourly paid regular rank
cuts it too short, thereby making it difficult to remove the
and file employees of MMPC. Nelson Paras was first
chicken parts without damaging these parts Second, she
employed by MMPC as a shuttle bus driver on March 19,
frequently mishandles the pullout of chicken parts, such that,
Page 13
Dan Gloria// Kat Ramirez// Jessica Bernardo// Joseph De Mesa// Arvin Figueroa// ChriszelQueano// Marvin Villardo Jr.// Bowdy Sales
1976. He resigned on June 16, 1982. He applied for and was terminationletter was served on November 26, 1996, Paras
hired as a diesel mechanic and heavy equipment operator in was still aprobationary employee.
Saudi Arabia from 1982 to 1993. When he returned to the
Philippines, he was rehired as a welder fabricator at the ISSUE:whether or not respondent Paras was already a regular
MMPC tooling shop from October 3, 1994 to October 31, employee on November 26, 1996
1994.On October 29, 1994, his contract was renewed from
November 1, 1994 up to March 3, 1995.
Sometime in May of 1996, Paras was rehired on a HELD: Indeed, an employer, in the exercise of its
probationary basis as a manufacturing trainee at the Plant management prerogative, may hire an employee on a
Engineering Maintenance Department. Paras started reporting probationary basis in order to determine his fitness to perform
for work on May 27, 1996. Hewas assigned at the paint ovens, work.Under Article 281 of the Labor Code, the employer must
air makeupandconveyors. As part of the MMPCs policy, inform the employee of the standards for which his
Paras wasevaluated by his immediate supervisors Lito employment may be considered for regularization. Such
R.Lacambacaland Wilfredo J. Lopezafter six (6) months,and probationary period, unless covered by an apprenticeship
received an average rating. Later, Lacambacalinformed Paras agreement, shall not exceed six (6) months from the date the
that based on his performance rating, hewould be employee started working. The employees services may be
regularized.However, the Department and Division Managers, terminated for just cause or for his failure to qualify as a
A.C.Velando and H.T. Victoria,including Mr. Dante regular employee based on reasonable standards made known
Ong,reviewed the performance evaluation made on Paras. to him. Respondent Paras was employed as a management
Theyunanimously agreed, along with Paras trainee on a probationary basis.
immediatesupervisors, that the performance of Paras
wasunsatisfactory.As a consequence, Paras was notconsidered During the orientation conducted on May 15, 1996,
for regularization. On November 26, 1996, hereceived a he was apprised of the standards upon which his regularization
Notice of Termination dated November 25, 1996,informing would be based. He reported for work on May 27, 1996. As
him that his services were terminated effectivethe said date per the companys policy, the probationary period was from
since he failed to meet the required company standards for three (3) months to a maximum of six (6) months. Applying
regularization. Article 13 of the Civil Code,the probationary period of six (6)
Utilizing the grievance machinery in the months consists of one hundred eighty (180) days.This is in
collectivebargaining agreement, the CPLU demanded the conformity with paragraph one, Article 13 of the Civil Code,
settlement of the dispute which arose from Paras which provides that the months which are not designated by
termination.The dispute was thereafter submitted for voluntary their names shall be understood as consisting of thirty (30)
arbitration, as the parties were unable to agree on a mutually days each. The number of months in the probationary period,
acceptable solution. CPLU posited that Paras was dismissed six (6), should then be multiplied by the number of days
on his one hundred eighty third (183rd) day of employment, or within a month, thirty (30)hence, the period of one hundred
three (3) days after the expiration of the probationary period of eighty (180) days. As clearly provided for in the last paragraph
six (6) months. It was contended that Paras was already a of Article 13,in computing a period, the first day shall be
regular employee on the date of the termination of his excluded andthe last day included. Thus, the one hundred
probationary employment. On November 3, 1997, the eighty (180)days commenced on May 27, 1996, and ended on
Voluntary Arbitrator (VA) rendered a decision finding the November23, 1996. The termination letter dated November
dismissal of Paras valid for his failure to pass the probationary 25, 1996was served on respondent Paras only at 3:00 a.m.
standards of MMPC. ofNovember 26, 1996. He was, by then, already a
Paras and CPLU asserted that pursuant to Article 13 regularemployee of the petitioner under Article 281 of the
of the New Civil Code, the period of May 27, 1996 to LaborCode.
November 26, 1996 consisted of one hundred eighty three An employee cannot be dismissed except for just
(183) days. They asserted that the maximum of orauthorized cause as found in the Labor Code and after
theprobationary period is six (6) months, which is equivalent dueprocess. The basis for which respondent Paras services
to 180 days as such, Paras, who continued to be employed wereterminated was his alleged unsatisfactory rating
even after the 180th day, had become a regular employee as arisingfrom poor performance. In the present case, the
provided for in Article 282 of the Labor Code. They averred immediate supervisor ofrespondent Paras gave him an average
that as a regular employee, Paras employment could be performance ratingand found him fit for regularization.
terminated only for just or authorized causes as provided for However, in a complete turnaround, thepetitioner made it
under the Labor Code, and after due notice. They posited that appear that after the performanceevaluation of respondent
in the Letter of Termination dated November 25, 1996, the Paras was reviewed by thedepartment and division heads, it
ground for Paras termination was not among those sanctioned was unanimously agreedthat the respondents performance
by the Labor Code hence, his dismissal was illegal. The rating wasunsatisfactory, making him unfit for regularization.
MMPC, for its part, averred that under Article 13 of the New Considering that respondent Paras was not dismissed for ajust
Civil Code, Paras probationary employment which or authorized cause, his dismissal from employmentwas
commenced on May 27, 1996 would expire on November 27, illegal.
1996. Since he received the notice of termination of his
employment on November 25, 1996, the same should be 17. Mariwasa Manufacturing, Inc. vs. Leogardo, Jr.
considered to have been served within the six-month G.R. No. 74246. January 26, 1989.
probationary period. According to the petitioner, when the

Page 14
Dan Gloria// Kat Ramirez// Jessica Bernardo// Joseph De Mesa// Arvin Figueroa// ChriszelQueano// Marvin Villardo Jr.// Bowdy Sales
Probationary employment The employer and the employee Buiser vs. Leogardo, Jr.recognized agreements
may by agreement extend the probationary period of stipulating longer probationary periods as constituting lawful
employment beyond the 6months period in Art. 282 of the exceptions to the statutory prescription limiting such periods
Labor Code. to six months, when it upheld as valid an employment contract
between an employer and two of its employees that provided
FACTS:Private respondent Joaquin A. Dequila (or Dequilla) for an eigthteen-month probation period. The single difference
was hired on probation by petitioner Mariwasa Manufacturing, between Buiserand the present case: that in the former
Inc. (hereafter, Mariwasa only) as a general utility worker on involved an eighteen-month probationary period stipulated in
January 10, 1979. Upon the expiration of the probationary the original contract of employment, whereas the latter refers
period of six months, Dequila was informed by his employer to an extension agreed upon at or prior to the expiration of the
that his work had proved unsatisfactory and had failed to meet statutory six-month period, is hardly such as to warrant or
the required standards. To give him a chance to improve his even suggest a different ruling here. In both cases the parties
performance and qualify for regular employment, instead of agreements in fact resulted in extensions of the period
dispensing with his service then and there, with his written prescribed by law.
consent Mariwasa extended his probation period for another For aught that appears of record, the extension of
three months from July 10 to October 9, 1979. His Dequilas probation was ex gratia, an act of liberality on the
performance, however, did not improve and on that account part of his employer affording him a second chance to make
Mariwasa terminated his employment at the end of the good after having initially failed to prove his worth as an
extended period. employee. Such an act cannot now unjustly be turned against
Dequila thereupon filed with the Ministry of Labor said employers account to compel it to keep on its payroll one
against Mariwasa and its VicePresident for Administration, who could not perform according to its work standards. The
Angel T. Dazo, a complaint for illegal dismissal and violation law, surely, was never meant to produce such an inequitable
of Presidential Decrees Nos. 920 and 1389. result.
His complaint was dismissed after hearing by By voluntarily agreeing to an extension of the
Director Francisco L. Estrella, Director of the Ministrys probationary period, Dequila in effect waived any benefit
National Capital Region, who ruled that the termination of attaching to the completion of said period if he still failed to
Dequilas employment was in the circumstances justified and make the grade during the period of extension. The Court
rejected his money claims for insufficiency of evidence.On finds nothing in the law which by any fair interpretation
appeal to the Office of the Minister, however, said disposition prohibits such a waiver. And no public policy protecting the
was reversed. Respondent Deputy Minister Vicente Leogardo, employee and the security of his tenure is served by
Jr. held that Dequila was already a regular employee at the proscribing voluntary agreements which, by reasonably
time of his dismissal, therefore, could not have been lawfully extending the period of probation, actually improve and
dismissed for failure to meet company standards as a further a probationary employees prospects of demonstrating
probationary worker. He was ordered reinstated to his former his fitness for regular employment.
position without loss of seniority and with full back wages
from the date of his dismissal until actually reinstated. 18. Villanueva vs. NLRC (Second Division)
Mariwasa and Dazo, now petitioners, thereafter be G.R. No. 127448. September 10, 1998.
sought this Court to review Hon. Leogardos decision on The termination of an employees employment contract on 21
certiorari and prohibition, urging its reversal for having been February 1995, as well as the subsequent issuance on 13
rendered with grave abuse of discretion and/or without or in March 1995 of a new contract for five months as data
excess of jurisdiction. encoder, was a devious, but crude, attempt to circumvent the
Art. 282.Probationary Employment.Probationary employees right to security of tenure as a regular employee.
employment shall not exceed six (6) months from the date the
employee started working, unless it is covered by an FACTS:Petitioner Juanito M. Villanueva started working with
apprenticeship agreement stipulating a longer period. The respondent Innodata Philippines, Inc.,/Innodata Processing
services of an employee who has been engaged on a Corporation as an abstractor with a daily salary of P180.
probationary basis may be terminated for a just cause or when The contract of employmentprovided for a period of
he fails to qualify as a regular employee in accordance with effectivity of one year commencing on Feb. 21, 1994, until
reasonable standards made known by the employer to the Aug. 21, 1995. It was also stipulated that from 21 February
employee at the time of his engagement. An employee who is 1994 to 21 August 1994, or for a period of six months,
allowed to work after probationary period shall be considered petitioners employment would be contractual and could be
a regular employee. terminated at whatever date within this period by mere service
of notice to that effect. However, should his employment be
ISSUE:W/N Article 282 of the Labor Code notwithstanding, continued beyond 21 August 1994, he would become a regular
probationary employment may validly be extended beyond the employee upon demonstration of sufficient skill to meet the
prescribed six-month period by agreement of the employer standards set by the respondent company. Should he fail to
and the employee. demonstrate the ability to master his task during the first six
months, he could be placed on probation for another six
HELD:The Court agrees with the Solicitor General, who takes months after which, he could be evaluated for promotion as a
the same position as the petitioners, that such an extension regular employee. On 21 February 1995, petitioners services
may lawfully be covenanted, notwithstanding the seemingly were terminated by reason of end of contract.
restrictive language of the cited provision.

Page 15
Dan Gloria// Kat Ramirez// Jessica Bernardo// Joseph De Mesa// Arvin Figueroa// ChriszelQueano// Marvin Villardo Jr.// Bowdy Sales
Three weeks thereafter, the petitioner was rehired by The termination of petitioners employment contract
the respondent corporation, this time, as a data encoder. on 21 February 1995, as well as the subsequent issuance on 13
effective 13 March 1995 to 15 August 1995, with a lesser March 1995 of a new contract for five months as data
pay of P164.10 per day.On 13 August 1995, the petitioner was encoder, was a devious, but crude, attempt to circumvent
again separated from the respondent company also on account petitioners right to security of tenure as a regular employee
of end of contract. guaranteed by Article 279 of the Labor Code.
This prompted the petitioner to file a complaint
against the respondent company and its president, Todd 19. Innodata Philippines, Inc. vs. Quejada Lopez
Solomon, for illegal dismissal with prayer for moral and G.R. No. 162839. October 12, 2006
exemplary damages and attorneys fees. In his 21 May 1996
Decision, Labor Arbiter Manuel R. Caday held that as an FACTS:Innodata Philippines, Inc., is engaged in the
abstractor engaged in processing, encoding of data, precoding, encoding/data conversion business. It employs encoders,
editing, proofreading and scoring activities which were indexers, formatters, programmers, quality/quantity staff, and
necessary and desirable in the usual business of the respondent others, to maintain its business and do the job orders of its
corporationthe petitioner was a regular employee pursuant clients. Estrella G. Natividad and Jocelyn L. Quejada were
to Article 280 of the Labor Code, who enjoyed security of employed as formatters by Innodata Philippines, Inc. They
tenure. Moreover, when he was allowed to work after 21 [worked] from March 4, 1997, until their separation on March
August 1994, when his probationary employment of six 3, 1998. Claiming that their job was necessary and desirable
months expired, he acquired a vested right to a permanent to the usual business of the company which is data
employment and could only be dismissed for a valid cause. processing/conversion and that their employment is regular
On appeal, respondent NLRC reversed the Labor pursuant to Article 280 of the Labor Code, [respondents] filed
Arbiters decision and upheld the validity of petitioners a complaint for illegal dismissal and for damages as well as
separation from the respondent company on the ground that for attorneys fees against Innodata Phils., Incorporated,
his employment contract was for a fixed period of one year Innodata Processing Corporation and Todd Solomon.
and six months, certain to end on 21 August 1994. [Respondents] further invoke the stare decicisdoctrine in the
case of JuanitoVillanuevavs. National Labor Relations
ISSUE:W/N the petitioner is a regular employee. Commission, et al., G.R. No. 127448 dated September 17,
1998 and the case of Joaquin Servidad vs.National Labor
HELD:Yes.We agree with the OSG that the contract cannot Relations Commission, et al., G.R. No. 128682 dated March
be strictly construed as one for a fixed term. For one, while the 18, 1999, arguing that the Highest Court has already ruled
first paragraph of Section 2 spoke of the contracts duration to with finality that the nature of employment at [petitioner]
be one year, it was in fact, for one year and six months corporation is regular and not on a fixed term basis, as the job
because it was to commence on 21 February 1994 and in the company is necessary and desirable to the usual
terminate on 21 August 1995. For another, while the second business of the corporation. On the other hand, [petitioner]
paragraph specified the first sixmonth period of employment, contends that [respondents] employment contracts expired,
21 February to 21 August 1994, as contractual, the third for [these were] only for a fixed period of one (1) year.
sentence of that paragraph granted the petitioner regular After examination of the pleadings filed, Labor
employment status should he continue his employment Arbiter Donato G. Quinto rendered a judgment in favor of
beyond August 21, 1994, . . . upon demonstration of sufficient complainants. National Labor Relations Commission, which
skill in terms of his ability to meet the standards set by the reversed and set aside the Labor Arbiters decision and
respondent company. It is clear that the first six months was in dismissed [respondents] complaint for lack of merit. It
reality the probation period under Article281 of the Labor declared that the contract between [respondents] and
Code,since petitioner would become a regular employee if the [petitioner] company was for a fixed term and therefore, the
employment would continue beyond that period upon dismissal of [respondents], at the end of their one year term
demonstration of sufficient skill in accord with the standards agreed upon, was valid. The CA ruled that respondents were
set by the respondent corporation. regular employees in accordance with Section 280 of the
Significantly, the respondent company alleges that it Labor Code. It said that the fixed term contract prepared by
has never placed the petitioner on probation.This could only petitioner was a crude attempt to circumvent respondents
mean that petitioners continuance in employment beyond 21 right to security of tenure.
August 1994 was not for probation purposes under the fourth
sentence of the second paragraph of Section 2 reading as ISSUE: WON the employment contract is valid.
follows: If the employee fails to demonstrate the ability to
master his task during the first six months he can be placed on HELD: No.While the Supreme Court has recognized the
probation for another six (6) months after which he will be validity of fixed termemploymentcontracts in a number of
evaluated for promotion as a regular employee. If the cases,it has consistently emphasized that when the
petitioner was thus allowed to remain in employment beyond circumstances of a caseshow that the periods were imposed to
21 August 1994, it could be for no other reason than that he block the acquisitionof security of tenure, they should be
demonstrated sufficient skill in terms of his ability to meet struck down for beingcontrary to law, morals, good customs,
the standards set by the respondent company. He, therefore, public order orpublic policy.
became a regular employee by virtue of the third sentence of
the second paragraph of Section 2 of the contract. A close scrutiny of the provisions of the employment
contract, however, show that the double-bladed scheme to

Page 16
Dan Gloria// Kat Ramirez// Jessica Bernardo// Joseph De Mesa// Arvin Figueroa// ChriszelQueano// Marvin Villardo Jr.// Bowdy Sales
block the acquisition of tenure security still exists. To stress, reasons other thanthe just and authorized causes under the
Servidadstruck down the following objectionable contract Labor Code. Settled is the rule that an employer can terminate
provisions: Section 2. This Contract shall be effective for a the services of an employee only for valid and just causes
period of 1 [year] commencing on May 10, 1994, until May which must be shown by clear and convincing evidence. The
10, 1995 unless sooner terminated pursuant to the provisions language of the contract in dispute is truly a double-
hereof. From May 10, 1994 to November 10, 1994, or for a bladedscheme to block the acquisition of the employee of
period of six (6) months, the EMPLOYEE shall be contractual tenurialsecurity. Thereunder, [the employer] has two options.
during which the EMPLOYER can terminate the It canterminate the employee by reason of expiration of
EMPLOYEEs services by serving written notice to that contract, or itmay use failure to meet work standards as the
effect. Such termination shall be immediate, or at whatever ground for theemployees dismissal. In either case, the tenor
date within the six-month period, as the EMPLOYER may of the contractjeopardizes the right of the worker to security of
determine. Should the EMPLOYEE continue his employment tenureguaranteed by the Constitution. Consequently, the
beyond November 10, 1994, he shall become a regular terms of the present contract should beconstrued strictly
employee upon demonstration of sufficient skill in the terms against petitioner, which prepared it.
of his ability to meet the standards set by the EMPLOYER. If Article 1700 of the Civil Code declares: Art. 1700.
the EMPLOYEE fails to demonstrate the ability to master his The relations between capital and labor are not merely
task during the first six months he can be placed on probation contractual. They are so impressed with public interest that
for another six (6) months after which he will be evaluated for labor contracts must yield to the common good. Therefore,
promotion as aregular employee. such contracts are subject to the special laws on labor unions,
collective bargaining, strikes and lockouts, closed shop,
In comparison, the pertinent portions of the present wages, working conditions, hours of labor and similar
employment contracts in dispute read as follows: subjects. Indeed, a contract of employment is impressed with
TERM/DURATION public interest. For this reason, provisions of applicable
1. The EMPLOYER hereby employs, engages and hires the statutes are deemed written into the contract. Hence, the
EMPLOYEE, and the EMPLOYEE hereby accepts such parties are not at liberty to insulate themselves and their
appointment as FORMATTER relationships from the impact of labor laws and regulations by
effectiveMarch 04, 1997 to March 03, 1998, a period of one simply contracting with each other.Moreover, in case of
(1) year. x xxxxxxxx doubt, the terms of a contract should be construed in favor of
TERMINATION labor.
7.1 This Contract shall automatically terminate on March 03,
1998 without need of notice or demand. 20. Buiser vs. Leogardo, Jr.
x xxxxxxxx No. L63316. July 31, 1984
7.4 The EMPLOYEE acknowledges that the EMPLOYER
entered into this Contract upon his express representation that FACTS: Petitioners were employed by the private respondent
he/she is qualified and possesses the skills necessary and GENERAL TELEPHONE DIRECTORY COMPANY as sales
desirable for the position indicated herein. Thus, the representatives and charged with the duty of soliciting
EMPLOYER is hereby granted the right to preterminate this advertisements for inclusion in a telephone directory. The
Contract within the first three (3) months of its duration upon records show that petitioners IluminadaVerBuiserand Ma.
failure of the EMPLOYEE to meet and pass the qualifications Mercedes P. Intengan entered into an Employment Contract
and standards set by the EMPLOYER and made known to the (on Probationary Status) on May 26, 1980 with private
EMPLOYEE prior to execution hereof. Failure of the respondent, a corporation engaged in the business of
EMPLOYER to exercise its right hereunder shall be without publication and circulation of the directory of the Philippine
prejudice to the automatic termination of the EMPLOYEEs Long Distance Telephone Company. Petitioner Ma. Cecilia
employment upon the expiration of this Contract or RilloAcua entered into the same employment contract on
cancellation thereof for other causes provided herein and by June 11, 1980 with the private respondent. Among others, the
law. Employment Contract (On Probationary Status) included the
The presentcontracts provide for two periods. Aside following commonprovisions:1. The company hereby
from the fixedoneyearterm set in paragraph 1, paragraph 7.4 employs the employee as telephone sales representative on a
providesfor a three-month period during which petitioner has probationary status for a period of eighteen (18) months, i.e.
the right to preterminate the employment for the failure of the from May 1980 to October 1981, inclusive. It is understood
employees to meet and pass the qualifications and standards that during the probationary period of employment, the
set by the employer and made known to the employee prior Employee may be terminated at the pleasure of the company
to their employment. Thus, although couched in ambiguous without the necessity of giving notice of termination or the
language, paragraph 7.4 refers in reality to a probationary payment of termination pay. The Employee recognizes the
period. Clearly, to avoid regularization, petitioner has again fact that the nature of thetelephone sales representatives job is
sought to resort alternatively to probationary employment and such that the companywould be able to determine his true
employment for a fixed term. character, conduct and selling
Noteworthy is the following pronouncement of this capabilities only after the publication of the directory, and that
Court in Servidad: it takes about eighteen (18) months before his worth as a
If the contract was really for a fixed term, the [employer] telephone sales representative can be fully evaluated inasmuch
should not have been given the discretion to dismiss the as the advertisement solicited by him for a particular year are
[employee] during the one year period of employment for published inthe directory only the following year.
Page 17
Dan Gloria// Kat Ramirez// Jessica Bernardo// Joseph De Mesa// Arvin Figueroa// ChriszelQueano// Marvin Villardo Jr.// Bowdy Sales
Corollary to this, the private respondent prescribed experience or training. Policy Instruction No. 11 of the
sales quotas to be accomplished or met by the petitioners. Minister of Labor and Employment has clarified any and all
Failing to meet their respective sales quotas, the petitioners doubts on the period of probationary employment. It states as
were dismissed from the service by the private respondent. follows:Probationary Employment has been the subject of
Thus, on May 27, 1981, petitioners filed with the National misunderstanding in some quarter. Some people believe six
Capital Region, Ministry of Labor and Employment, a (6) months is the probationary period in all cases. On the other
complaint for illegal dismissal with claims for backwages, hand, employees who have already served the probationary
earned commissions and other benefits. period aresometimes required to serve again on
Petitioners contend that under Articles 281-282 of the probation.Under the Labor Code, six (6) months is the general
Labor Code, having served the respondent company probationary period, but the probationary period is actually the
continuously for over six (6) months, they have become period needed to determine fitness for the job. This period, for
automatically regular employees notwithstanding an lack of a better measurement is deemed to be the period
agreement to the contrary. Articles 281-282 needed to learn the job.
read thus: Art. 282. Probationary Employment. In the case at bar, it is shown that private respondent
Probationary employment shall not exceed six (6) months Company needs at least eighteen (18) months to determine the
from the date the employee started working, unless it is character and selling capabilities of the petitioners as sales
covered by an apprenticeship agreement stipulating a longer representatives. The Company is engaged in advertisement
period. The services of an employee who has been engaged on and publication in the Yellow Pages of the PLDT Telephone
a probationary basis may be terminated for a just cause or Directories. Publication of solicited ads are only made a year
when he fails to qualify as a regular employee in accordance after the sale has been made and only then will the company
with reasonable standards made known by the employer to the be able to evaluate the efficiency, conduct, and selling ability
employee at the time of his engagement. An employee who is of its sales representatives, theevaluation being based on the
allowed to work after a probationary period shall be published ads. Moreover, an eighteen monthprobationary
considered a regular employee. (As amended by PD 850). period is recognized by the Labor Union in the private
Art. 281. Regular and Casual Employment.The provisions respondent company, which is Article V of the Collective
of written agreement to the contrary notwithstanding and Bargaining Agreement. We, therefore, hold and rule that the
regardless of the oral agreements of the parties, an probationary employment of petitioners set to eighteen (18)
employment shall be deemed to be regular where the months is legal and valid and that the Regional Director and
employee has been engaged to perform activities which are the Deputy Minister of Labor and Employment committed no
usually necessary or desirable in the usual business or trade of abuse of discretion in ruling accordingly.
the employer, except where the employment has been fixed
for a specific project or undertaking the completion or 21. Nitto Enterprises vs. National Labor Relations
termination of which has been determined at the time of the Commission
engagement of the employee or where the work or services to G.R. No. 114337. September 29, 1995
be performed is seasonal in nature and the employment is for
the duration of the season. An employment shall be deemed to FACTS: Petitioner Nitto Enterprises, a company engaged in
be casual if it is not covered by the preceding paragraph: the sale of glass and aluminum products, hired Roberto Capili
Provided, That, any employee who has rendered at least one sometime in May 1990 as an apprentice machinist, molder and
year of service, whether such service is continuous or broken, core maker as evidenced by an apprenticeship agreementfor a
shall be considered a regular employee with respect to the period of six (6) months from May 28, 1990 to November 28,
activity in which he is employed and his employment shall 1990 with a daily wage rate of P66.75 which was 75% of the
continue while such actually exists. It is petitioners applicable minimum wage.
submission that probationary employment cannot exceed six At around 1:00 p.m. of August 2, 1990, Roberto
(6) months, the only exception being apprenticeship and Capili who was handling a piece of glass which he was
learnership agreements as provided in the Labor Code. working on, accidentally hit and injured the leg of an office
secretary who was treated at a nearby hospital. Later that same
day, after office hours, private respondent entered a workshop
ISSUE:WON probationary period cannot exceed 6 months within the office premises which was not his work station.
There, he operated one of the power press machines without
HELD:No. Generally, the probationary period of employment authority and in the process injured his left thumb. Petitioner
is limited to six (6) months. The exception to this general rule spent the amount of P1,023.04 to cover the medication of
is when the parties to an employment contract may agree private respondent. The following day, Roberto Capili was
otherwise, such as when the same is established by company asked to resign.
policy or when the same is required by the natureof work to be On August 6, 1990, private respondent formally filed
performed by the employee. before the NLRC Arbitration Branch, National Capital Region
In the latter case, there is recognition of the exercise a complaint for illegal dismissal and payment of other
of managerial prerogatives in requiring a longer period of monetary benefits. On October 9, 1991, the Labor Arbiter
probationary employment, such as in the present case where rendered his decision finding the termination of private
the probationary period was set for eighteen (18) months, i.e. respondent as valid and dismissing the money claim for lack
from May, 1980 to October, 1981 inclusive, especially where of merit. On July 26, 1993, the National Labor Relations
the employee must learn a particular kind of work such as Commission issued an order reversing the decision of the
selling, or when the job requires certain qualifications, skills, Labor Arbiter. The NLRC declared that private respondent
Page 18
Dan Gloria// Kat Ramirez// Jessica Bernardo// Joseph De Mesa// Arvin Figueroa// ChriszelQueano// Marvin Villardo Jr.// Bowdy Sales
was a regular employee of petitioner. As correctly pointed out G.R. No. 58028. April 18, 1989
by the complainant, we cannot understand how an
apprenticeship agreement filed with the Department of Labor FACTS: On the first week of July, 1968, Fausta Oh was told
only on June 7, 1990 could be validly used by the Labor that she had no assignment for the next semester. Oh was
Arbiter as basis to conclude that the complainant was hired by shocked. She had been teaching in the school since 1932 for a
respondent as a plain apprentice on May 8, 1990.Clearly, continuous period of almost 33 years. And now, out of the
therefore, the complainant was respondents regular employee blue, and for no apparent or given reason, this abrupt
under Article 280 of the Labor Code, as early as May 28,1990, dismissal. Oh sued. She demanded separation pay, social
who thus enjoyed the security of tenure guaranteed in Section security benefits, salary differentials, maternity benefits and
3, Article XIII of our 1987 Constitution. moral and exemplary damages.
Petitioner assails the NLRCs finding that private The petitioner says the private respondent had not
respondent Roberto Capili cannot plainly be considered an been illegally dismissed because her teaching contract was on
apprentice since no apprenticeship program had yet been filed a yearly basis and the school was not required to rehire her in
and approved at the time the agreement was executed. 1968. The argument is that her services were terminable at the
Petitioner further insists that the mere signing of the end of each year at the discretion of the school. Significantly,
apprenticeship agreement already established an employer no explanation was given by the petitioner, and no advance
apprentice relationship. notice either, of her relief. After teaching year in and, year out
for all of thirty-two years, the private respondent was simply
ISSUE:WON Roberto Capili is a regular employee. told she could not teach any more.

HELD:Yes. The law is clear on this matter. Article 61 of the ISSUE: Whether or not Fausta Oh was illegally dismissed
Labor Code provides: Contents of apprenticeship
agreement.Apprenticeship agreements, including the main HELD: The private respondents position is no different from
rates of apprentices, shall conform to the rules issued by the that of the rank and file employees involved in Gregorio
Minister of Labor and Employment. The period of Araneta University Foundation v. NLRC, of whom the Court
apprenticeship shall not exceed six months. Apprenticeship had the following to say:
agreements providing for wage rates below the legal minimum Undoubtedly, the private respondents positions as
wage, which in no case shall start below 75% per cent of the deans and department heads of the petitioner university are
applicable minimum wage, may be enteredinto only in necessary in its usual business. Moreover, all the private
accordance with apprenticeship program duly respondents have been serving the university from 18 to 28
approved by the Minister of Labor and Employment . The years. All of them rose from the ranks starting as instructors
Ministry shall develop standard model programs of until they became deans and department heads of the
apprenticeship. university. A person who has served the University for 28
In the case at bench, the apprenticeship agreement years and who occupies a high administrative position in
between petitioner and private respondent was executed on addition to teaching duties could not possibly be a temporary
May 28, 1990 allegedly employing the latter as an apprentice employee or a casual.
in the trade of care maker/molder. On the same date, an The applicable law is the Termination Pay Law,
apprenticeship program was prepared by petitioner and which provided:SECTION 1. In cases of employment, without
submitted to the Department of Labor and Employment. a definite period, in a commercial, industrial, or agricultural
However, the apprenticeship Agreement was filed only on establishment or enterprise, the employer or the employee may
June 7, 1990. Notwithstanding the absence ofapproval by the terminate at any time the employment with just cause or
Department of Labor and Employment, the apprenticeship without just cause in the case of an employee by serving
agreement was enforced the day it was signed. Based on the written notice on the employer at least one month in advance,
evidence before us, petitioner did not comply with the or in the case of an employer, by serving such notice to the
requirements of the law. It is mandated that apprenticeship employee at least one month in advanceor one-half month for
agreements entered into by the employer and apprentice shall every year of service of the employee, whichever, is longer, a
be entered only in accordance with the apprenticeship program fraction of at least six months being considered as one whole
duly approved by the Minister of Labor and Employment. year. The employer, upon whom no such notice was served in
Prior approval by the Department of Labor and case of termination of employment without just cause may
Employment of the proposed apprenticeship program is, hold the employee liable for damages. The employee, upon
therefore, a condition sine qua non before an apprenticeship whom no such notice was served in case of termination of
agreement can be validly entered into. Hence, since the employment without just cause shall be entitled to
apprenticeship agreement between petitioner and private compensation from the date of termination of his employment
respondent has no force and effect in the absence of a valid in an amount equivalent to his salaries or wages corresponding
apprenticeship program duly approved by the DOLE, private to the required period of notice. x xx.
respondents assertion that he was hired not as an apprentice Parenthetically, R.A. No. 4670, otherwise known as
but as a delivery boy (kargador or pahinante) deserves the Magna Carta for Public School Teachers, confers security
credence. of tenure on the teacher upon appointment as long as he
possesses the required qualification. And under the present
22. CHIANG KAI SHEK SCHOOL, petitioner, vs. policy of the Department of Education, Culture and Sports, a
COURT OF APPEALS and FAUSTINA FRANCO OH, teacher becomes permanent and automatically acquires
respondents. security of tenure upon completion of three years in the

Page 19
Dan Gloria// Kat Ramirez// Jessica Bernardo// Joseph De Mesa// Arvin Figueroa// ChriszelQueano// Marvin Villardo Jr.// Bowdy Sales
service. While admittedly not applicable to the case at bar, We see no clear evidence that the individual
these rules nevertheless reflect the attitude of the government complainants were terminated either for a just cause or that
on the protection of the workers security of tenure, which is they have failed to qualify as regular employees in accordance
now guaranteed by no less than the Constitution itself. We with the standards set by respondent school made known to
find that the private respondent was arbitrarily treated by the the former at the time ofhiring. In fact, it is shown that the
petitioner, which has shown no cause for her removal nor had individual complainants were issued individual certifications
it given her the notice required by the Termination Pay Law. of employment and whose performance ratings ranged from
As the respondent court said, the contention that she did not 85% to 90% It is not denied that the complainants were hired
report one week before the start of classes is a flimsy as probationary teachers, but the reason for their termination
justification for replacing her. She had been in its employ for should nevertheless be for a valid cause or causes. It must be
all of thirty-two years. Her record was apparently clearly shown that they have failed to meet certain standards
unblemished. There is no showing of any previous strained or criteria made known to them beforehand. It cannot be said
relations between her and the petitioner. Oh had every reason that they failed to meet respondents standards because of their
to assume, as she had done in previous years, that she would high marks of performance. Hence, We see no valid reason for
continue teaching as usual. the school not to rehire them, except, of course, for some
reasons known only to the school authorities but which they
23. Espiritu Santo Parochial School vs. NLRC did not make known to herein complainants.
G.R. No. 82325.September 26, 1989
24. HANJIN HEAVY INDUSTRIES AND
FACTS: The seven individual private respondents were hired CONSTRUCTION
by the petitioner school CO., LTD., vs. FELICITO IBAEZ ET AL
on a probationary basis on June 1, 1984,1 whereupon, G.R. No. 170181. June 26, 2008
sometime between April 1 and 15, 1985,their services were
terminated. On May 8, 1985, they charged the petitioner FACTS: Petitioner HANJIN is a foreign company duly
school with unfair labor practice and illegal dismissal. The registered with the Securities and Exchange Commission to
labor arbiter ruled finding respondents guilty of unfair labor engage in the construction business in the Philippines.
practice. The school appealed to the National Labor Petitioners HakKon Kim and JhunieAdajar were employed as
RelationsCommission, but the decision of the labor arbiter Project Director and Supervisor, respectively, by HANJIN. On
wasaffirmed on February 29, 1988 except for the charge 11 April 2002, respondents FelicitoIbaez, AligwasCarolino,
ofunfair labor practice which was dismissed for Elmer Gacula, Enrique Dagotdot, RuelCalda, and four other
insufficiencyof evidence. In the present petition, the school coworkers filed a complaint before theNLRC, for illegal
submits the followingissues: dismissal with prayer for reinstatement and full backwages
1. The respondents teachers were not dismissed. Their against petitioners.
contracts have, simply, expired and not renewed. In their Position Paper dated 29 July 2002,
2. Their contracts subsist from schoolyear to respondents alleged that HANJIN hired them for various
schoolyear, and unless renewed, the same positions on different dates,Respondents stated that their tasks
automatically expire were usual andnecessary or desirable in the usual business or
trade ofHANJIN. Respondents additionally averred that they
In their comment, however, the said seven private wereemployed as members of a work pool from which
respondents teachers contend as follows: HANJINdraws the workers to be dispatched to its
Under the Manual of Regulations for Private Schools, teachers variousconstruction projects with the exception of
undergo a probationary period of three years, during which, RuelCalda,who as a warehouseman was required to work
they may be dismissedonly for a just cause. Since the said inHANJINs main office.Among the various construction
seven teachers were laid off after less than one year of projects to which they were supposedly assigned, respondents
probationary service, the layoff was illegal. named the North Harbor project in 1992-1994 Manila
International Port in 1994-1996 Batangas Port in 1996-1998
ISSUE:whether or not respondents committed acts of unfair theBatangas Pier, and La Mesa Dam.On 15 April 2002, Hanjin
labor practice in terminating the services of individual dismissed respondents from employment. Respondents
complainants claimed that at the time of their dismissal, HANJIN had
several construction projects that were still in progress, such
HELD: There is no dispute that the individual complainants as Metro Rail Transit (MRT) II and MRT III, and continued to
were probationary employees pursuant to the policy hire employees to fill the positions vacated by the respondents.
enunciated by the Bureau of Private Schools extending the Petitioners denied the respondents allegations. They
probationary employment of teachers to three (3) years.The maintained that respondents were hired as project employees
above policy, however, did not repeal or render inoperative for the construction of the LRT/MRT Line 2 Package 2 and 3
Article 282 of the Labor Code, as amended which provides Project. HANJIN and respondents purportedly executed
that:x xx The services of an employee who has been engaged contracts of employment, in which it was clearly stipulated
on a probationary basis may be terminated for a just cause or that the respondents were to be hired as project employees for
when he fails to qualify as a regular employee in accordance a period of only three months, but that the contracts may be
with reasonable standards made known to the employee at the renewed. Petitioners call attention to the fact that they
time of his engagement.xxx complied with two of the indicators of project employment, as
prescribed under Section 2.2(e) and (f) of Department Order
Page 20
Dan Gloria// Kat Ramirez// Jessica Bernardo// Joseph De Mesa// Arvin Figueroa// ChriszelQueano// Marvin Villardo Jr.// Bowdy Sales
No. 19, Series of 1993, entitled Guidelines Governingthe From the foregoing provision, the principal test for
Employment of Workers in the Construction Industry, issued determining whether particular employees are properly
by the DOLE: characterized as project employees as distinguished from
2.2 Indicators of project employment.Either one or more regular employees is whether or not the project employees
of the following circumstances, among others, may be were assigned to carry out a specific project or undertaking,
considered as indicators that an employee is a project the duration and scope of which were specified at the time the
employee. employees were engaged for that project.
(a) The duration of the specific/identified undertaking for In Abesco Construction and Development
which the worker is engaged is reasonably determinable. Corporation v.Ramirez,which also involved a construction
(b) Such duration, as well as the specific work/service to be company and its workers, this Court considered it crucial that
performed, is defined in an employment agreement and is the employees were informed of their status as project
made clear to the employee at the time of hiring. employees:
(c) The work/service performed by the employee is in The principal test for determining whether employees are
connection with the particular project/undertaking for which project employees or regular employees is whether they
he is engaged. are assigned to carry out a specific project or undertaking, the
(d) The employee, while not employed and awaiting duration and scope of which are specified at the time they are
engagement, is free to offer his services to any other engaged for that project. Such duration, as well as the
employer. particularwork/service to be performed, is defined in an
(e) The termination of his employment in the particular employmentagreement and is made clear to the employees at
project/undertaking is reported to the Department of the time of
Labor and Employment (DOLE) Regional hiring.
Office having jurisdiction over the workplace within 30
days following the date of his separation from work, using In this case, petitioners did not have that kind of
the prescribed form on employees agreement with respondents. Neither did they inform
terminations/dismissals/suspensions. respondents of the nature of the latters work at the time of
(f) An undertaking in the employment contract by the hiring. Hence, for failure of petitioners to substantiate their
employer to pay completion bonus to the project employee claim that respondents were project employees, we are
as practiced by most construction companies. (Emphasis constrained to declare them as regular employees. even
provided.) though the absence of a written contractdoes not by itself grant
regular status to respondents, sucha contract is evidence that
Petitioners argue that the Termination Report filed respondents were informed ofthe duration and scope of their
before the DOLE Regional Office (IV) in Cainta, Rizal on 11 work and their status asproject employees. In this case, where
April 2002 signifies that respondents services were engaged no other evidencewas offered, the absence of an employment
merely for the LRT/MRT Line 2 Package 2 and 3 contract putsinto serious question whether the employees were
Project.Petitioners also insist that the payment to the properlyinformed at the onset of their employment status as
respondents of a completion bonus indicates that respondents projectemployees. It is doctrinally entrenched that in illegal
were project employees. dismissal cases, the employer has the burden of proving with
clear, accurate, consistent and convincing evidence that a
ISSUE:whether respondents were project employees dismissal was valid.
Absent any other proof that the project employees
were informed of their status as such, it will be presumed that
HELD: Article 280 of the Labor Code distinguishes a project they are regular employees in accordance with Clause 3.3(a)
employee from a regular employee thus:Article 280. of Department Order No. 19, Series of 1993, which states that:
Regular and Casual EmploymentTheprovisions of written a) Project employees whose aggregate period of continuous
agreement to the contrary notwithstanding and regardless of employment in a construction company is at least one year
the oral agreement of the parties, an employment shall be shall be considered regular employees, inthe absence of a day
deemed to be regular where the employee has been engaged to certain agreed upon by the parties for the termination of their
perform activities which are usually necessary or desirable in relationship. Project employees who have become regular
the usual business or trade of the employer, except where the shall be entitled to separation pay. A day as used herein, is
employment has been fixed for aspecific project or understood to be that which must necessarily come, although
undertaking the completion or termination of which has been it may not be known exactly when. This means that where the
determined at the time ofthe engagement of the employee or final completion of a project or phase Thereof is in fact
where the work or services to be performed is seasonal in determinable and the expected completion is made known to
nature and the employment is for theduration of the season. the employee, such project employee may not beconsidered
An employment shall be deemed to be casual if it is regular, notwithstanding the oneyear duration of employment
not covered by the preceding paragraph: Provided, That, any in the project or phase thereof or the oneyear duration of two
employee who has rendered at least one year service, whether or more employments in the same project or phase of the
such service is continuous or broken, shall be considered a project.
regular employee with respect to the activity in which he is
employed and hisemployment shall continue while such On Termination Report
activity exists.

Page 21
Dan Gloria// Kat Ramirez// Jessica Bernardo// Joseph De Mesa// Arvin Figueroa// ChriszelQueano// Marvin Villardo Jr.// Bowdy Sales
Given the particular facts established in this case, Respondent countered that Visca was an independent
petitioners argument fails to persuade this Court. Petitioners contractor and was called from time to time for contractual
were not able to offer evidence to refute or works and be determined as project employees.
controvert the respondents claim that they were assigned to
various construction projects, particularly the North Harbor ISSUE/s: What are the indicators of Project Employment
Project in 19921994
Manila International Port in 1994-1996 Batangas Port in Read with Sec. 2.2 of DO 19
19961998
theBatangas Pier and La Mesa Dam.3 It also bears to note HELD: In the present case, respondents cannot be classified
that petitioners did not present other Termination Reports as project employees, since they worked continuously for
apart from that filed on 11 April petitioners from three to twelve years without any mention of
2002. The failure of an employer to file a Termination Report a "project" to which they were specifically assigned. While
with the DOLE every time a project or a phase thereof is they had designations as "foreman," "carpenter" and "mason,"
completed indicates that respondents were not project they performed work other than carpentry or masonry. They
employees. were tasked with the maintenance and repair of the furniture,
motor boats, cottages, and windbreakers and other resort
On Completion Bonus facilities. There is likewise no evidence of the project
A completion bonus, if paid as a mere afterthought, employment contracts covering respondents' alleged periods
cannot be used to determine whether or not the employment of employment. More importantly, there is no evidence that
was regular or merely for a project. Otherwise, an employer petitioners reported the termination of respondents' supposed
may defeat the workers security of project employment to the DOLE as project employees.
tenure by paying them a completion bonus at any time it is Department Order No. 19, as well as the old Policy
inclined to unjustly dismiss them. Furthermore, after Instructions No. 20, requires employers to submit a report of
examining the payroll documents submitted by petitioners, an employees termination to the nearest public employment
this Court finds that the payments termed as completion office every time his employment is terminated due to a
bonus are not the completion bonus paid in connection with completion of a project. Petitioners' failure to file termination
the termination of the project. The payrolls, bears no relevance reports is an indication that the respondents were not project
to a completion bonus. A completion bonus is paid in employees but regular employees.
connection with the completion of the project, and is not based
on a fifteen day period. This Court has held that an employment ceases to be
Secondly, the amount paid to each employee as his completion coterminous with specific projects when the employee is
bonus was uniformly equivalent to his fifteen day wages, continuously rehired due to the demands of employers
without consideration of the number of years of service business and re-engaged for many more projects without
rendered. interruption.

25. Coco Mangas Beach Resort v. Visca A project employee is one whose "employment has
G.R. No. 167045 August 29, 2008 been fixed for a specific project or undertaking, the
completion or termination of which has been determined at the
FACTS: The controversy stemmed from the five individual time of the engagement of the employee or where the work or
complaint of illegal dismissal filed by FredericoVisca et al, service to be performed is seasonal in nature and the
composed of carpenter, mason and foreman, against employment is for the duration of the season."34 Before an
Cocomangas Beach Resort and its owner. employee hired on a per-project basis can be dismissed, a
report must be made to the nearest employment office, of the
On May 8, 1999, Maria NidaIigo-Taala, the Front termination of the services of the workers every time
Desk Officer/Sales Manager, informed them not to report for completes a project, pursuant to Policy Instruction No. 20.
work since the ongoing constructions and repairs would be
temporarily suspended because they caused irritation and 26. Abesco Construction v. Ramirez
annoyance to the resort's guests; as instructed, they did not G.R. No. 141168 April 10, 2006
report for work the succeeding days; John Munro, husband of
petitioner Susan Munro, subsequently visited respondent FACTS: Petitioner company was engaged in a construction
foreman Visca and informed him that the work suspension business where respondents were hired on different dates from
was due to budgetary constraints; when respondent Visca later 1976 to 1992 either as laborers, road roller operators, painters
discovered that four new workers were hired to do or drivers.
respondents' tasks, he confronted petitioner Munro who
explained that respondents' resumption of work was not Respondents filed two separate complaints[1] for
possible due to budgetary constraints; when not less than ten illegal dismissal against the company and its General Manager
workers were subsequently hired by petitioners to do repairs in without valid reason. Petitioners allegedly dismissed them
two cottages of the resort and two workers were retained after without a valid reason and without due process of law. The
the completion without respondents being allowed to resume LA later on ordered the consolidation of the two complaints.
work, they filed their individual complaints for illegal
dismissal. Petitioners denied liability to respondents and
countered that respondents were project employees since their
Page 22
Dan Gloria// Kat Ramirez// Jessica Bernardo// Joseph De Mesa// Arvin Figueroa// ChriszelQueano// Marvin Villardo Jr.// Bowdy Sales
services were necessary only when the company had projects HELD: In the instant case, there needs to be a finding as to
to be completed. Petitioners argued that, being project whether or not the duration and scope of the project or projects
employees, respondents employment was coterminous with were determined or specified and made known to herein
the project to which they were assigned. They were not regular private respondents at the time of their engagement.
employees who enjoyed security of tenure and entitlement to
separation pay upon termination from work. The labor arbiter tried to do this, relying heavily on
the "Contract(s) Employment" presented in petitioner's
Petitioners countered that respondents were project Annexes as well as on private respondents' own Annex
employees since their services were necessary only when the "A"[10] attached to their Position Paper, and citing the fact
company had projects to be completed. that the said contracts of employment indicated the duration of
the projects to which the private respondents had been
ISSUE/s: Effect of not being notified of their status as project assigned. He then held that "(t)here is no denial that
employees complainants were assigned to work in these projects,"[11]
and concluded that they were indeed project employees.
HELD: The principal test for determining whether employees
are project employees or regular employees is whether they But the arbiter completed ignored the fact that all the
are assigned to carry out a specific project or undertaking, the "Contract(s) Employment" presented in evidence by both
duration and scope of which are specified at the time they are petitioner and private respondents had been signed only by
engaged for that project. Such duration, as well as the petitioner's president and general manager, Luis F. Ortega, but
particular work/service to be performed, is defined in an not by the employees concerned, who had precisely refused to
employment agreement and is made clear to the employees at sign them. The said contracts therefore could in no wise be
the time of hiring. deemed conclusive evidence. Thus, private respondents
faulted the labor arbiter for giving credence and probative
In this case, petitioners did not have that kind of value to said contracts. Besides, they claimed, only seven
agreement with respondents. Neither did they inform contracts in all were presented in evidence, pertaining to seven
respondents of the nature of the latters work at the time of individual employees, while there are fifteen employees
hiring. Hence, for failure of petitioners to substantiate their involved in the complaints. Moreover, these contracts,
claim that respondents were project employees, we are purportedly issued either in July or December of 1991, except
constrained to declare them as regular employees. for one dated May 1992), were all one-shot contracts of short
duration, the longest being for about five months. Now,
27. RaycorAircontrol System (RAS) v. NLRC inasmuch as petitioner had not denied nor rebutted private
respondents' allegations that they had each worked several
G.R. No. 114290. September 9, 1996 years for the petitioner, the obvious question is, why didn't
petitioner produce in evidence similar contracts for all the
FACTS: Petitioner's sole line of business is installing other years that private respondents had worked as project
airconditioning systems in the buildings of its clients. In employees? To these points, petitioner offered no explanation
connection with such installation work, petitioner hired private whatsoever.
respondents. They insist that they had been regular employees
all along, but petitioner maintains that they were project Even if the contracts presented by petitioner had been
employees who were assigned to work on specific projects of signed by the employees concerned, still, they would not
petitioner, and that the nature of petitioner's business-- mere constitute conclusive proof of petitioner's claim.
installation (not manufacturing) of aircon systems and
equipment in buildings of its clients -- prevented petitioner 28. Caramol v. NLRC
from hiring private respondents as regular employees.
G.R. No. 102973 August 24, 1993
They filed for regularization but were denied and was
successively given a termination of employment.. Private FACTS: Rogelio Caramol, a worker hired by respondent
respondents felt they were given their walking papers after ATLANTIC GULF, on a "project-to-project" basis but whose
they refused to sign a "Contract Employment" providing for, employment was renewed forty-four (44) times by the latter.
among others, a fixed period of employment which
"automatically terminates without necessity of further notice" The Labor Arbiter had earlier declared respondent
or even earlier at petitioner's sole discretion ATLANTIC GULF guilty of unfair labor practice, ordered it
to cease and desist from further committing unfair labor
NLRC believed that the projects shown was practice against petitioner, declared illegal the constructive
contrived and not reliable as no name of the project appears in dismissal of petitioner and directed respondent ATLANTIC
the contract of employment. GULF to immediately reinstate petitioner to his former
position without loss of seniority rights and others.
ISSUE/s: When the employment contract is only signed by
the president and the manager but not the employee The factual findings of the Labor Arbiter show that
concerned. petitioner was hired by respondent ATLANTIC GULF on 2
June 1983 for the position of rigger. Until the occurrence of

Page 23
Dan Gloria// Kat Ramirez// Jessica Bernardo// Joseph De Mesa// Arvin Figueroa// ChriszelQueano// Marvin Villardo Jr.// Bowdy Sales
the strike on 10 May 1986, his last assignment was at Labor Code, at the time of their agreement or
respondent ATLANTIC GULF's plant in Batangas. employment contract, was not yet promulgated. Code of
Commerce, instead, governs fixed term employment.
Petitioner claims that because of his involvement in
unionism, particularly in actively manning the picket lines, he ISSUE/s: Exception to Art. 280, a fixed period employment,
was among those who were not re-admitted after the strike. a day certain; requisites

Respondent asserted that his employment was HELD: The concept of the employee's duties as being
covered by Project Employment Contract for a particular "usually necessary or desirable in the usual business or trade
project and for a definite period of time and was dismissed of the employer" is not synonymous with or identical to
after Nauru project for the expiration of employment contract. employment with a fixed term. Logically, the decisive
determinant in term employment should not be the activities
ISSUE/s: Was the Consent knowingly and voluntarily without that the employee is called upon to perform, but the day
force, duress or improper pressure certain agreed upon by the parties for the commencement and
termination of their employment relationship, a day certain
HELD: There is no question that stipulation on employment being understood to be "that which must necessarily come,
contract providing for a fixed period of employment such as although it may not be known when." Seasonal employment,
"project-to-project" contract is valid provided the period was and employment for a particular project are merely instances
agreed upon knowingly and voluntarily the parties, without of employment in which a period, where not expressly set
any force, duress or improper pressure being brought to bear down, is necessarily implied.
upon the employee and absent any other circumstances
vitiating his consent, or where it satisfactorily appears that the It should be apparent that this settled and familiar
employer and employee dealt with each other on more or less notion of a period, in the context of a contract of employment,
equal terms with no moral dominance whatever being takes no account at all of the nature of the duties of the
exercised by the former over the latter. 4 However, where employee; it has absolutely no relevance to the character of his
from the circumstances it is apparent that periods have been duties as being "usually necessary or desirable to the usual
imposed to preclude the acquisition of tenurial security by the business of the employer," or not.
employee, they should be struck down as contrary tenurial
security by the employee, they should be struck down as Under American law 15 the principle is the same.
contrary to public policy, morals, good custom or public order. "Where a contract specifies the period of its duration, it
terminates on the expiration of such period. "A contract of
In the case before us, we find sufficiently established employment for a definite period terminates by its own terms
circumstances showing that the supposed fixed period of at the end of such period."
employment by way of a project-to-project contract has been
imposed to preclude acquisition of tenurial security by the -respondent Alegre's employment was terminated
petitioner. upon the expiration of his last contract with Brent School on
July 16, 1976 without the necessity of any notice. The advance
29. Brent School vs. Zamora written advice given the Department of Labor with copy to
G.R. No. L-48494 February 5, 1990 said petitioner was a mere reminder of the impending
expiration of his contract, not a letter of termination, nor an
FACTS: An employment contract in virtue of which Doroteo application for clearance to terminate.
R. Alegre was engaged as athletic director by Brent School.
The contract fixed a specific term for its existence, five (5) 30. Price v. Innodata Philippines Inc.
years. Some three months before the expiration of the G.R. No. 178505 September 30, 2008
stipulated period,
FACTS: INNODATA was a domestic corporation engaged in
Alegre was given a copy of the report filed by Brent the data encoding and data conversion business. It employed
School advising of the termination of his services. The stated encoders, indexers, formatters, programmers, quality/quantity
ground for the termination was "completion of contract, staff, and others, to maintain its business and accomplish the
expiration of the definite period of employment." job orders of its clients.

However, at the investigation conducted by a Labor Petitioners Cherry J. Price, Stephanie G. Domingo,
Conciliator of said report of termination of his services, and Lolita Arbilera were employed as formatters by
Alegre protested the announced termination of his INNODATA. The parties executed an employment contract
employment. He argued that although his contract did stipulate denominated as a "Contract of Employment for a Fixed
that the same would terminate on July 17, 1976, since his Period," stipulating that the contract shall be for a period of
services were necessary and desirable in the usual business of one year.
his employer, and his employment had lasted for five years, he
had acquired the status of a regular employee and could not be According to INNODATA, petitioners employment
removed except for valid cause. already ceased due to the end of their contract. Petitioners
claimed that they should be treated regular employees as they
are necessary and desirable to the usual business.
Page 24
Dan Gloria// Kat Ramirez// Jessica Bernardo// Joseph De Mesa// Arvin Figueroa// ChriszelQueano// Marvin Villardo Jr.// Bowdy Sales
HELD: It is not enough that an employee is hired for a
Petitioners finally argued that they could not be specific project or phase of work. There must also be a
considered project employees considering that their determination of, or a clear agreement on, the completion or
employment was not coterminous with any project or termination of the project at the time the employee was
undertaking, the termination of which was predetermined. engaged if the objectives of Article 280 are to be achieved.
This second requirement was not met in this case.
Respondents explained that INNODATA was
engaged in the business of data processing, typesetting, 32. Maraguinot vs. NLRC
indexing, and abstracting for its foreign clients. The bulk of G.R. No. 120969. January 22, 1998
the work was data processing, which involved data encoding. FACTS: Petitioner Alejandro Maraguinot, part of the filming
Data encoding, or the typing of data into the computer, crew, then became assistant electrician also the other
included pre-encoding, encoding 1 and 2, editing, petitioner, Enero, employed Maraguinot as member of
proofreading, and scanning. Almost half of the employees of shooting crew. Petitioner requested for minimum wage law
INNODATA did data encoding work, while the other half compliance but was conditioned to sign a blank employment
monitored quality control. Due to the wide range of services contract. Petitioner was terminated for refusing to sign twice
rendered to its clients, INNODATA was constrained to hire the blank employment contract. Thus filed the complaint.
new employees for a fixed period of not more than one year. Private respondent assert that they contract producers,
Respondents asserted that petitioners were not illegally associate producers, to produce or make movies for private
dismissed, for their employment was terminated due to the respondents and the petitioners are project employees or
expiration of their terms of employment. Petitioners contracts independent contractors.
of employment with INNODATA were for a limited period
only, commencing on 6 September 1999 and ending on 16 ISSUE/s: What are the elements before a project employee
February 2000. attains the status of a regular employee

ISSUE/s: Effect of absence in the provision in the contract of HELD: "once a project or work pool employee has been: (1)
employment of a specific project or undertaking. continuously, as opposed to intermittently, rehired by the same
employer for the same tasks or nature of tasks; and (2) these
HELD: Scrutinizing petitioners employment contracts with tasks are vital, necessary and indispensable to the usual
INNODATA, however, failed to reveal any mention therein of business or trade of the employer, then the employee must be
what specific project or undertaking petitioners were hired for. deemed a regular employee, pursuant to Article 280 of the
Although the contracts made general references to a "project," Labor Code and jurisprudence."
such project was neither named nor described at all therein.
33. Tomas Lao Construction vs. NLRC
The one-year period for which petitioners were hired GR 116781, Sept. 5, 1997
was simply fixed in the employment contracts without
reference or connection to the period required for the FACTS: Respondents were construction workers hired in
completion of a project. More importantly, there is also a different capacities for various periods alleging they were
dearth of evidence that such project or undertaking had illegally dismissed by Petitioners Tomas Lao Construction
already been completed or terminated to justify the dismissal (TLC) Thomas and James Developers (T&J) and LVM
of petitioners. In fact, petitioners alleged - and respondents Construction Corporation (LVM) however their case was
failed to dispute that petitioners did not work on just one dismissed by Labor Arbiter Velasquez on Nov. 10, 1992 after
project, but continuously worked for a series of projects for finding that respondents were project employees whose
various clients of INNODATA. employments could be terminated upon project completion.
NLRC however reversed the Labor Arbiters decision
31. Chua v. Court of Appeals declaring private respondents as regular employees who were
G.R. No. 125837 October 6, 2004 illegally dismissed.
FACTS: Private respondents Paguio et al filed a Petition. with
the SSC for SSS coverage and contributions against petitioner ISSUE: Whether or not Respondent employees were regular
Reynaldo Chua, owner of Prime Mover Construction employees?
Development, claiming that they were all regular employees
of the petitioner in his construction business. HELD: NLRC held that private respondents were regular
Respondents claimed that they were assigned by petitioner in employees who were illegally dismissed. The repeated re-
his various construction projects continuously. Chua claimed hiring and the continuing need for private respondents'
that they were not regular employees but only project services over a long span of time (the shortest, at seven [7]
employees, hence, not subject to SSS coverage. years) undeniably made them regular employees. Thus, we
ordered petitioners to reinstate private respondents to their
ISSUE/s: Effect of having no notice to the employees of the former positions without loss of seniority rights and other
nature of employment, the specific projects or specific phrase privileges with full back wages, inclusive of allowances,
thereof. computed from the time compensation was withheld up to the
time of actual reinstatement.

Page 25
Dan Gloria// Kat Ramirez// Jessica Bernardo// Joseph De Mesa// Arvin Figueroa// ChriszelQueano// Marvin Villardo Jr.// Bowdy Sales
1) There is a continuous rehiring of project
employees even after [the] cessation of a
project; and

2) The tasks performed by the alleged


"project employee" are vital, necessary and
indispensable to the usual business or trade
34. IMBUIDO, petitioner, vs. NATIONAL LABOR of the employer
RELATIONS COMMISSION, INTERNATIONAL
INFORMATION SERVICES, INC. and GABRIEL In the case at bar, the courts find the petitioner to be a
LIBRANDO, respondents. regular employee. A work pool may exist although the
[G.R. No. 114734. March 31, 2000] workers in the pool do not receive salaries and are free to seek
other employment during temporary breaks in the business,
FACTS: Petitioner was employed as a data encoder by private provided that the worker shall be available when called to
respondent International Information Services, Inc., a report for a project. Although primarily applicable to regular
domestic corporation engaged in the business of data encoding seasonal workers, this set-up can likewise be applied to project
and keypunching, from August 26, 1988 until October 18, workers insofar as the effect of temporary cessation of work is
1991 when her services were terminated. From August 26, concerned. This is beneficial to both the employer and
1988 until October 18, 1991, petitioner entered into thirteen employee for it prevents the unjust situation of "coddling labor
(13) separate employment contracts with private respondent, at the expense of capital" and at the same time enables the
each contract lasting only for a period of three (3) months. In workers to attain the status of regular employees.
September 1991, petitioner and twelve (12) other employees
of private respondent allegedly agreed to the filing of a
The Court's ruling here is meant precisely to give life
petition for certification election involving the rank-and-file
to the constitutional policy of strengthening the labor sector,
employees of private respondent. Subsequently, on October
but, we stress, not at the expense of management. Lest it be
18, 1991, petitioner received a termination letter from Edna
misunderstood, this ruling does not mean that simply because
Kasilag, Administrative Officer of private respondent,
an employee is a project or work pool employee even outside
allegedly "due to low volume of work. Petitioner alleged that
the construction industry, he is deemed, ipso jure, a regular
her employment was terminated not due to the alleged low
employee. All that we hold today is that once a project or
volume of work but because she "signed a petition for
work pool employee has been: (1) continuously, as opposed to
certification election among the rank and file employees of
intermittently, re-hired by the same employer for the same
respondents," thus charging private respondent with
tasks or nature of tasks; and (2) these tasks are vital, necessary
committing unfair labor practices. Petitioner further
and indispensable to the usual business or trade of the
complained of non-payment of service incentive leave benefits
employer, then the employee must be deemed a regular
and underpayment of 13th month pay maintained that it had
employee, pursuant to Article 280 of the Labor Code and
valid reasons to terminate petitioners employment and
jurisprudence. To rule otherwise would allow circumvention
disclaimed any knowledge of the existence or formation of a
of labor laws in industries not falling within the ambit of
union among its rank-and-file employees at the time
Policy Instruction No. 20/Department Order No. 19, hence
petitioners services were terminated. Private respondent
allowing the prevention of acquisition of tenurial security by
stressed that its business "relies heavily on companies availing
project or work pool employees who have already gained the
of its services. Its retention by client companies with particular
status of regular employees by the employer's conduct."
emphasis on data encoding is on a project to project
basis," usually lasting for a period of "two (2) to five (5)
months." Private respondent further argued that petitioners 35. MAGCALAS vs. NLRC
employment was for a "specific project with a specified period [G.R. No. 100333. March 13, 1997]
of engagement." According to private respondent, "the
certainty of the expiration of complainants engagement has FACTS: In their basic complaint and counter position paper,
been determined at the time of their (sic) engagement (until 27 the complainants alleged (inter alia) that they were all regular
November 1991) or when the project is earlier completed or employees of the respondent company, having rendered
when the client withdraws," as provided in the contract. "The continuous services in various capacities, ranging from
happening of the second event [completion of the project] has leadman, tinsmith, tradeshelper to general clerk; that the
materialized, thus, her contract of employment is deemed respondent has been engaged in the business of installing air
terminated per the Brent School ruling conditioning (should be air-conditioning) and refrigeration
equipment in its different projects and jobsites where the
ISSUE: Whether or not petitioner is a project employee? complainants have been assigned; that the complainants have
worked for a number of years, the minimum of which was one
HELD: The courts held that "a project employee or a member and a half years and the maximum (was) eight years under
of a work pool may acquire the status of a regular employee several supervisors; that on August 30, 1988, they were
when the following concur: dismissed (en masse) without prior notice and investigation,
and that their dismissals were effected for no other cause than
their persistent demands for payment of money claims as
mandated by law.

Page 26
Dan Gloria// Kat Ramirez// Jessica Bernardo// Joseph De Mesa// Arvin Figueroa// ChriszelQueano// Marvin Villardo Jr.// Bowdy Sales
On the other hand, the respondents interposed the [G.R. No. 117043. January 14, 1998]
defense of contract/project employment. The respondent
company is engaged in the business of manufacturing and FACTS: Respondent National Steel Corporation (NSC), one
installation of air-conditioning and refrigeration equipment. of the biggest modern steel mills in Southeast Asia, produces
hot rolled products, cold rolled products, tinplates and billets.
The manufacturing aspect of its operation is handled These products are in turn transformed by downstream
by its regular employees, while the installation aspect, by industries into truss assemblies, farm implements, pipe
reason of its intermittence, is carried out by its project or structures, shipbuilding and repairing materials, automotive
contract employees. The installation of the air-conditioning structures and machine parts, GI roof sheets or galvanized
equipment at the Asian Development Bank Building and (the) iron, drums, nails, fasteners and wires. The NSC embarked on
Interbank building was awarded to the respondent herein. The a Five-Year Expansion Program (FYEP) the first phase of
complainants herein were among the contract employees hired which consisted of the various projects.
by the respondent to install the air-conditioning equipment at
the Asian Development Bank and Interbank projects ISSUE: Whether or not petitioners are regular employees?
The aforesaid employees were engaged to work on
the installation projects until August 31, 1988, when their task HELD: Contracts for project employment are valid under the
was expected to be completed. This is evidenced by their law. By entering into such a contract, an employee is deemed
respective employment contracts. With the completion of their to understand that his employment is coterminous with the
task on August 31, 1988 in their respective installation project. He may not expect to be employed continuously
projects, the employment of the complainants (ipso facto) beyond the completion of the project. It is of judicial notice
expired as they had no more work to do. They now claim that that project employees engaged for manual services or those
they were illegally dismissed. for special skills like those of carpenters or masons, are, as a
rule, unschooled. However, this fact alone is not a valid reason
ISSUE: Are Petitioners Regular Workers? for bestowing special treatment on them or for invalidating a
contract of employment.
HELD: Regular employees cannot at the same time be project A project employment terminates as soon as the project is
employees. Article 280 of the Labor Code states that regular completed. Thus, an employer is allowed by law to reduce the
employees are those whose work is necessary or desirable to work force into a number suited for the remaining work to be
the usual business of the employer. The two exceptions done upon the completion or proximate accomplishment of the
following the general description of regular employees refer to project. However, the law requires that, upon completion of
either project or seasonal employees. It has been ruled in the the project, the employer must present proof of termination of
case of ALU-TUCP vs. National Labor Relations Commission the services of the project employees at the nearest public
that in the realm of business and industry, we note, that employment office.
'project' could refer to one or the other of at least two (2) The fact that petitioners were required to render
distinguishable types of activities. Firstly, a project could refer services necessary or desirable in the operation of NSCs
to particular job or undertaking that is within the regular or business for a specified duration did not in any way impair the
usual business of the employer company, but which is distinct validity of their contracts of employment which stipulated a
and separate, and identifiable as such, from the other fixed duration therefor.
undertakings of the company. Such job or undertaking begins Extant in the record are the findings of the NLRC
and ends at determined or determinable times. The typical that the petitioners in this case were utilized in operations
example of this first type of project is a particular construction other than billet making or other components of the FYEP I
job or project of a construction company. A construction and II, such as shipbreaking. We are constrained to rule that
company ordinarily carries out two or more discrete (should while it is true that they performed other activities which were
be distinct) identifiable construction projects: e.g., a twenty- necessary or desirable in the usual business of the NSC and
five-storey hotel in Makati; a residential condominium that the duration of their employment was for a period of more
building in Baguio City; and a domestic air terminal in Iloilo than one year, these factors did not make them regular
City. Employees who are hired for the carrying out of one of employees in contemplation of Article 280 of the Labor Code.
these separate projects, the scope and duration of which has Thus, the fact that petitioners worked for NSC under
been determined and made known to the employees at the different project employment contracts for several years
time of employment, are properly treated as 'project cannot be made a basis to consider them as regular employees,
employees,' and their services may be lawfully terminated at for they remain project employees regardless of the number of
completion of the project/ projects in which they have worked. Length of service is not
The employment of seasonal employees, on the other the controlling determinant of the employment tenure of a
hand, legally ends upon completion of the project or the project employee.[40] In the case of Mercado, Sr. v.
season, thus clearly, therefore, petitioners being project NLRC,[41] this Court ruled that the proviso in the second
employees, or to use the correct term, seasonal employees, paragraph of Article 280, providing that an employee who has
their employment legally ends upon completion of the project served for at least one year, shall be considered a regular
or the season. The termination of their employment cannot and employees, relates only to casual employees and not to project
should not constitute an illegal dismissal. employees.

36. Villa vs. NLRC 37. Tucor vs. NLRC


[G.R. Nos. 96608-09 May 20, 1991]
Page 27
Dan Gloria// Kat Ramirez// Jessica Bernardo// Joseph De Mesa// Arvin Figueroa// ChriszelQueano// Marvin Villardo Jr.// Bowdy Sales
such as drilling, construction, civil works, structural works,
FACTS: Petitioner is a corporation principally engaged in the mechanical works, and electrical works until the project is
moving and storage of various goods owned by military finally completed. Aside from its projects in Negros Oriental,
personnel residing within the United States military facilities petitioner also had geothermal projects in other areas.
in the Philippines. On various dates herein private respondents Petitioners Southern Negros Geothermal Production Field in
were hired as packers, drivers and utilitymen/carpenters. They Negros Oriental is divided into two phases: Palinpinon I (PAL
signed uniform company-prepared master employment I) and Palinpinon II (PAL II).
contracts. In a memorandum-letter dated July 17, 1989, the
Chief of Traffic Management of Clark Air Base reminded all To augment its manpower requirement occasioned by
agents, including petitioner of the base policy that "Employees the increased activities in the development of PAL II,
who already have passes in their possession and who fail the petitioner hired respondents with termination/expiration of
polygraph administered by an acknowledged security their respective employment were specified in their initial
company will be required to return their passes. On the same employment contracts, which, however, were renewed and
day petitioner terminated the employment of private extended on their respective expiry dates. n May 29, 1998,
respondents by sending them separate identical notices of petitioner submitted report to the Department of Labor and
termination. All of private respondents had continuously been Employment (DOLE) Regional Sub-Branch No. VII
employed by petitioner for more than a year before the in Dumaguete City, stating that six of its employees were
services were terminated.On August 2, 1989, private being terminated. Petitioner thereafter furnished the
respondents, except PacificoDizon, filed a complaint for employees uniformly worded notices of termination, stating
illegal dismissal against petitioner. that they were being terminated from employment
effective June 30, 1998 due to the substantial completion of
ISSUE: Whether or not respondents are regular employees the civil works phase of PAL II. On October 29, 1998, the six
under Art. 280 of the Labor Code? employees, herein respondents, filed before the National
Labor Relations Commission (NLRC) a complaint for illegal
HELD: An examination of the contract of employment does dismissal against petitioner.
not show that private respondents were hired for a "specific
project or undertaking" nor was the completion or termination ISSUE: Whether or not respondents are considered regular
of the alleged project for which private respondents were hired employees?
determined at the start of the employment.
The term "specific project or undertaking" under HELD: As defined by Art. 280 of the Labor Code, project
Article 280 of the Labor Code contemplates an activity which employees are those workers hired (1) for a specific project or
was commonly or habitually performed or such type of work undertaking, and (2) the completion or termination of such
which is not done on a daily basis but only for a specific project or undertaking has been determined at the time of the
duration of time or until the completion of the project. The engagement of the employee. However, petitioner failed to
services employed are thus necessary or desirable in the substantiate its claim that respondents were hired merely as
employer's usual business only for the period of time it takes project employees. A perusal of the records of the case reveals
to complete the project. Without the performance of such that the supposed specific project or undertaking of petitioner
services on a regular basis, the employer's main business is not was not satisfactorily identified in the contracts of
expected to grind to a halt. respondents.
In the case at bar, private respondents were assigned Unmistakably, the alleged projects stated in the
to do carpentry work, packing and driving, activities which are employment contracts were either too vague or imprecise to be
usually necessary and desirable in petitioners' usual business considered as the specific undertaking contemplated by
and which thus had to be done on a regular basis. law. Petitioners act of repeatedly and continuously hiring
The fact that private respondents had rendered more respondents to do the same kind of work belies its contention
than one year of service at the time of their dismissal overturns that respondents were hired for a specific project or
the petitioner's allegation that private respondents were hired undertaking. The absence of a definite duration for the
for a specific or a fixed undertaking for a limited period of project/s has led the Court to conclude that respondents are, in
time. fact, regular employees.
Private respondents are therefore regular employees
of petitioner the provisions of their contract of employment 39. Olongapo Maintenance Services Inc. vs. Chantengco
notwithstanding. GR. 156146 June 21, 2007

38. PNOC-Energy Dev Corp vs. NLRC FACTS: OMSI is a corporation engaged in the business of
G.R. No. 169353 April 13, 2007 providing janitorial and maintenance services to various
clients, including government-owned and controlled
FACTS: Petitioner PNOC-Energy Development Corporation corporations. On various dates beginning 1986, OMSI hired
is a government-owned and controlled corporation engaged in the respondents as janitors, grass cutters, and degreasers, and
the exploration, development, and utilization of energy. It assigned them at the Ninoy Aquino International Airport
undertakes several projects in areas where geothermal energy (NAIA). On January 14, 1999, OMSI terminated respondents'
has been discovered. Each geothermal project undergoes employment.
the stages of exploration, development, and utilization or
production. For each stage, several activities are undertaken
Page 28
Dan Gloria// Kat Ramirez// Jessica Bernardo// Joseph De Mesa// Arvin Figueroa// ChriszelQueano// Marvin Villardo Jr.// Bowdy Sales
Claiming termination without just cause and non- employees and were employed only when there were electrical
payment of labor standard benefits, respondents filed a jobs to be done in a particular housing unit contracted by
complaint for illegal dismissal, underpayment of wages, and petitioner. He maintained that the services of respondents as
non-payment of holiday and service incentive leave pays, with project employees were coterminous with each project. As
prayer for payment of separation pay, against OMSI. For its project employees, the time of rendition of their services was
part, OMSI denied the allegations in the complaint. It averred not fixed. Thus, there was no practical way of determining the
that when Manila International Airport Authority (MIAA) appropriate compensation of the value of respondents
awarded to OMSI the service contracts for the airport, OMSI accomplishment, as their work assignment varied depending
hired respondents as janitors, cleaners, and degreasers to do on the needs of a specific project.
the services under the contracts. OMSI informed the
respondents that they were hired for the MIAA project and ISSUE: Whether or not respondents are considered project
their employments were coterminous with the contracts. As employees?
project employees, they were not dismissed from work but
their employments ceased when the MIAA contracts were not HELD: Petitioners business, specializing in installing
renewed upon their expiration. The termination of electrical devices, needs electricians only when there are
respondents employment cannot, thus, be considered illegal. electrical devices to be installed in subdivision homes or
buildings covered by an appropriate contract. Petitioner, as an
ISSUE: Where or not respondents are project employees? electrical contractor, depends for his business on the contracts
that he is able to obtain from real estate developers and
HELD: The principal test in determining whether an builders of buildings. Thus, the work provided by petitioner
employee is a project employee is whether he/she is assigned depends on the availability of such contracts or projects. The
to carry out a specific project or undertaking, the duration and duration of the employment of his work force is not permanent
scope of which are specified at the time the employee is but coterminous with the projects to which the workers are
engaged in the project,[7] or where the work or service to be assigned. Viewed in this context, the respondents are
performed is seasonal in nature and the employment is for the considered as project employees of petitioner. A project
duration of the season.[8] A true project employee should be employee is one whose "employment has been fixed for a
assigned to a project which begins and ends at determined or specific project or undertaking, the completion or termination
determinable times, and be informed thereof at the time of of which has been determined at the time of the engagement of
hiring. In the instant case, the record is bereft of proof that the the employee or where the work or service to be performed is
respondents engagement as project employees has been seasonal in nature and the employment is for the duration of
predetermined, as required by law. OMSI did not provide the season.
convincing evidence that respondents were informed that they
were to be assigned to a specific project or undertaking when 41. Sandoval Shipyard Inc. V. NLRC
OMSI hired them. Notably, the employment contracts for the G.R. No. L-65689 and 66119, May 31, 1985
specific project signed by the respondents were never
presented. All that OMSI submitted in the proceedings a FACTS: These cases are about the dismissal of alleged
quo are the service contracts between OMSI and the project workers. Petitioner has been engaged in the building
MIAA. Clearly, OMSI utterly failed to establish by substantial and repair of vessels. It contends that each vessel is a separate
evidence that, indeed, respondents were project employees and project and that the employment of workers is terminated with
their employment was coterminous with the MIAA contract. the completion of each project. The workers contend that they
are regular workers and that the termination of one project
40. Saberola vs. Suarez does not mean the end of their employment since they can be
G.R. No. 151227 July 14, 2008 assigned to unfinished projects.
In G.R No. 65689, the private respondents were
FACTS: Petitioner is the owner and manager of G.S. Saberola assigned to the construction of LCT Catarman and after three
Electrical Services, a firm engaged in the construction months of work, the project was completed and the five
business specializing in installing electrical devices in workers were served a termination notice. The five workers
subdivision homes and in commercial and non-commercial filed a complaint for illegal dismissal. In G.R No. 66119, the
buildings. Respondents were employed by petitioner as respondents were assigned to worked on a construction of a
electricians. They worked from Monday to Saturday and, tanker ordered by Mobil Oil Philippines. Upon the yard
occasionally, on Sundays, with a daily wage of P110.00. managers recommendation, the manager terminated the
services of the welders, helpers and construction workers. All
Respondent Ronald Suarez (Suarez) was employed the termination was duly reported to the Ministry of Labor and
by petitioner from February 1995 until October 1997; while Employment.
respondent RaymundoLirasan, Jr. (Lirasan) worked from Three days later, 27 out of the 55 workers were hired
February 1995 until September 1997.[4] Respondent Lirasan for a new project and the 27 included 4 of the 17 respondents
alleged that he was dismissed without cause and due process. who filed a complaint for illegal dismissal.
He was merely informed by petitioner that his services were ISSUE: W/N the workers are project employees?
no longer needed without any explanation why he was RULING:
terminated. Both respondents claimed that they received Yes. SC hold that private respondents were project
compensation below the minimum wage. In his defense, employees whose work was coterminous with the project for
petitioner averred that respondents were part-time project which they were hired. Project employees, as distinguished
Page 29
Dan Gloria// Kat Ramirez// Jessica Bernardo// Joseph De Mesa// Arvin Figueroa// ChriszelQueano// Marvin Villardo Jr.// Bowdy Sales
from regular or non-project employees, are mentioned in them wages even if there are no projects for them to work on.
section 281 of the Labor Code as those "where the We hold, therefore, that the NLRC did not abuse its discretion
employment has been fixed for a specific project or in finding, based on substantial evidence in the records, that
undertaking the completion or termination of which has been the petitioners are only project workers of the private
determined at the time of the engagement of the employee." respondent.
Policy Instructions No. 20 of the Secretary of Labor,
which was issued to stabilize employer-employee relations in
the construction industry, provides:
Project employees are those employed in connection 43. Samson v. NLRC
with a particular construction project. Non-project G.R. No. 113166, February 1, 1996
(regular) employees are those employed by a
construction company without reference to any FACTS: This is a petition for review on certiorari under rule
particular project. 65 , herein petitioner Ismael Samson assails the decision of
Project employees are not entitled to termination pay public respondent NLRC dated November 29, 1993, which
if they are terminated as a result of the completion of declared that he was a project employee, in effect reversing
the project or any phase thereof in which they are the earlier finding of Labor Arbiter Felipe T. Garduque II that
employed, regardless of the number of projects in he is actually a regular employee. Petitioner has been
which they have been employed by a particular employed with private respondent AGPC Manila in a various
construction company. Moreover, the company is not construction projects since April 1965. On November,
required to obtain clearance from the Secretary of petitioner filed a complaint for the conversion of his
Labor in connection with such termination. employment status from project employee to regular
The petitioner cited three of its own cases wherein the employee. Petitioner alleged therein that on the basis of his
National Labor Relations Commission, Deputy Minister of considerable and continuous length of service with AGP, he
Labor and Employment Inciong and the Director of the should be considered a regular employee therefore entitled to
National Capital Region held that the layoff of its project the benefits and privileges appurtenant thereto.
employees was lawful. The public respondents in the instant Labor Arbiter, declared in his decision that petitioner
two cases acted with grave abuse of discretion amounting to should be considered a regular employee on the ground that it
lack of jurisdiction in disregarding these precedents. has not been shown that AGP&P had made the corresponding
report to the nearest Public Employment Office every time a
42. Cartagenas v Romago Electric Co. Inc. project wherein petitioner assigned had been completed and
G.R. No. 82973, September 15, 1989 his employment contract terminated as required under DOLE
FACTS: Respondent Romago is a general contractor engaged Policy Instruction No. 20. Furthermore, petitioner was not free
in contracting and subcontracting of specific building to leave anytime and to offer his services to other employees,
construction projects or undertaking such as electrical, he should considered an employee for an indefinite period
mechanical and civil engineering aspects in the repair of because he is a member of work pool from AG&P draws its
buildings and from other kindred services. Individual project employees and is considered employee thereof during
complainants and Lawrence Deguit were temporarily laid-off his membership therein, hence the completion of project does
by virtue of a memorandum issued by the respondent. In said not mean termination of the employer-employee relationship.
memorandum they were also informed that a meeting On appeal, NLRC reversed the decision of the labor
regarding the resumption of operation will be held on July 16, arbiter and dismissed the complaint for lack of merit. NLRC
1986 and that they will be notified as to when they will ruled that the evidence shows that petitioner was engaged for a
resume work. On July 28, 1986, complainants filed the instant fixed and determinable period, and there was noevidence
case for illegal dismissal but before the respondent could presented nor any allegation made by petitioner to support the
receive a copy of the complaint and the notification and labor arbiters decision. According to NLRCPolicy Instruction
summons issued by the NLRC National Capital Region No. 20 was superseded by D.O No. 19, Series of 1993, which
(actually received only on August 22, 1986, page 4, records) provides that non-compliance with the required report to the
individual complainants re-applied with the respondent and nearest PEO no longer affixes a prescription of regular
were assigned to work with its project at Robinson-EDSA. employment, and that the repeated constant hiring of project
workers for subsequent projects is permitted without such
ISSUE: Whether the petitioners are project employees of the workers being considered regular employees.
private respondent Romago Electric Company, Inc.
ISSUE: W/N Petitioner is a project or regular employee.
RULING: As an electrical contractor, the private respondent
depends for its business on the contracts it is able to obtain RULING: Petitioner argues that his being a regular employee
from real estate developers and builders of buildings. Since its is supported by evidence. It shows that he performed the same
work depends on the availability of such contracts or kind of work as rigger throughout his period of employment,
"projects," necessarily the duration of the employment of its and his task is necessary and desirable to private respondents
work force is not permanent but co-terminus with the projects usual trade or business.
to which they are assigned and from whose payrolls they are SC agrees with petitioner, with the observation that after a
paid. It would be extremely burdensome for their employer particular project has been accomplished, petitioner would
who, like them, depends on the availability of projects, if it rehired immediately the following day save for a gap of one
would have to carry them as permanent employees and pay day to one week from the last project, and that between 1965
Page 30
Dan Gloria// Kat Ramirez// Jessica Bernardo// Joseph De Mesa// Arvin Figueroa// ChriszelQueano// Marvin Villardo Jr.// Bowdy Sales
to 1977, and there was 50 occasions wherein petitioner was the dismissal of the private respondents without just cause and
hired by private respondent for a continuous period of time. ordering petitioner to reinstate them without loss of benefits
Respondent AG&P insists that a petitioner is a project and seniority rights and to pay back wages. The LA explained
employee for several reasons: that their dismissal was not actually based on the expiration of
The factual findings of respondent commission which the terms of employment because some of them were
is supported by substantial evidence is already conclusive and dismissed before the end of the contract and others after its
binding. expiration. The NLRC also affirmed the decision of LA. It
D.O No. 19 amended Policy Instruction No. 20 by ruled that the employment period need not reach six months in
doing away with the required notice of termination upon order that the private respondents could not be considered
completion of the project. contract workers because they worked even after the
His services were engaged for a fixed and expiration of their contracts of employment.
determinable period which thus makes each employment for
every project separate and distinct from one another Dissatisfied with the decision of NLRC, petitioner
It falls under the exception in Article 280 of the appealed to this Court by way of special civil action of
Labor Code to the effect that the provisions of written certiorari under Rule 65 of Rules of Court. Petitioner
agreement to the contrary notwithstanding and regardless of maintains that the private respondents were project employees
the oral agreement of the parties, an employment shall be since they were hired on a project-to-project basis. They
deemed to be regular where the employee has been engaged to cannot be regular employees because they were all employed
perform activities which are usually necessary or desirable in for less than six (6) months such that even assuming that they
the usual business or trade of the employer, except the were not project employees, they have not attained that status
employment has been fixed for a specific project or of regular employment.
undertaking the completion or termination of which has been
determined at the time of the engagement of the employee. Private claims that they were dismissed form their
employment on March 2, 1993, even though the project was
It is not disputed that petitioner had been working for not yet completed. They were terminated without just cause.
private respondent for approximately twenty-eight (28) years
as of the adjudication of his plaint by respondent NLRC, and ISSUE: W/O private respondents were project employees
that his "project-to-project" employment was renewed several
times. With the successive contracts of employment wherein
petitioner continued to perform virtually the same kind of RULING: The petition is with merit.
work, i.e., as rigger, throughout his period of employment, it is Project employee is one whose employment has been
manifest that petitioner's assigned tasks were usually fixed for a specific project or undertaking the completion or
necessary or desirable in the usual business or trade of private termination of which has been determined at the time of the
respondent.8 The repeated re-hiring and continuing need for engagement of the employee or where the work or services to
his services are sufficient evidence of the necessity and be performed is seasonal in nature and the employment is for
indispensability of such services to private respondent's the duration of the season. The court held that the length of
business or trade. Where from the circumstances it is apparent service of a project employee is not the controlling test of
that periods have been imposed to preclude the acquisition of employment tenure but whether or not the employment has
tenurial security by the employee, they should be struck down been fixed for a specific project or undertaking the completion
as contrary to public policy, morals, good customs or public or termination of which has been determined at the time of
order. As observed by the Solicitor General, the record of this engagement of the employee. The contracts of employment
case discloses, as part of petitioner's position paper, a show that the private respondents were employed with respect
certification 11 duly issued by private respondent clearly to specific project. The contracts of employment provide that
showing that the former's services were engaged by private the term is one (1) month which was the estimated project to
respondent on a continuing basis since 1965. The certification be finished. Private respondents claim not as regular
indubitably indicates that after a particular project has been employees but were terminated before the completion of the
accomplished, petitioner would be re-hired immediately the project without just cause and due process.
following day save for a gap of one (1) day to one (1) week The court has examined the standard contracts signed by
from the last project to the succeeding one. 12 There can, respondents and enunciated the three ways by which their
therefore, be no escape from the conclusion that petitioner is a employment may be terminated: One, the expiration of the one
regular employee of private respondent. month period, which was the estimated period for the
completion of the project; Two, the completion of the project
or phase of the project for which they were engaged prior to
44. D.M Consunji, Inc. v. NLRC
G.R. No. 116572, December 18, 2000 the expiration of the one month period; three, upon the finding
of unsatisfactory services or other just cause. The private
FACTS: The petitioner executed a contract by virtue of terms respondents admitted before the LA that they signed their
and conditions of employment which stipulated for a period of contract voluntarily, hence they bound themselves to be
(1) month. On March 2, 1993, private respondents terminated employed for a fixed duration knowingly and voluntarily
allegedly without regard to the date of termination as specified without force, duress or improper pressure. Being a valid
in the contracts. The private respondents then filed a contract between the private respondents and petitioners, the
complaint for illegal dismissal. LA rendered a decision finding one (1) month period has the force of law between the parties.

Page 31
Dan Gloria// Kat Ramirez// Jessica Bernardo// Joseph De Mesa// Arvin Figueroa// ChriszelQueano// Marvin Villardo Jr.// Bowdy Sales
Other respondents were terminated because their respective affirming the Decision of the Labor Arbiter dated April 17,
contracts were already expired. 2000 dismissing the petitioners complaint for illegal
dismissal and claims for under payment (sic) and non-
But however, the contracts of Alexander Agraviador payment of monetary benefits for lack of merit, and its
and JovencioMendrez had not expired when their services Resolution of November 29, 2001 denying petitioners Motion
were terminated on March 2, 1993. Petitioner claims that all for Reconsideration are hereby AFFIRMED, but MODIFIED,
private respondents were terminated because of the expiration in that although petitioners were project employees, their
of the period of the contract. Petitioner did not alleged that the dismissal as such project employees is hereby declared
premature termination of the services of private respondents ILLEGAL, and private respondent C.E. Construction
(Alexander Agraviador and JovencioMendrez) was due to the Corporation is directed to pay back wages computed from the
earlier completion of the project or any phases thereof to date of termination, i.e., May 27, 1999 for petitioners Isaac
which they were assigned or the services is unsatisfactory. Cioco, Jr., Carmelo Juanzo, Cecelio (sic) Soler and Benito
Galvadores and from June 5, 1999 for petitioners Rebie
In termination cases, the burden of proving that an Mercado, Baysa Benjamin (sic) and Rodrigo Napoles, up to
employee has been lawfully dismissed lies with the employer. the date of completion of the construction of the GTI Tower
The inescapable conclusion is that Alexander Agraviador and project.
JovencioMendrez were terminated prior to the expiration of
the period of their employment without just cause, hence The workers contend that they are regular employees,
termination is illegal. However they cannot be reinstated hence entitled to reinstatement and back wages from the time
because the project was finished. What they are entitled is the of their illegal dismissal.
payment of their salaries corresponding to the unexpired The company, however, contends that the workers are its
portions of their employment. They are entitled to the payment project employees, that they are not illegally dismissed.
of their salaries equivalent to their salary from the time of
termination until the expiration of their employment period of
one (1) month, the estimated period the project was to be ISSUE: W/N the workers were regular employees of the
completed. company.

Petitioner is ordered to pay private respondents, the


unexpired portion of their contract. RULING: The LA, NLRC, CA unanimously found that the
workers were project employees. They hold that fact the
45. Cioco v . C.E. Construction Corp., workers have been employed with company for several years,
G.R. No. 156748, September 8, 2004 did not automatically make them regular employees
considering that the definition of regular employment in
FACTS: Isaac Cioco, et al. were hired by C.E. Construction Article 280 of the Labor Code, makes specific exception with
Corporation, a domestic corporation engaged in the respect to project employment. The re-hiring of petitioners on
construction business, hired workers as carpenters and a project-to-project basis did not confer upon them regular
laborers in various construction projects from 1990 to 1999. employment status. The practice was dictated by the practical
The latest of which was GTI Tower in Makati. Prior to the consideration that experienced construction workers are more
start of every project, the workers signed individual preferred.It did not change their status as project employees
employment contracts. The contract stipulated that the period
of employment shall be co-terminus with the completion of The next issue is whether the WORKERS were
the project, unless sooner terminated by you prior to the illegally dismissed. The CA ruled that they were illegally
completion of the projects. dismissed as:
May and June 1999, workers were terminated by the company There was no evidence presented by the COMPANY
on the ground of completion of the phases of the GTI Tower to show that the WORKERS had been duly notified or
project. Alleging that they were regular employees, they filed informed beforehand of their dismissal and the reasons
for illegal dismissal. therefore.
Furthermore, the COMPANY allegedly failed to
LA arbiter rendered judgment in favor of the present evidence conclusively proving completion of the GTI
company on April 17, 2000. He ruled that the workers were Tower project or phases thereof for which the services of the
project employees as evident in the employment contracts; WORKERS had been engaged.
that due notice were afforded; required termination reports Individual notices of termination had been sent to the
were submitted to DOLE. WORKERS
Section 2(III), Rule XXIII, Book V of the Omnibus
The NLRC affirmed the LAs decision on appeal. Rules Implementing the Labor Code provides that no prior
The workers filed special civil action for certiorari with CA. notice of termination is required if the termination is brought
about by completion of the contract or phase thereof for which
On August 28, 2002, the CA rendered its now the worker has been engaged. This is because completion of
assailed decision, the dispositive portion of which is: the work or project automatically terminates the employment,
WHEREFORE, premises considered, the instant petition is in which case, the employer is, under the law, only obliged to
partially given DUE COURSE. The assailed Decision of the
National Labor Relations Commission dated October 26, 2001
Page 32
Dan Gloria// Kat Ramirez// Jessica Bernardo// Joseph De Mesa// Arvin Figueroa// ChriszelQueano// Marvin Villardo Jr.// Bowdy Sales
render a report to the DOLE on the termination of the Article 280 of the Labor Code provides the three
employment. kinds of employees a.) Regular employees are those who have
COMPANY failed to present evidence conclusively been engaged to perform activities which are usually
showing actual completion of the GTI Tower project or necessary or desirable in the usual business or trade of the
respective phases thereof for which the WORKERS had been employer; b.) Project employees are those employment has
hired been fixed for a specific project or undertaking the completion
or termination of which has been determined at the time of the
The court again review the records and it shows that engagement of the employee or where the work or services to
hat the COMPANY submitted the needed evidence. In its be performed is seasonal in nature and the employment is for
motion for reconsideration of the CAs decision, the the duration of the season; c.) casual employees those who are
COMPANY attached the following: neither regular nor project employees.
a. Progress Billing Reports clearly showing that the GTI An employee shall be deemed to be casual if it not covered by
Tower project was already 80.9203% and 81.3747% the Article 280 of the Labor Code. Provided, that any
accomplished as of May 31, 1999 and June 30, 1999, employee who has rendered at least one year of service,
respectively whether such service is continuous or broken shall be
b. Specifically, the particular form, concreting and considered a regular employee with respect to the activity in
masonry works for which the WORKERS had been hired and which he is employed and his employment shall continue while
assigned were already completed or near completion such actually exists.
The WORKERS did not question the veracity of the evidence
presented and just insisted that they are regular employees of The principal test for determining whether an
the COMPANY, hence, not liable for termination on mere employee is a project employee or a regular employee is
ground of project completion. whether the employment has been fixed for a specific project
or undertaking the completion or termination of which has
Considering the foregoing, we hold that the been determined at the time of the employee. A project
COMPANY complied with the procedural as well as the employee is one whose employment has been fixed for a
substantive requirements of due process with respect to the specific project or undertaking, the completion or termination
WORKERS termination, as found by the Labor Arbiter and of which has been determined at the time of the engagement of
the NLRC. We reverse the CA. the employee or where the work or service to be performed is
seasonal in nature and the employment is for the duration of
The termination from employment of project the season.
employees Isaac Cioco, Jr., Their award of backwages A true project employee should be assigned to a project which
computed from the date of their termination is set aside begins and ends at determined or determinable times, and be
informed thereof at the time of hiring.
46. Caseres v. Univeral Robina Sugar Milling Corp.,
G.R. No. 159343, Septemberr 28, 2007 However, petitioners contend that repeated hiring of
their services qualifies them to the status of regular
FACTS: URSMC is a corporation engaged in the sugar cane employees.
milling business. The petitioner started working 1989 and
AnditoPael in 1993 to the respondent. At the start, they signed On this score, the LA ruled:
a contract of Employment for Specific Project or Undertaking. This is further buttress[ed] by the fact that
It will be renewed from time to time, until May 1999, that they the relationship between complainants and the
informed that they no longer be renewed. Petitioners filed a respondent URSUMCO, would clearly reveal that the
complaint for illegal dismissal, regularization and other very nature of the terms and conditions of their hiring
claims. would show that complainants were required to
August 24, 1999, LA dismissed the complaint for not being perform phases of special projects which are not
substantiated with clear and convincing evidence. related to the main operation of the respondent for a
definite period, after which their services are
NLRC affirmed the LAs dismissal. available to any farm owner.

Court of Appeals (CA) dismissed the petition filed The NLRC, agreeing with the LA, further ruled that:
before it. Hence, petitioner petitions for review on Certiorari We note that complainants never bothered to
under Rule 65 of Rules of Court. deny that they voluntarily, knowingly and willfully
executed the contracts of employment. Neither was
ISSUE: W/N petitioners are seasonal/ Project/ Term there any showing that respondents exercised moral
Employees not regular Employees of Respondents dominance on the complainants. it is clear that the
contracts of employment are valid and binding on the
complainants.
RULING: The LA, NLRC, CA are one in ruling that
petitioners were not illegally dismissed as they were not Finally, the CA noted:
regular but contractual or project employees. Upon application, Caseres was interviewed and made
to understand that his employment would be co-terminus with
the phase of work to which he would be then assigned, that is
Page 33
Dan Gloria// Kat Ramirez// Jessica Bernardo// Joseph De Mesa// Arvin Figueroa// ChriszelQueano// Marvin Villardo Jr.// Bowdy Sales
until February 5, 1989 and thereafter he would be free to seek 47. Tomas Lao Construction v NLRC
employment elsewhere. Caseres agreed and signed the G.R. No. 116781
contract of employment for specific project or undertaking.
After an absence of more than five (5) months, Caseres re- FACTS: A complaint before the Labor Arbiter private
applied with respondent as a seasonal project worker assisting respondents Mario O. Labendia Sr., et al., construction
in the general underchassis reconditioning to transport units on workers hired in different capacities for various periods,
July 17, 1989. It must be noted that there were intervals in claimed they were illegally dismissed by petitioners Tomas
petitioners' respective employment contracts, and that their Lao Construction (TLC), Thomas and James Developers (T &
work depended on the availability of such contracts or J) and LVM Construction Corporation (LVM), 1 Altogether
projects. informally referred to as the "Lao Group of Companies." for
which private respondents alternatively worked.
Petitioners' repeated and successive re-employment on the
basis of a contract of employment for more than one year On 10 November 1992 Labor Arbiter Gabino A.
cannot and does not make them regular employees. Length of Velasquez Jr. dismissed the complaints after finding that
service is not the controlling determinant of the employment private respondents were project employees whose
tenure of a project employee employments could be terminated upon completion of the
projects or project phase for which they were hired.
The Court ruled:
By stressing the ruling in Villa v. National Labor In its Decision of 5 August 1994 the 4th Division of
Relations Commission that by entering into such contract, an the NLRC reversed the Labor Arbiter. It declared that private
employee is deemed to understand that his employment is respondents were regular employees who were dismissed
coterminous with the project. He may not expect to be without just cause and denied due process. The NLRC also
employed continuously beyond the completion of the project. overruled the Labor Arbiter in the fixing of the terms of
The fact that petitioners were constantly re-hired does not ipso employment of complainants uniformly at five (5) years since
facto establish that they became regular employees. Their their periods of employment as alleged in their complaints
respective contracts with respondent show that there were were never refuted by petitioners.In granting monetary awards
intervals in their employment. to complainants, the NLRC disregarded the veil of corporate
fiction and treated the three (3)corporations as forming only
In petitioner Caseres's case, while his employment one (1) entity on the basis of the admission of petitioners that
lasted from August 1989 to May 1999, the duration of his "the three (3)operated as one (1)intermingling and
employment ranged from one day to several months at a time, commingling resources, including manpower facility."
and such successive employments were not continuous.
Petitioners came to us on certiorari questioning the
With regard to petitioner Pael, his employment never reversal by the NLRC of the Labor Arbiter's Decision.
lasted for more than a month at a time. These support the On 5 September 1997 we dismissed the petition and affirmed
conclusion that they were indeed project employees, and since the 5 August 1994 Decision of the NLRC.
their work depended on the availability of such contracts or
projects, necessarily the employment of respondents work ISSUE: W/N private respondents were illegally dismissed.
force was not permanent but co-terminous with the projects to
which they were assigned and from whose payrolls they were RULING: The main thrust of petitioners expostulation is that
paid respondents have no valid cause to complain about their
employment contracts since these documents merely
Moreover, even if petitioners were repeatedly and formalized their status as project employees.They cite Policy
successively re-hired, still it did not qualify them as regular Instruction No. 20 of the Department of Labor which defines
employees, as length of service is not the controlling project employees as those employed in connection with a
determinant of the employment tenure of a project particular construction project. It applies squarely to the
employee,1[19] but whether the employment has been fixed for instant case because there the Court declared that the
a specific project or undertaking, its completion has been employment of project employees is co-terminous with the
determined at the time of the engagement of the employee. 2[20] completion of the project regardless of the number of projects
Further, the provisoin Article 280, stating that an employee in which they have worked. Petitioners thus argue that their
who has rendered service for at least one (1) year shall be dismissal from the service of private respondents was legal
considered a regular employee, pertains to casual employees since the projects for which they were hired had already been
and not to project employees completed.
We are not convinced. The principal test in determining
Petition is denied. whether particular employees are project employees
distinguished from regular employees is whether the
project employees are assigned to carry out specific project
or undertaking, the duration (and scope) of which are
specified at the time the employees are engaged for the
project. Project in the realm of business and industry refers
to a particular job or undertaking that is within the regular or
usual business of employer, but which is distinct and separate

Page 34
Dan Gloria// Kat Ramirez// Jessica Bernardo// Joseph De Mesa// Arvin Figueroa// ChriszelQueano// Marvin Villardo Jr.// Bowdy Sales
and identifiable as such from the undertakings of the their "mandarols", that is, persons who take charge in
company. Such job or undertaking begins and ends at supplying the number of workers needed by owners of various
determined or determinable times farms, but only to do a particular phase of agricultural work
necessary in rice production and/or sugar cane production,
We held that private respondents were regular after which they would be free to render services to other farm
employees who were illegally dismissed. The repeated re- owners who need their services. Respondent Labor Arbiter
hiring and the continuing need for private respondents' Luciano P. Aquino ruled in favor of private respondents and
services over a long span of time (the shortest, at seven [7] held that petitioners were not regular and permanent workers
years) undeniably made them regular employees. of the private respondents, for the nature of the terms and
conditions of their hiring reveal that they were required to
While length of time may not be a controlling test for perform phases of agricultural work for a definite period of
project employment, it can be a strong factor in determining time after which their services would be available to any other
whether the employee was hired for a specific undertaking or farm owner. The NLRC ruled in favor of private respondents
in fact tasked to perform functions which are vital, necessary affirming the decision of the respondent Labor Arbiter, with
and indispensable to the usual business or trade of the the modification of the deletion of the award for financial
employer. In the case at bar, private respondents had already assistance to petitioners.
gone through the status of project employees. But their
employments became non-coterminous with specific projects ISSUE: Whether or not petitioners employment that
when they started to be continuously re-hireddue to the continued for so many years could be considered regular and
demands of petitioners business and were re-engaged for permanent by express provision of Article 280.
many more projects without interruption. Moreover, if private
respondents were indeed employed as project employees, RULING: NO. The contention of petitioners that the second
petitioners should have submitted a report of termination to paragraph of Article 280 of the Labor Code should have been
the nearest public employment office every time their applied in their case presents an opportunity to clarify the
employment was terminated due to completion of each aforementioned provision of law. The first paragraph of
construction project. The records show that they did not. Article 280 answers the question of who are employees. It
Clearly, here was an attempt to circumvent labor laws on states that, regardless of any written or oral agreement to the
tenurial security. Settled is the rule that when periods have contrary, an employee is deemed regular where he is engaged
been imposed to preclude the acquisition of tenurial security in necessary or desirable activities in the usual business or
by the employee, they should be struck down as contrary to trade of the employer, except for project employees. A project
public morals, good customs or public order. employee has been defined to be one whose employment has
been fixed for a specific project or undertaking, the
Thus, we ordered petitioners to reinstate private completion or termination of which has been determined at the
respondents to their former positions without loss of seniority time of the engagement of the employee, or where the work or
rights and other privileges with full back wages, inclusive of service to be performed is seasonal in nature and the
allowances, computed from the time compensation was employment is for the duration of the season as in the present
withheld up to the time of actual reinstatement. Art. 279 of the case. The second paragraph of Art. 280 demarcates as "casual"
Labor Code which provides that "[a]n employee who is employees, all other employees who do not fan under the
unjustly dismissed from work shall be entitled to reinstatement definition of the preceding paragraph. The proviso, in said
without loss of seniority rights and other privileges and to his second paragraph, deems as regular employees those "casual"
full back wages, inclusive of allowances, and to his other employees who have rendered at least one year of service
benefits or their monetary equivalent computed from the time regardless of the fact that such service may be continuous or
his compensation was withheld from him up to the time of his broken. Petitioners, in effect, contend that the proviso in the
actual reinstatement. second paragraph of Art. 280 is applicable to their case and
that the Labor Arbiter should have considered them regular by
48. Mercado v NLRC virtue of said proviso. The contention is without merit. That in
G.R. No. 79869, September 5, 1991 all things, God may be glorified. The general rule is that the
office of a proviso is to qualify or modify only the phrase
FACTS: Petitioners alleged in their complaint that they were immediately preceding it or restrain or limit the generality of
agricultural workers utilized by private respondents in all the the clause that it immediately follows. The proviso is
agricultural phases of work on the 7 1/2 hectares of ace land applicable only to the employees who are deemed "casuals"
and 10 hectares of sugar land owned by the Aurora L. Cruz, but not to the "project" employees nor the regular employees
Francisco Borja, Leticia C. Borja and Sto. Nio Realty treated in paragraph one of Art. 280. Clearly, therefore,
Incorporated. Fortunato Mercado, Sr. and Leon Santillan petitioners being project employees, or, to use the correct
worked in the farm of private respondents since 1949, term, seasonal employees, their employment legally ends upon
Fortunato Mercado, Jr. and Antonio Mercado since 1972 and completion of the project or the season. The termination of
the rest of the petitioners since 1960 up to April 1979, when their employment cannot and should not constitute an illegal
they were all allegedly dismissed from their employment. dismissal.
Private respondent Aurora Cruz in her answer to petitioners'
complaint denied that said petitioners were her regular
employees and instead averred that she engaged their services,
through Spouses Fortunato Mercado, Sr. and Rosa Mercado,
Page 35
Dan Gloria// Kat Ramirez// Jessica Bernardo// Joseph De Mesa// Arvin Figueroa// ChriszelQueano// Marvin Villardo Jr.// Bowdy Sales
Petitioners also aver that private respondents are
seasonal workers whose work with the mill solely depends on
the availability of items or products to be worked upon in the
mill and who could, therefore, be temporarily laid-off or
considered on leave of absence without pay and, that the
operation of a rice and corn mill is inherently seasonal in
49. TACLOBAN SAGKAHAN RICE and CORN MILLS, nature so much so that in its normal operation, workers are
CO., and/ or TAN CHENG PIAN (alias PIANA), Owner, either temporarily laid-off and/ or deemed on leave of
petitioners, vs. THE HONORABLE NATIONAL LABOR absence without pay during off milling season and sometimes,
RELATIONS COMMISSION, SECOND DIVISION, THE even during on milling season as was done in this case.
HONORABLE EXECUTIVE LABOR ARBITER, Respondents, however, argue Contrary to petitioners
REGIONAL ARBITRATION BRANCH NO. VIII, assertion, private respondents employment does not fall under
NATIONAL LABOR RELATIONS COMMISSION, any of the two (2) exceptions provided in Art. 281 of the
TACLOBAN CITY, and, CARLITO CODILAN, Labor Code, to wit: (a) where the employment has been fixed
MAXIMO DOCENA, TEOFILO TRANGRIA, EUGENIO for a specific project or undertaking the completion or
GO, and, REYNALDO TULIN, respondents. termination of which has been determined at the time of
G.R. No. 73806. March 21, 1990 engagement of the employee or (b) where the work or
services to be performed is seasonal in nature and the
Labor Law Illegal Dismissal Regular and Casual employment is for the duration of the season.
Employment Private respondents employment was not fixed
for a specific project or undertaking the completion of which ISSUE :Whether or not private respondents are
has been determined at the time of their appointment or hiring. considered regular employees?
Nor the services performed or to be performed seasonal in
nature.The evidence on record has established that private RULING :We do not find merit in petitioners
respondents CarlitoCodilan and MaximoDocena had been position. We must first determine the real status of the former
working for petitioners for 25 years, respondent Eugenio Go as employees. The pertinent provision of the Labor Code
for 22 years, respondent TeofiloTrangria for 15 years and reads:
respondent Reynaldo Tulin for 6 years. Aside from their ART. 280.Regular and Casual Employment.
lengthy service, it should be noted that private respondents The provisions of written agreement to the contrary
employment was not notwithstanding and regardless of the oral agreement of the
fixed for a specific project or undertaking the completion or parties, an employment shall be deemed to be regular where
termination of which has been determined at the time of their the employee has been engaged to perform activities which are
appointment or hiring. Likewise, it must be borne in mind that usually necessary or desirable in the usual business or trade of
petitioners never rebutted private respondents claim that they the employer, except where the employment has been fixed
performed activities usually necessary or desirable in the usual for a specific project or undertaking the completion or
business of the former. Furthermore, the services performed or termination of which has been determined at the time of the
to be performed by private respondents are not seasonal in engagement of the employee or
nature. While it may be true that the harvest of palay is where the work or services to be performed is seasonal in
seasonal, the milling operations which is the main business of nature and the employment is for the duration of the season.
petitioners are not seasonal. The fact is that big rice mills such An employment shall be deemed to be casual if it is not
as the one owned by petitioners continue to operate and do covered by the preceding paragraph: Provided, That, any
business throughout the year even if there are only two or employee who has rendered at least one year of service,
three harvest seasons within the year. It is a common practice whether such service is continuous or broken, shall be
among farmers and rice dealers to store their palay and to have considered a regular employee with respect to the activity in
the same milled as the need arises. Thus, the milling which he is employed and his employment shall continue
operations have no letup. while such actually exists.
The evidence on record has established that private
FACTS :Petitioners submit these allegations: It appears that respondents CarlitoCodilan and MaximoDocena had been
private respondents, before their termination on July 25, 1983, working for petitioners for 25 years, respondent Eugenio Go
were all regular employees of petitioners. CarlitoCodilan and for 22 years, respondent TeofiloTrangria for 15 years and
MaximoDocena started working in 1958 Eugenio Go in 1961 respondent Reynaldo Tulin for 6 years. Aside from their
TeofiloTrangria in 1968 and Reynaldo Tulin in 1977. On July lengthy service, it should be noted that private respondents
25, 1983, petitioner Tan Cheng Pian alias Piana told private employment was not fixed for a specific project or
respondents to look for another job without giving any undertaking the completion or termination of which has been
reason. Private respondents thus filed their complaint for determined at the time of their appointment or hiring.
illegal dismissal with the Regional Office, NLRC at Tacloban Likewise, it must be borne in mind that petitioners never
City on August 23, 1983. At the hearing of September 28, rebutted private respondents claim that they performed
1983, private respondents, who had been employed elsewhere, activities usually necessary or desirable in the usual business
demanded payment of separation pay instead of seeking of the former.
reinstatement. Furthermore, the services performed or to be
performed by private respondents are not seasonal in nature.
While it may be true that the harvest of palay is seasonal, the

Page 36
Dan Gloria// Kat Ramirez// Jessica Bernardo// Joseph De Mesa// Arvin Figueroa// ChriszelQueano// Marvin Villardo Jr.// Bowdy Sales
milling operations which is the main business of petitioners themselves who refused to or were choosy in their work. As
are not seasonal. The fact is that big rice mills such as the one found by the NLRC, the record of this case is replete with
owned by petitioners continue to operate and do business complainants persistence and dogged determination in going
throughout the year even if there are only two or three harvest back to work.
seasons within the year. It is a common practice among
farmers and rice dealers to store their palay and to have the ISSUE :Whether or not the Court of Appeals erred in holding
same milled as the need arises. Thus, the milling operations that respondents, admittedly seasonal workers, were regular
have no letup. employees?
And finally, considering the number of years that
they have worked for petitioners (the lowest is 6 years), RULING :The petition has no merit. Contrary to petitioners
private respondents have long attained the status of regular contention, the CA did not err when it held that respondents
employees as defined under Art. 280 of the Labor Code. were regular employees. Article 280 of the Labor Code, as
amended, states:
50. HACIENDA FATIMA and/or PATRICIO ART. 280.Regular and Casual
VILLEGAS, ALFONSO VILLEGAS and CRISTINE Employment.The provisions of written
SEGURA, agreement to the contrary notwithstanding
petitioners, vs. NATIONAL FEDERATION OF and regardless of the oral agreement of the
SUGARCANE WORKERS FOOD AND GENERAL parties, an employment shall be deemed to
TRADE, respondents. be regular where the employee has been
G.R. No. 149440. January 28, 2003 engaged to perform activities which are
usually necessary or desirable in the usual
Labor Law Employment Regular Employee business or trade of the employer, except
Definition. [T]he test of whether or not an employee is a where the employment has been fixed for a
regular employee has been laid down in De Leon v. NLRC, in specific project or undertaking the
which this Court held: The primary standard, therefore, of completion or termination of which has been
determining regular employment is the reasonable connection determined at the time of the engagement of
between the particular activity performed by the employee in the employee or
relation to the usual trade or business of the employer. The test where the work or services to be performed
is whether the former is usually necessary or desirable in the is seasonal in nature and the employment is
usual trade or business of the employer. The connection can be for the duration of the season. An
determined by considering the nature of the work performed employment shall be deemed to be casual if
and its relation to the scheme of the particular business or it is not covered by the preceding paragraph:
trade in its entirety. Also if the employee has been performing Provided, That, any employee who has
the job for at least a year, even if the performance is not rendered at least one year of service,
continuous and merely intermittent, the law deems repeated whether such service is continuous or
and continuing need for its performance as sufficient evidence broken, shall be considered a regular
of the necessity if not indispensability of that activity to the employee with respect to the activity in
business. Hence, the employment is considered regular, but which he is employed and his employment
only with respect to such activity and while such activity shall continue while such actually exists.
exists. x xxxxxxxx x xx [T]he fact that [respondents] do not For respondents to be excluded from those classified
work continuously for one whole year but only for the as regular employees, it is not enough that they perform work
duration of the x xx season does not detract from considering or services that are seasonal in nature. They must have also
them in regular employment since in a litany of cases this been employed only for the duration of one season. The
Court has already settled that seasonal workers who are called evidence proves the existence of the first, but not of the
to work from time to time and are temporarily laid off during second, condition. The fact that respondentswith the
offseason are not separated from service in said period, but exception of Luisa Rombo, Ramona Rombo, BobongAbriga
merely considered on leave until reemployed. and Boboy Silvarepeatedly worked as sugarcane workers
for petitioners for several years is not denied by the latter.
FACTS :The facts are summarized in the NLRC Decision as Evidently, petitioners employed respondents for more than
follows: one season. Therefore, the general rule of regular employment
Contrary to the findings of the Labor Arbiter that is applicable.
complainants [herein respondents] refused to work and/or The fact that [respondents] do not work continuously
were choosy in the kind of jobs they wanted to perform, the for one whole year but only for the duration of the x xx season
records is replete with complainants persistence and dogged does not detract from considering them in regular employment
determination in going back to work. The CA affirmed that since in a litany of cases this Court has already settled that
while the work of respondents was seasonal in nature, they seasonal workers who are called to work from time to time
were considered to be merely on leave during the offseason and are temporarily laid off during off season are not
and were therefore still employed by petitioners. Moreover, separated from service in said period, but merely considered
the workers enjoyed security of tenure. Any infringement on leave until reemployed.
upon this right was deemed by the CA to be tantamount to
illegal dismissal. The appellate court found neither rhyme nor
reason in petitioners argument that it was the workers
Page 37
Dan Gloria// Kat Ramirez// Jessica Bernardo// Joseph De Mesa// Arvin Figueroa// ChriszelQueano// Marvin Villardo Jr.// Bowdy Sales
The primary standard, therefore, of
determining regular employment is the
reasonable connection between the
particular activity performed by the
51. ELVIRA ABASOLO Et Al., petitioners vs. NATIONAL employee in relation to the usual trade or
LABOR RELATIONS COMMISSION, LABOR business of the employer. The test is
ARBITER RICARDO N. OLAIREZ, LA UNION whether the former is usually necessary or
TOBACCO REDRYING CORPORATION and SEE LIN desirable in the usual business or trade of the
CHAN, respondents. employer. The connection can be
G.R. No. 118475. November 29, 2000 determined by considering the nature of the
work performed and its relation to the
Labor Law Classification of Employment Nature of ones scheme of the particular business or trade in
employment does not depend solely on the will or word of the its entirety. Also if the employee has been
employer nor on the procedure for hiring and the manner of performing the job for at least a year, even if
designating the employee, but on the nature of the activities to the performance is not continuous and
be performed by the employee, considering the employers merely intermittent, the law deems repeated
nature of business and the duration and scope of work to be and continuing need for its performance as
done.The nature of ones employment does not depend sufficient evidence of the necessity if not
solely on the will or word of the employer. Nor on the indispensability of that activity to the
procedure for hiring and the manner of designating the business. Hence, the employment is
employee, but on the nature, of the activities to be performed considered regular, but only with respect to
by the employee, considering the employers nature of such activity and while such activity exists.
business and the duration and scope of work to be done. Thus, the nature of ones employment does not
depend solely on the will or word of the employer. Nor on the
FACTS :Petitioners have been under the employ of procedure for hiring and the manner of designating the
LUTORCO for several years until their employment with employee, but on the nature of the activities to be performed
LUTORCO was abruptly interrupted sometime in March 1993 by the employee, considering the employers nature of
when Compania General de Tabaccos de Filipinas (also business and the duration and scope of work to be done.
known as TABACALERA) took over LUTORCOs tobacco In the case at bar, while it may appear that the work
operations. of petitioners is seasonal, inasmuch as petitioners have served
On March 17, 1993, the disgruntled employees the company for many years, some for over 20 years,
instituted before the NLRC Regional Arbitration Branch No. performing services necessary and indispensable to
1, San Fernando, La Union a complaint for separation pay LUTORCOs business, serve as badges of regular
against private respondent LUTORCO on the ground employment. Moreover, the fact that petitioners do not work
that there was a termination of their employment due to the continuously for one whole year but only for the duration of
closure of LUTORCO as a result of the sale and turnover to the tobacco season does not detract from considering them in
TABACALERA. regular employment since in a litany of cases this Court has
On December 29, 1993, Labor Arbiter Ricardo N. already settled that seasonal workers who are called to work
Olairez rendered his decision dismissing the complaint for from time to time and are temporarily laid off during off-
lack of merit. Petitioners appealed then the decision of the season are not separated from service in said period, but are
Labor Arbiter to the public respondent NLRC where it was merely considered on leave until reemployed.
assigned to the Third Division
In its Opposition to Appeal dated February 5, 1994 52. KIMBERLY-CLARK (PHILS.), INC., petitioner, vs.
private respondent LUTORCO presented new allegations and SECRETARY OF LABOR, Respondent
a different stand for denying separation pay. It alleged that G.R. No. 156668. November 23, 2007
LUTORCO never ceased to operate but continues to operate Labor Law Regular Employees Owing to their
even after TABACALERA took over the operations of its length of service with the company, these workers became
redrying plant in Aringay, La Union. Petitioners were not regular employees, by operation of law, one year after they
terminated from employment but petitioners instead refused to were employed by KIMBERLY through RANK. While the
work with TABACALERA, despite the notice to petitioners to actual regularization of these employees entails the
return to work in view of LUTORCOs need for workers at its mechanical act of issuing regular appointment papers and
Agoo plant which had approximately 300,000 kilos of compliance with such other operating procedures as may be
Virginia tobacco for processing and redrying. Furthermore, adopted by the employer, it is more in keeping with the intent
petitioners are not entitled to separation pay because and spirit of the law to rule that the status of regular
petitioners are seasonal workers. employment attaches to the casual worker on the day
immediately after the end of his first year of service.
ISSUE :Whether petitioners are regular or seasonal workers,
as defined by law? FACTS :On November 13, 1986, MOLE issued an Order
RULING :The test of whether or not an employee is a regular stating, among others, that the casual workers not performing
employee has been laid down in De Leon v. NLRC,25 janitorial and yard maintenance services had attained regular
in which this Court held: status on even date. Pursuant thereto, on August 1, 2000, the
Bureau of Working Conditions (BWC) submitted its report
Page 38
Dan Gloria// Kat Ramirez// Jessica Bernardo// Joseph De Mesa// Arvin Figueroa// ChriszelQueano// Marvin Villardo Jr.// Bowdy Sales
finding 47 out of the 76 complainants as entitled to be work reinforces the conviction that Lagrama was indeed an
regularized. employee of petitioner.
Kimberly filed a motion for reconsideration of the
DOLE Order as well as the BWC Report, arguing in the main FACTS :Petitioner Rolando Tan is the president of Supreme
that the decision in G.R. Nos. 77629 and 78791 only pertained Theater Corporation and the general manager of Crown and
to casuals who had rendered one year of service as of April 21, Empire Theaters in Butuan City. Private respondent
1986, the filing date of KILUSANOLALIAs petition for LeovigildoLagrama is a painter, making ad billboards and
certification election. murals for the motion pictures shown at the Empress,
Kimberly, in this case, contends that the reckoning Supreme, and Crown Theaters for more than 10 years.
point in determining who among its casual employees are Lagrama filed a complaint with the Sub-Regional
entitled to regularization should be April 21, 1986, the date Arbitration Branch No. X of the National Labor Relations
KILUSANOLALIA filed a petition for certification election to Commission (NLRC) in Butuan City. He alleged that he had
challenge the incumbency of UKCEO-PTGWO. been illegally dismissed and sought reinvestigation and
payment of 13th month pay, service incentive leave pay,
ISSUE :Whether or not the petitioners are considered regular salary differential, and damages.
employees? Petitioner Tan denied that Lagrama was his
employee. He asserted that Lagrama was an independent
RULING :We do not agree with KIMBERLY. contractor who did his work according to his methods, while
The law thus provides for two kinds of regular he (petitioner) was only interested in the result thereof.
employees, namely: (1) those who are engaged to perform
activities which are usually necessary or desirable in the usual ISSUE : Whether or not an employer- employee relationship
business or trade of the employer and (2) those who have existed between petitioner and private respondent?
rendered at least one year of service, whether continuous or
broken, with respect to the activity in which they are RULING :In determining whether there is an
employed. The individual petitioners herein who have been employeremployee relationship, we have applied a fourfold
adjudged to be regular employees fall under the second test, to wit: (1) whether the alleged employer has the power
category. These are the mechanics, electricians, machinists, of selection and engagement of employees (2) whether he has
machine shop helpers, warehouse helpers, painters, carpenters, control of the employee with respect to the means and
pipefitters and masons. It is not disputed that these workers methods by which work is to be accomplished (3) whether he
have been in the employ of KIMBERLY for more than one has the power to dismiss and (4) whether the employee was
year at the time of the filing of the petition for certification paid wages. These elements of the employer-employee
election by KILUSAN-OLALIA. relationship are present in this case.
Owing to their length of service with the company, First. The existence in this case of the first element is
these workers became regular employees, by operation of law, undisputed. It was petitioner who engaged the services of
one year after they were employed by KIMBERLY through Lagrama without the intervention of a third party.
RANK. While the actual regularization of these employees Second. That petitioner had the right to hire and fire
entails the mechanical act of issuing regular appointment was admitted by him in his position paper submitted to the
papers and compliance with such other operating procedures NLRC.
as may be adopted by the employer, it is more in keeping with By stating that he had the right to fire Lagrama,
the intent and spirit of the law to rule that the status of regular petitioner in effect acknowledged Lagrama to be his
employment attaches to the casual worker on the day employee. For the right to hire and fire is another important
immediately after the end of his first year of service. To rule element of 13 the employer-employee relationship. Indeed, the
otherwise, and to instead make their regularization dependent fact that, as petitioner himself said, he waited for Lagrama to
on the happening of some contingency or the fulfillment of report for work but the latter simply stopped reporting for
certain requirements, is to impose a burden on the employee work reinforces the conviction that Lagrama was indeed an
which is not sanctioned by law. employee of petitioner. For only an employee can nurture such
an expectancy, the frustration of which, unless satisfactorily
53. ROLANDO Y TAN, petitionervs LEOVOGILDO explained, can bring about some disciplinary action on the part
LAGRAMA and THE HONORABLE COURT OF of the employer.
APPEALS, respondents. Third. Payment of wages is one of the four factors to
G.R. No. 151228. August 15, 2002 be considered in determining the existence of employer-
employee relation. That Lagrama worked for Tan on a fixed
Labor Law Employee-Employee Relationship piece-work basis is of no moment. Payment by result is a
Four-Fold Test to Determine the Existence of an method of compensation and does not define 15 the essence of
Employer-Employee Relationship. The right to hire and fire is the relation.
another important element of the employer-employee This Court has held that if the employee has been
relationship.By stating that he had the right to fire Lagrama, performing the job for at least one year, even if not
petitioner in effect acknowledged Lagrama to be his continuously but intermittently, the repeated and continuing
employee. For the right to hire and fire is another important need for its performance is sufficient evidence of the
element of the employer-employee relationship. Indeed, the necessity, if not indispensability, of that activity to the
fact that, as petitioner himself said, he waited for Lagrama to business of his employer. Hence, the employment is also
report for work but the latter simply stopped reporting for
Page 39
Dan Gloria// Kat Ramirez// Jessica Bernardo// Joseph De Mesa// Arvin Figueroa// ChriszelQueano// Marvin Villardo Jr.// Bowdy Sales
considered regular, although with respect only to such 24 commencement and termination of their employment
activity, and while such activity exists. relationship, a day certain being understood to be that which
must necessarily come, although it may not be known when.
Seasonal employment, and employment for a particular
project are merely instances of employment in which a period,
54.BRENT SCHOOL, INC., and REV. GABRIEL where not expressly set down, is necessarily implied.
DIMACHE, petitioners, vs. RONALDO ZAMORA, the Accordingly, and since the entire purpose behind the
Presidential Assistant for Legal Affairs, Office of the development of legislation culminating in the present Article
President, and DOROTEO R. ALEGRE, respondents. 280 of the Labor Code clearly appears to have been, as already
G.R. No. 48494. February 5, 1990. observed, to prevent circumvention of the employees right to
be secure in his tenure, the clause in said article
Same Same Same Stipulations in employment indiscriminately and completely ruling out all written or oral
contracts providing for term employment or fixed period agreements conflicting with the concept of regular
employment are valid when the period where agreed upon employment as defined therein should be construed to refer to
knowingly, and voluntarily by the parties without force, duress the substantive evil that the Code itself has singled out:
or improper pressure exerted on the employee and when such agreements entered into precisely to circumvent security of
stipulations were not designed to circumvent the laws on tenure. It should have no application to instances where a
security of tenure. Accordingly, and since the entire purpose fixed period of employment was agreed upon knowingly and
behind the development of legislation culminating in the voluntarily by the parties, without any force, duress or
present Article 280 of the Labor Code clearly appears to have improper pressure being brought to bear upon the employee
been, as already observed, to and absent any other circumstances vitiating his consent, or
where it satisfactorily appears that the employer and employee
FACTS : The root of the controversy at bar is an employment dealt with each other on more or less equal terms with no
contract in virtue of which Doroteo R. Alegre was engaged as moral dominance whatever being exercised by the former over
athletic director by Brent School, Inc. at a yearly the latter. Unless thus limited in its purview, the law would be
compensation of P20,000.00. The contract fixed a specific made to apply to purposes other than those explicitly stated by
term for its existence, five (5) years, i.e., from July 18, 1971, its framers it thus becomes pointless and arbitrary, unjust in
the date of execution of the agreement, to July 17, 1976. Some its effects and apt to lead to absurd and unintended
three months before the expiration of the stipulated period, or consequences.
more precisely on April 20, 1976, Alegre was given a copy of
the report filed by Brent School with the Department of Labor 55. PHILIPPINE NATIONAL OIL COMPANY-ENERGY
advising of the termination of his services effective on July 16, DEVELOPMENT CORPORATION/FRANCIS
1976. PALAFOX, petitioners, vs. NATIONAL LABOR
The Director pronounced the ground relied upon by RELATIONS COMMISSION and FRANCISCO MATA,
the respondent (Brent) in terminating the services of the respondents.
complainant (Alegre) x xx (as) not sanctioned by P.D. 442, G.R. No. 97747. March 31, 1993.
and, quite oddly, as prohibited by Circular No. 8, series of
1969, of the Bureau of Private Schools.Brent School filed a Labor Law Classification of laborers or employees
motion for reconsideration. The Regional Director denied the Private respondent Mata is a project employee considering
motion and forwarded the 8 case to the Secretary of Labor 9 that he does not belong to a work pool.Paraphrasing Rada v.
for review. The latter sustained the Regional Director. NLRC, it is clear that private respondent Mata is a project
employee considering that he does not belong to a "work pool"
ISSUE :Is it then the legislative intention to outlaw from which petitioner PNOC would draw workers for
stipulations in employment contracts laying down a definite assignment to other projects at its discretion. It is likewise
period therefor? apparent from the facts of the case that private respondent
Mata was utilized only for one particular project, the
RULING :From the premisethat the duties of an employee well-completion project which was part of the exploration
entail activities which are usually necessary or desirable in stage of the PNOC Bacon-Manito Geothermal Project. Hence,
the usual business or trade of the employerthe conclusion private respondent Mata can be dismissed upon the
does not necessarily follow that the employer and employee termination
should be forbidden to stipulate any period of time for the
performance of those activities. There is nothing essentially FACTS :When petitioner Philippine National Oil
contradictory between a definite period of an employment Company-Energy Development Corporation started to develop
contract and the nature of the employees duties set down in the Bacon- Manito Geothermal Project in Bonga, Sorsogon,
that contract as being usually necessary or desirable in the one question which arose is whether or not an employee
usual business or trade of the employer. The concept of the contracted to drive for petitioner during the construction of the
employees duties as being usually necessary or desirable in steam wells is considered a project employee or a regular
the usual business or trade of the employer is not employee.
synonymous with or identical to employment with a fixed On November 8, 1985, private respondent
term. Logically, the decisive determinant in term employment complained of illegal dismissal, and accused petitioners of
should not be the activities that the employee is called upon to withholding his backwages, overtime pay, and separation
perform, but the day certain agreed upon by the parties for the pay.Petitioners maintained that private respondent was a
Page 40
Dan Gloria// Kat Ramirez// Jessica Bernardo// Joseph De Mesa// Arvin Figueroa// ChriszelQueano// Marvin Villardo Jr.// Bowdy Sales
project employee whose employment was for a definite period employment as a college dean on April 24, 2000. On August
and coterminous with the project for which he was hired. It 22, 2000, respondents appointment as dean was confirmed.
was for this reason that his employment was terminated. Sometime in August 2000, respondent was charged with
violating AMAs Employees Conduct and Discipline 7
ISSUE : Whether or not Mata is considered a project provided in its Orientation Handbook (Handbook)
employee? On October 27, 2000, respondent filed a Complaint for Illegal
Dismissal, Illegal Suspension, Non-Payment of Salary and
RULING :As can be gleaned from the said case, the two 13th Month Pay with prayer for Damages and Attorneys Fees
guidelines, by which fixed contracts of employments can be against AMA and the rest of the petitioners. Trial on the
said NOT to circumvent security of tenure, are either: merits ensued.
1. The fixed period of employment was
knowingly and voluntarily agreed upon by the ISSUE : Whether respondent is a regular, probationary, or
parties, without any force, duress or improper fixed term employee?
pressure being brought to bear upon the employee
and absent any other circumstances vitiating his RULING :Petitioners submit that the nature of respondents
consent or: employment as dean is one with a fixed term. We agree. We
2. It satisfactorily appears that the employer held that Article 280 of the Labor Code does not pro- scribe or
and employee dealt with each other on more or less prohibit an employment contract with a fixed period. Even if
equal terms with no moral dominance whatever being the duties of the employee consist of activities necessary or
exercised by the former on the latter. desirable in the usual business of the employer, the parties are
Paraphrasing Rada v. NLRC, it is clear that private respondent free to agree on a fixed period of time for the performance of
Mata is a project employee considering that he does not such activities. There is nothing essentially contradictory
belong to a "work pool" from which petitioner PNOC would between a definite period of employment and the nature of the
draw workers for assignment to other projects at its discretion. employees duties.
It is likewise apparent from the facts of the case that private The letter of appointment was clear. Respondent was
respondent Mata was utilized only for one particular project, confirmed as Dean of AMA College, Paraaque, effective
the well completion project which was part of the exploration from April 17, 2000 to September 17, 2000. In numerous
stage of the PNOC Bacon- Manito Geothermal Project. Hence, cases decided by this Court, we had taken notice, that by way
private respondent Mata can be dismissed upon the of practice and tradition, the position of dean is normally an
termination of the projects as there would be no need for his employment for a fixed term. Although it does not appear on
services. We should not expect petitioner to continue on hiring recordand neither was it alleged by any of the partiesthat
private respondent in the other phases of the project when his respondent, other than holding the position of dean,
services will no longer be needed. concurrently occupied a teaching position, it can be deduced
from the last paragraph of said letter that the respondent shall
56. AMA COMPUTER COLLEGE, PARAAQUE, be considered for a faculty position in the event he gives up
and/or AMABLE C. AGUILUZ IX, President, MRS. his deanship or fails to meet AMAs standards. Such provision
CELESTE BANSALE, School Director, MS. SOCORRO, reasonably serves the intention set forth in Brent School that
MR. PATRICK AZANZA, GRACE BERANIA and the deanship may be rotated among the other members of the
MAJAL JACOB, petitioners, vs. ROLANDO A. faculty.
AUSTRIA, respondent.
G.R. No. 164078. November 23, 2007 57. Pantranco North Express Inc. v. NLRC
G.R. No. 106654, Dec. 16, 1994
Same Labor Standards Fixedterm Employment
Article 280 of the Labor Code does not proscribe or prohibit FACTS: Private respondent Peronila was employed as a
an employment contract with a fixed period. Even if the duties driver of Pantranco North Express, Inc. In 1973, Peronila was
of the employee consist of activities necessary or desirable in administratively investigated by the corporation for his
the usual business of the employer, the parties are free to absence from work of more than two and one-half months
agree on a fixed period of time for the performance of such without leave. According to an investigation report of
activities.We held that Article 280 of the Labor Code does petitioners' area manager, Peronila claimed that he went on
not proscribe or prohibit an employment contract with a fixed absence without leave from his work from November 1, 1972
period. Even if the duties of the employee consist of activities up to February 16, 1973 which was date of the investigation,
necessary or desirable in the usual business of the employer, or one hundred seven calendar days continuously, because "he
the parties are free to agree on a fixed period of time for the went to Cotabato, Mindanao to visit his dead grandfather
performance of such activities. There is nothing essentially during the period of his unofficial absence." Finding the
contradictory between a definite period of employment and belated explanation of Peronila insufficient, the petitioner
the nature of the employees duties. dismissed the respondent from service upon receipt of the
approved clearance from the NLRC.Fifteen years after such
FACTS :Petitioner AMA Computer College, Paraaque termination of his employment, Peronila reappeared in 1988
(AMA) is an educational institution duly organized under the and implored petitioner to reconsider his dismissal, which plea
laws of the Philippines. The rest of the petitioners are principal was initially denied by petitioner. However, due to insistent
4 officers of AMA. Respondent Rolando A. Austria appeals by Peronila, petitioner eventually acceded and hired
(respondent) was hired by AMA on probationary 5 him as a driver, but on a contractual basis for a fixed period of
Page 41
Dan Gloria// Kat Ramirez// Jessica Bernardo// Joseph De Mesa// Arvin Figueroa// ChriszelQueano// Marvin Villardo Jr.// Bowdy Sales
one month.Barely fifteen days from such employment as a factual findings of the NLRC and the labor arbiter will
contractual driver, or on April 20, 1988, private respondent necessitate a review of such factual findings. The impugned
was involved in a vehicular mishap in Nueva Vizcaya wherein decision of respondent commission appears to have laid too
the bus he was driving hit another vehicle. After an much stress on the conceptual principles of social justice in
administrative investigation conducted by petitioner labor cases without the corresponding specifics to support its
corporation, Peronila was found guilty thereof, hence his conclusions.
employment contract was terminated and was no longer
renewed thereafter. 58. Caparoso v. NLRC, G.R. No. 155505, 02.15.200
On January 18, 1989, private respondent filed a case for
illegal dismissal against petitioner in the Arbitration Branch of FACTS: Composite Enterprises Incorporated (Composite) is
the NLRC-NCR wherein he argued that he was refused engaged in the distribution and supply of confectioneries to
assignment after May 5, 1988, which refusal was tantamount various retail establishments within the Philippines. Emilio
to constructive dismissal. Accordingly, he sought his M. Caporoso and Joeve P. Quindipan (Quindipan) were
reinstatement and the payment of his back wages. Labor Composites deliverymen. Caparoso alleged that he was hired
Arbiter Patricio P. Libo-on dismissed the case on February 12, on 8 November 1998 while Quindipan alleged that he was
1991. On appeal, public respondent NLRC set aside the hired on intermittent basis since 1997. Quindipan further
decision of the labor arbiter declaring that the dismissal was alleged that he had been working continuously with
illegal since there was no just cause. However, respondent Composite since August 1998.On 8 October
commission rejected the argument of Peronila that he was not 1999, Caparoso and Quindipan (petitioners) were dismissed
afforded the opportunity to adduce evidence before the labor from the service. They filed a consolidated position paper
arbiter. The NLRC maintained that there was no error in the before the Labor Arbiter charging Composite
procedure conducted by the labor arbiter because he is given and its Personnel Manager Edith Tan (Tan) with illegal
ample discretion to determine whether there is a need to dismissal.Composite and Tan (respondents) alleged that
conduct further hearings after the parties have submitted their petitioners were both hired on 11 May 1999 as deliverymen,
position papers and supporting proofs. Petitioner's motion for initially for three months and then on a month-to-month
reconsideration was denied for lack of merit, hence this basis.Respondents alleged that petitioners termination from
petition alleging grave abuse of discretion on the part of the employment resulted from the expiration of their contracts of
NLRC in ordering the reinstatement of private respondent and employment on 8 October 1999.
the payment to him of one year back wages. The Labor Arbiter ruled that petitioners are regular
employees of respondents.Respondents appealed to the
ISSUE: Whether or not NLRC committed grave abuse of National Labor Relations Commission (NLRC). The NLRC
discretion set aside the Labor Arbiters Decision and dismissed
petitioners complaint for illegal dismissal. The NLRC ruled
RULING: The Court Finds adequate and compelling merit in that the mere fact that the employees duties are necessary or
the petition. The determinative issue in this case is whether or desirable in the business or trade of the employer does not
not the employment contract which stipulates that there is no mean that they are forbidden from stipulating the period of
employer-employee relationship between petitioner and employment. The NLRC held that petitioners contracts of
Peronila is valid. Relevant to this issue, we are persuaded to employment are valid and binding between the contracting
hold that the re-employment of Peronila as a contractual bus parties and shall be considered as the law between them. The
driver was merely an act of generosity on the part of NLRC ruled that petitioners are bound by their employment
petitioner. contracts.Petitioners filed a petition for certiorari before the
In the present dispute, the services of respondent Court of Appeals.The CA dismissed the petition and affirmed
Peronila had been validly terminated by petitioner, when the the NLRCs decision. Hence, this petition.
latter absented himself without official leave, fifteen years
before he was re-hired as a contractual driver for just one ISSUES: 1. Whether petitioners are regular employees of
month. Definitely, his re-hiring cannot be construed to mean respondents; and2. Whether respondents are guilty of illegal
that Peronila reacquired his former permanent status. dismissal.
Furthermore, as correctly pointed out by the Solicitor
General, "there is no evidence on record that private RULING: Petitioners are Not Regular Employees Under
respondent in fact held the position of a bus driver for nearly Article 280 of the Labor Code, a regular employee is (1) one
seventeen years, except his bare and unsupported allegations who is engaged to perform activities that are necessary or
to that effect in his Position Paper. . . . There is ample and desirable in the usual trade or business of the employer, or (2)
unrebutted evidence that private respondent's employment by a casual employee who has rendered at least one year of
PNEI in 1971 was illegally terminated on March 20, 1973. service, whether continuous or broken, with respect to the
The Order issued by the Mediator-Factfinder Loreto V. activity in which he is employed.
Poblete of the Regional Office No. II, National Labor The Court thus laid down the criteria under which fixed-
Relations Commission, Tuguegarao, Cagayan in the case term employment could not be said to be in circumvention
entitled, 'Pantranco vs. Rodolfo Peronila' docketed as NLRC of the law on security of tenure, thus:
Case No. 85, attests to this fact. 1. The fixed period of employment was
We once again reiterate that the findings of an knowingly and voluntarily agreed upon by the parties
administrative agency, to be conclusive and binding, must be without any force, duress, or improper pressure being
supported by substantial evidence. A conflict between the

Page 42
Dan Gloria// Kat Ramirez// Jessica Bernardo// Joseph De Mesa// Arvin Figueroa// ChriszelQueano// Marvin Villardo Jr.// Bowdy Sales
brought to bear upon the employee and absent any the decision to the National Labor Relations Commission
other circumstances vitiating his consent; or (NLRC) the NLRC affirmed the Labor Arbiter's decision.
2. It satisfactorily appears that the employer The petition having been denied, hence this petition.
and the employee dealt Petitioners contend that grave abuse of discretion amounting
with each other on more or less equal terms with to lack of jurisdiction by the NLRC.
no moral dominance exercised by the former or
the latter. ISSUE: Whether or not the NLRC committed grave abuse of
At most, petitioners employment for less than six months discretion.
can be considered probationary. Article 281 of the Labor Code
provides: RULING: No. Article 280 of the Labor Code defines regular
Art. 281. Probationary Employment. - Probationary and casual employment as follows: ART. 280. Regular and
employment shall not exceed six (6) months from the date the Casual Employment.-- The provisions of written agreement to
employee started working, unless it is covered by an the contrary notwithstanding and regardless of the oral
apprenticeship agreement stipulating a longer period. The argument of the parties, an employment shall be deemed to be
services of an employee who has been engaged on a regular where the employee has been engaged to perform
probationary basis may be terminated for a just cause or when activities which are usually necessary or desirable in the usual
he fails to qualify as a regular employee in accordance with business or trade of the employer, except where the
reasonable standards made known by the employer to the employment has been fixed for a specific project or
employee at the time of his engagement. An employee who is undertaking the completion or termination of which has been
allowed to work after a probationary period shall be determined at the time of the engagement of the employee or
considered a regular employee. where the work or services to be performed is seasonal in
Petitioners were not Illegally Dismissed from nature and the employment is for the duration of the season.
Employment Petitioners terms of employment are governed An employment shall be deemed to be casual if it is
by their fixed-term contracts. Petitioners fixed-term not covered by the preceding paragraph; Provided, That, any
employment contracts had expired. They were not illegally employee who has rendered at least one year of service,
dismissed from employment. This Court has ruled that if from whether such service is continuous or broken, shall be
the circumstances it is apparent that periods have been considered a regular employee with respect to the activity in
imposed to preclude acquisition of tenurial security by the which he is employed and his employment shall continue
employee, they should be disregarded for being contrary to while such activity exists.
public policy. In this case, it was not established that The NLRC was, thus, correct in finding that the
respondents intended to deny petitioners their right to security private respondents were regular employees and that they
of tenure. Besides, petitioners employment did not exceed six were illegally dismissed from their jobs. Under Article 279 of
months. Thus, the Court of Appeals did not err in sustaining the Labor Code and the recent jurisprudence,the legal
petitioners dismissal from employment. consequence of illegal dismissal is reinstatement without loss
of seniority rights and other privileges, with full back wages
59. Pure Foods Corp. v. NLRC computed from the time of dismissal up to the time of actual
G.R. No. 122653, Dec. 12, 1997 reinstatement, without deducting the earnings derived
elsewhere pending the resolution of the case.
FACTS: The private respondents (numbering 906) were hired However, since reinstatement is no longer possible because
by petitioner Pure Foods Corporation to work for a fixed the petitioner's tuna cannery plant had, admittedly, been closed
period of five months at its tuna cannery plant in Tambler, the proper award is separation pay equivalent to one month
General Santos City. After the expiration of their respective pay or one-half month pay for every year of service,
contracts of employment in June and July 1991, their services whichever is higher, to be computed from the commencement
were terminated. They forthwith executed a Release and of their employment up to the closure of the tuna cannery
Quitclaim stating that they had no claim whatsoever against plant. The amount of back wages must be computed from the
the petitioner.On 29 July 1991, the private respondents filed time the private respondents were dismissed until the time
before the National Labor Relations Commission (NLRC) petitioner's cannery plant ceased operation.
Sub-Regional Arbitration Branch No. XI, General Santos City,
a complaint for illegal dismissal against the petitioner and its 60. Universal Robina Corp. v. Catapang
plant manager, Marciano Aganon. G.R. No. 164736, Oct. 14, 2005
On 23 December 1992, Labor Arbiter Arturo P. Aponesto
handed down a decisiondismissing the complaint on the FACTS: Petitioner Universal Robina Corporation is a
ground that the private respondents were mere contractual corporation duly organized and existing under the Philippine
workers, and not regular employees; hence, they could not laws, while petitioner Randy Gregorio is the manager of the
avail of the law on security of tenure. The Labor Arbiter also petitioner companys duck farm in Calauan, Laguna. The
observed that an order for private respondents reinstatement respondents were hired by petitioner to work at a duck farm in
would result in the reemployment of more than 10,000 Calauan, Laguna on five month period contract. After
former contractual employees of the petitioner.Besides, by expiration of contract, petitioner would renew them to
executing a Release and Quitclaim, the private respondents reemploy them. The said practice continued until the
had waived and relinquished whatever right they might have respondents were informed that their contract were no longer
against the petitioner. The private respondents appealed from going to be renewed.

Page 43
Dan Gloria// Kat Ramirez// Jessica Bernardo// Joseph De Mesa// Arvin Figueroa// ChriszelQueano// Marvin Villardo Jr.// Bowdy Sales
Respondents filed separate complaints for illegal The petitioners received the CA Decision on
dismissal, reinstatement, backwages, damages and attorneys September 8, 2003; hence, they had until September 23, 2003
fees against the petitioners. The complaints were later within which to file a motion for reconsideration, or an appeal,
consolidated. through a petition for review, with this Court. Instead, the
After due proceedings, the Labor Arbiter rendered a petitioners filed a motion for extension of time to file a motion
decision in favor of the respondents. On May 17, 1999, the for reconsideration on September 23, 2003, which is a
petitioners filed an Appeal Memorandum with the National prohibited pleading. Thus, it did not suspend the running of
Labor Relations Commission (NLRC) on the ground that the the period for filing an appeal.
Labor Arbiter erred in ruling that the respondents are the In filing their petition for review with this Court, the
petitioner companys regular employees.Meanwhile, on May petitioners counted the 15-day period from their receipt of the
18, 1999, the respondents filed a Motion for Enforcement of July 19, 2004 CA Resolution on August 4, 2004. Hence,
Reinstatement Order with the Labor Arbiter. On June 3, 1999, according to their Motion for Extension of Time to File
the latter issued an Order, which states that the respondents Petition for Review which they filed on August 19, 2004, they
should comply with the reinstatement in good faith. On June had until that day within which to file a petition for review.
21, 1999, the Labor Arbiter issued a Writ of Execution They then asked the Court that they be granted an extension of
enforcing the immediate reinstatement of the respondents as 30 days, or until September 21, 2004 within which to file their
mandated in the March 30, 1999 Decision. petition. The Court granted the motion on the belief that the
However only 17 employees were reinstated because petitioners motion for reconsideration before the CA was duly
there were no other available positions substantially similar to filed and that the assailed July 19, 2004 CA Resolution had
the positions previously occupied by the other 13 respondents, denied the said motion. Thereafter, the petitioners filed their
but that 10 of them could be accommodated at the farms Duck petition for review on September 20, 2004. It is, therefore,
Dressing Section which operates at an average of three days a evident from the foregoing that the present petition was filed
week only.On February 21, 2000, the respondents moved for way beyond the reglementary period. Hence, its outright
the issuance of a notice of garnishment to collect the dismissal would be proper. The perfection of an appeal in the
accumulated withheld wages of the 17 respondents who were manner and within the period prescribed by law is not only
reinstated on December 16, 1999 amounting to P649,400.00. mandatory but jurisdictional, and failure to perfect an appeal
The Labor Arbiter granted the motion and issued a Second has the effect of rendering the judgment final and executory.
Alias Writ of Execution directing the Sheriff to proceed to
collect the said amount plus execution fees.Petitioners filed a 61. Viernes v. NLRC
motion to quash. The Labor arbiter later on denied the G.R. No. 108405, April 4, 2003
petitioners petition.On March 13, 2000, the petitioners filed a
Memorandum and Notice of Appeal with Prayer for the FACTS: Fifteen (15) in all, these are consolidated cases for
Issuance of a Temporary Restraining Order [15] with the NLRC, illegal dismissal, underpayment of wages and claim for
assailing the February 9, 2000 and March 1, 2000 Orders and indemnity pay against a common respondent, the Benguet
the two Alias Writs of Execution issued by the Labor Electric Cooperative, Inc., (BENECO for short) represented
Arbiter.On November 22, 2000, the NLRC affirmed the by its Acting General Manager, Gerardo P.
decision of the Labor Arbiter with the modification that the Versoza.Complainants services as meter readers were
award of attorneys fees was reduced to 10% of the total contracted for hardly a months duration, or from October 8 to
monetary award.Aggrieved, the petitioners filed a petition 31, 1990. Their employment contracts, couched in identical
for certiorari with thehold Court of Appeals (CA). On August terms, read:You are hereby appointed as METER READER
21, 2003, the CA denied the petition for lack of merit. Holding (APPRENTICE) under BENECO-NEA Management with
that thePetitioners were regular emlployees. compensation at the rate of SIXTY-SIX PESOS AND
SEVENTY-FIVE CENTAVOS (P66.75) per day from
ISSUE: Whether the petition was filed within the October 08 to 31, 1990.The said term notwithstanding, the
reglamentary period complainants were allowed to work beyond October 31, 1990,
or until January 2, 1991. On January 3, 1991, they were each
Ruling: Clearly, the period for filing a motion for served their identical notices of termination dated December
reconsideration and a petition for review with this Court are 29, 1990.The complainants filed separate complaints for
the same, that is, 15 days from notice of the judgment. When illegal dismissal.The contention of the complainants that they
an aggrieved party files a motion for reconsideration within were not apprentices but regular employees whose services
the said period, the period for filing an appeal is suspended. If were illegally and unjustly terminated in a manner that was
the motion is denied, the aggrieved party is given another 15- whimsical and capricious.
day period from notice of such denial within which to file a On the other hand, the respondent invokes Article 283 of
petition for review under Rule 45. It must be stressed that the the Labor Code in defense of the questioned dismissal. The
aggrieved party will be given a fresh 15-day period only when labor arbiter dismissed the complaints for illegal dismissal
he has filed his motion for reconsideration in due time on or filed by the complainants for lack of merit. Aggrieved by the
before the expiration of the original 15-day period. Otherwise, LaborArbiters decision, the complainants and the respondent
if the motion for reconsideration is filed out of time and no filed their respective appeals to the NLRC. The NLRC
appeal has been filed, the subject decision becomes final and modified its judgment declaring complainants dismissal
executory.[34] As such, it becomes immutable and can no illegal, thus ordering their reinstatement to their former
longer be attacked by any of the parties or be modified, position as meter readers or to any equivalent position with
directly or indirectly, even by the highest court of the land. payment of backwages limited to one year and deleting the
Page 44
Dan Gloria// Kat Ramirez// Jessica Bernardo// Joseph De Mesa// Arvin Figueroa// ChriszelQueano// Marvin Villardo Jr.// Bowdy Sales
award of indemnity and attorneys fees.On August 27, 1992, effecting the dismissal of petitioners from their employment,
complainants filed a Motion for Clarification and Partial failed to comply with the provisions of Article 283 of the
Reconsideration On September 24, 1992, the NLRC issued a Labor Code which requires an employer to serve a notice of
resolution denying the complainants motion for dismissal upon the employees sought to be terminated and to
reconsideration. Hence this petition. the Department of Labor, at least one month before the
intended date of termination. Hence NLRC committed grave
ISSUES: Whether the respondent NLRC committed grave abuse of discretion in deleting the award of indemnity.
abuse of discretion in ordering the reinstatement of petitioners
to their former position as meter readers on probationary status 62. Megascope General Services v. NLRC
in spite of its finding that they are regular employees under G.R. No. 109224, June 19, 1997
Article 280 of the Labor Code.
Whether the respondent NLRC committed grave FACTS: Petitioner Megascope General Services is a sole
abuse of discretion in limiting the backwages of petitioners to proprietorship engaged in contracting out general services. In
one year only in spite of its finding that they were illegally 1977, it entered into a landscaping contract with the System
dismissed, which is contrary to the mandate of full backwages and Structures, Inc. (SSI) which subcontracted the
until actual reinstatement but not to exceed three years. construction of the National Power Corporation Housing
Whether the respondent NLRC committed grave Village in Bagac, Bataan. Between February 15, 1977 and
abuse of discretion in deleting the award of indemnity pay January 1, 1989, it contracted the services of the nineteen (19)
which had become final because it was not appealed and in private respondents as gardeners, helpers and maintenance
deleting the award of attorneys fees because of the absence of workers. They were deployed at the National Power
a trial-type hearing. Corporation (NPC) in Bagac, Bataan. Except for Gener J. del
Rosario whose employment ended on April 30, 1989, the work
RULING: First Issue: The Court sustain petitioners claim of the other workers ceased on January 31, 1991. At the time,
that they should be reinstated to their former position as meter private respondents Nestor Seril and Gener J. del Rosario were
readers, not on a probationary status, but as regular receiving P65.00 a day; private respondent Reynaldo
employees. Reinstatement means restoration to a state or Magtanong, P56.00 a day, and the rest of the private
condition from which one had been removed or separated. In respondents, P54.00 a day.
case of probationary employment, Article 281 of the Labor Private respondents filed before Regional
Code requires the employer to make known to his employee at Arbitration Branch No. III in San Fernando, Pampanga, a
the time of the latters engagement of the reasonable standards complaint for illegal dismissal, underpayment of
under which he may qualify as a regular employee. A review salaries, nonpaymentof five-day service incentive leave credits
of the records shows that petitioners have never been and holiday pay against petitioner and Andres M. David.
probationary employees. While it is true that initially they On October 7, 1992, Labor Arbiter Ariel C. Santos
were employed on a fixed term basis, later on they were promulgated his decision finding that, by the nature of their
allowed to work beyond their employment period which employment, private respondents were "usually contractual
renders them to be regular employees. Under Article 280 of employees." Nonetheless, he opined, in view of the length of
the Labor Code, a regular employee is one who is engaged to their service, that private respondents had attained the status of
perform activities which are necessary or desirable in the usual "regular contractual employees" who, pursuant to Policy
business or trade of the employer, or a casual employee who Instruction No. 20, "cannot just be terminated after the
has rendered at least one year of service, whether continuous expiration of a contract in an area to where they are assigned
or broken, with respect to the activity in which he is without paying them the corresponding separation pay from
employed. the time they have served respondent's company." He also
Second Issue: Article 279 provides that an illegally held that since private respondents' termination of
dismissed employee is entitled to full backwages, inclusive of employment was the result of the expiration of petitioner's
allowances, and to his other benefits or their monetary contract with the NPC, there was no unlawful dismissal.
equivalent computed from the time his compensation was Respoents were directed to pay separation pay based on one-
withheld from him up to the time of his actual half month for every year of service. The other monetary
reinstatement. Since petitioners were employed on October 8, claims were dismissed for lack of merit. Petitioner and David
1990, the amended provisions of Article 279 of the Labor appealed to the National Labor Relations Commission
Code shall apply to the present case. Hence, it was patently (NLRC). On November 20, 1992, the NLRC, affirmed the
erroneous, tantamount to grave abuse of discretion on the part decision of the Labor Arbiter. Hence this petition.
of the public respondent in limiting to one year the backwages
awarded to petitioners. ISSUE/S:1.Whether or not there was grave abuse of discretion
Third Issue: The Court does not agree with the made by the NLRC in its decision. ; Whether or not private
ruling of the NLRC that indemnity is incompatible with the respondents were regular employees of petitioner.
award of backwages. These two awards are based on different
considerations. Backwages are granted on grounds of equity to RULING: 1. No. Petitioner's allegations are premised on its
workers for earnings lost due to their illegal dismissal from stand that the complaint was not supported by facts. It avers
work. On the other hand, the award of indemnity, as we have that the Labor Arbiter should have conducted a trial on the
earlier held, is meant to vindicate or recognize the right of an merits of the case in order that the parties could fully ventilate
employee to due process which has been violated by the their respective sides of the controversy. the determination of
employer. In the present case, the private respondent, in the existence of an employer-employee relationship consists of
Page 45
Dan Gloria// Kat Ramirez// Jessica Bernardo// Joseph De Mesa// Arvin Figueroa// ChriszelQueano// Marvin Villardo Jr.// Bowdy Sales
these four (4) elements: (1) the selection and engagement of considering that private respondents have been the former's
the employee; (2) the payment of wages; (3) the power of regular employees for periods ranging from two to fourteen
dismissal, and (4) the power to control the employee's years. This is the correct basis of the award of separation pay
conduct, All these elements are present in this case. They drew to private respondents.
their salaries from petitioner which eventually dismissed
them. Petitioner's control over private respondents was 63. AgusandelNorte Electric Coop, Inc. v. Cagampang and
manifest in its power to assign and pull them out of clients at Garzon
its own discretion. Power of control refers merely to the G.R. No. 167627, Oct. 10, 2008
existence of the power and not to the actual exercise thereof.
Based on the recorded admission, the petitioner cannot claim FACTS: Respondents Joel Cagampang and Glenn Garzon
that the decision of the NLRC with regard to the existence of started working as linemen for petitioner AgusandelNorte
employee employer relationship was based on conjuncture. Electric Cooperative, Inc. (ANECO) on October 1, 1990,
Moreover, proceedings before administrative and quasi- under an employment contract which was for a period not
administrative bodies, substantial evidence is sufficient to exceeding three months. They were both allegedly required to
establish a fact in issue. Hence, grave abuse of discretion work eight hours a day and sometimes on Sundays, getting a
cannot be attributed to the public respondents for holding that daily salary of P122.00. When the contract expired, the two
an employee employer relationship did exist between were laid-off for one to five days and then ordered to report
petitioner and private respondents. back to work but on the basis of job orders. After several
2. On whether or not private respondents were renewals of their job contracts in the form of job orders for
regular employees of petitioner, the Labor Code provides: similar employment periods of about three months each, the
"ART. 280. Regular and Casual Employment.- The provisions said contracts eventually expired on April 31, 1998 and July
of written agreement to the contrary notwithstanding and 30, 1999. Respondents contracts were no longer renewed,
regardless of the oral agreement of the parties, an employment resulting in their loss of employment. Thus, on January 11,
shall be deemed to be regular where the employee has been 2001, respondents filed an illegal dismissal case against
engaged to perform activities which are usually necessary or petitioners before the HonorableLabor Arbiter Alim D.
desirable in the usual business or trade of the employer, except Pangandaman, Regional Arbitration Branch No. XIII, Butuan
where the employment has been fixed for a specific project or City. They prayed for payment of backwages, salary
undertaking the completion or termination of which has been differential, allowances, premium for alleged work during
determined at the time of the engagement of the employee or holidays and rest days,service incentive leave, and separation
where the work or services to be performed is seasonal in pay.
nature and the employment is for the duration of the season. On June 22, 2001, the Labor Arbiter declared the
An employment shall be deemed to be casual if it is dismissal illegal and directed petitioners to pay respondents
not covered by the preceding paragraph: Provided, That, any the amount of P371,596.84 representing their money claims.
employee who has rendered at least one year of service, Petitioners appealed the decision to the NLRC Fifth Division.
whether such service is continuous or broken shall be On October 14, 2002, the NLRC set aside the decision of the
considered a regular employee with respect to the activity in Labor Arbiter except the portions granting service incentive
which he is employed and his employment shall continue leave pay attorneys fees, fixed at ten percent (10%) of the
while such actually exists." total money award to both respondents; and salary differential
The cessation of the NPC operations in Bagac and to respondent Garzon.After their motion for reconsideration
Morong did not in any way affect the employer-employee was denied, respondents filed a petition for certiorari with the
relationship between petitioner and private respondents. While Court of Appeals. On January 25, 2005, the Court of Appeals
this might have severed the contractual relationship between granted the petition. Hence, the instant petition.
petitioner and the NPC, as regards the supply of services, it
did not and will not, under the law, affect the employer- ISSUE: Whether or not the respondents were illegally
employee relationship between petitioner and private dismissed.
respondents.Petitioner asserts that private respondents were
not prejudiced because all along they knew that they were RULING: Yes. After considering the facts and the
employed for the specific NPC "project" and therefore when submissions of the parties the Court finds that the respondents
the NPC ceased operations in Bagac and Morong, private were illegaly dismissed. There is no dispute that the
respondents knew that their employment perforce ceased. respondents work as linemen was necessary or desirable in
Petitioner's assertion would have been tenable if it were the usual business of ANECO. Additionally, the respondents
successfully established that private respondents were indeed have been performing the job for at least one year. The law
project employees. By not redeploying private respondents to deems the repeated and continuing need for its performance as
other clients, there being no evidence that petitioner itself had sufficient evidence of the necessity, if not indispensability, of
ceased its business operations, petitioner is deemed to have that activity to the business.
constructively dismissed private respondents. Constructive The test to determine whether employment is regular
dismissal exists when there is a quitting because continued or not is the reasonable connection between the particular
employment is rendered impossible, unreasonable or activity performed by the employee in relation to the usual
unlikely. In the instant case, the failure of petitioner to business or trade of the employer. Also, if the employee has
redeploy private respondents to other projects without been performing the job for at least one year, even if the
showing that it had ceased its operations after the termination performance is not continuous or merely intermittent, the law
of its contract with NPC may be deemed unreasonable deems the repeated and continuing need for its performance as
Page 46
Dan Gloria// Kat Ramirez// Jessica Bernardo// Joseph De Mesa// Arvin Figueroa// ChriszelQueano// Marvin Villardo Jr.// Bowdy Sales
sufficient evidence of the necessity, if not indispensability of The respondent filed at the NLRC for illegal
that activity to the business. Thus, we held that where the dismissal against the petitioner. Labor arbiter dismissed the
employment of project employees is extended long after the complaint for lack of merit. NLRC affirmed the decision of
supposed project has been finished, the employees are the Labor arbiter. The NLRC explained that the respondent
removed from the scope of project employees and are was a contractual employee whose period of employment was
considered regular employees. fixed in the successive contracts of employment she had
While length of time may not be the controlling test executed with the petitioner. Thus, upon the expiration of her
for project employment, it is vital in determining if the contract, the respondents employment automatically
employee was hired for a specific undertaking or tasked to ceased. The respondents employment was not terminated;
perform functions vital, necessary and indispensable to the neither was she dismissed. The CA rendered a decision
usual business or trade of the employer. Here, private reversing the decisions of the NLRC and the Labor Arbiter
respondent had been a project employee several times over. and granting the respondents petition.Hence, this petition.
His employment ceased to be coterminous with specific
projects when he was repeatedly re-hired due to the demands ISSUE: Whether or not the respondent was still a contractual
of petitioners business. Where from the circumstances it is employee of the petitioner as of June 4, 1993.
apparent that periods have been imposed to preclude the RULING: In ruling for the respondent, the appellate court
acquisition of tenurial security by the employee, they should applied Article 280 of the Labor Code of the Philippines, as
be struck down as contrary to public policy, morals, good amended, which reads:
customs or public order. Art. 280. Regular and Casual Employment. The provisions of
Respondents in the present case being regular written agreement to the contrary notwithstanding and
employees, ANECO as the employer had the burden of proof regardless of the oral argument of the parties, an employment
to show that the respondents termination was for a just cause. shall be deemed to be regular where the employee has been
Unfortunately, however, what petitioners did was merely to engaged to perform activities which are usually necessary or
refuse, without justifiable reason, to renew respondents work desirable in the usual business or trade of the employer, except
contracts for the performance of what would otherwise be where the employment has been fixed for a specific project or
regular jobs in relation to the trade or business of the former. undertaking the completion or termination of which has been
Such conduct dismally falls short of the requirements of our determined at the time of the engagement of the employee or
labor laws egarding dismissals. No twin notices of termination where the work or services to be performed is seasonal in
were issued to the employees, hence the employer did not nature and the employment is for the duration of the season.
observe due process in dismissing them from their An employment shall be deemed to be casual if it is
employment. Their dismissals were patently illegal. not covered by the preceding paragraph; Provided, That, any
employee who has rendered at least one year of service,
64. Philips Semiconductors v. Fadriquela whether such service is continuous or broken, shall be
G.R. No. 141717, 04.14, 2004 considered a regular employee with respect to the activity in
which he is employed and his employment shall continue
FACTS: Respondent Eloisa Fadriquela executed a Contract of while such activity exists.
Employment with the petitioner in which she was hired as a
production operator with a daily salary of Php118. Her initial The Court agrees with the appellate court. In this
contract was for a period of three months up to August 8, case, the respondent was employed by the petitioner on May
1992, but was extended for two months when she garnered a 8, 1992 as production operator. She was assigned to wire
performance rating of 3.15 Her contract was again renewed building at the transistor division. There is no dispute that the
for two months or up to December 16, 1992, when she work of the respondent was necessary or desirable in the
received a performance rating of 3.8. After the expiration of business or trade of the petitioner. She remained under the
her third contract, it was extended anew, for three months, that employ of the petitioner without any interruption since May 8,
is, from January 4, 1993 to April 4, 1993. 1992to June 4, 1993 or for one (1) year and twenty-eight (28)
After garnering a performance rating of 3.4 the days. The original contract of employment had been extended
respondents contract was extended for another three months, or renewed for four times, to the same position, with the same
that is, from April 5, 1993 to June 4, 1993. She, however, chores. Such a continuing need for the services of the
incurred five absences in the month of April, three absences in respondent is sufficient evidence of the necessity and
the month of May and four absences in the month of June. indispensability of her services to the petitioners business By
Line supervisor Shirley F. Velayo asked the respondent why operation of law, then, the respondent had attained the regular
she incurred the said absences, but the latter failed to explain status of her employment with the petitioner, and is thus
her side. The respondent was warned that if she offered no entitled to security of tenure as provided for in Article 279 of
valid justification for her absences, Velayo would have no the Labor Code.
other recourse but to recommend the non-renewal of her
contract. The respondent still failed to respond, as a
consequence of which her performance rating declined to
2.8. Velayo recommended to the petitioner that the
respondents employment be terminated due to habitual
absenteeism, in accordance with the Company Rules and
Regulations. Thus, the respondents contract of employment
was no longer renewed.
Page 47
Dan Gloria// Kat Ramirez// Jessica Bernardo// Joseph De Mesa// Arvin Figueroa// ChriszelQueano// Marvin Villardo Jr.// Bowdy Sales

You might also like